1. Trang chủ
  2. » Hoá học lớp 12

Tạp chí Epsilon số 17

182 26 0

Đang tải... (xem toàn văn)

Tài liệu hạn chế xem trước, để xem đầy đủ mời bạn chọn Tải xuống

THÔNG TIN TÀI LIỆU

Nội dung

Sau khi trình bày phác thảo của tác phẩm, trong đó tác giả đặt ra để phân định kiến trúc của kiến thức con người, cho thấy các nguyên tắc làm nền tảng cho các lĩnh vực kiến thức khác nha[r]

(1)

SI

2 mé

t

2 mé

2 mé

t

2 mé

t

Dạy robot đánh cờ hộp diêm - Võ Bích Khuê, Nguyễn Hùng Sơn Mở rộng bất đẳng thức hình học Finsler – Hadwiger - Trần Quang Hùng

Vắc-xin bệnh đậu mùa: Cuộc tranh luận Bernoulli d’Alembert tính tốn xác suất - Camila Colombo, Mirko Diamanti

“Trong cánh đồng Galois tràn đầy bơng hoa, ngun thủy nhảy nhót tiếng đồng hồ”

Mã kỳ thi Olympic Toán (phần cuối)

S B Gashkov

“Hai hàm ngược có tính chất gì? Tính chất f (x) = y g(y) = x, tức đồ thị hai hàm ngược đối xứng với qua đường thẳng y = x”

Phương pháp gián tiếp giải toán định lượng

Trần Nam Dũng

(2)

2 mé

2 mé

t

2 mét

Lê Viết Ân

Lê Phúc Lữ

Tống Hữu Nhân

Nguyễn Tất Thu

Võ Quốc Bá Cẩn

Trần Quang Hùng

Ngô Quang Dương

(3)

Các bạn cầm máy tính số Epsilon, tạp chí online người yêu Toán: Epsilon17

Epsilon17được khởi động xuất xưởng bối cảnh đặc biệt đại dịch COVID-19

xuất phát từ Vũ Hán (Trung Quốc) lan bùng nổ khắp toàn cầu Và ảnh hưởng vơ lớn lao, lớn kể từ sau Chiến tranh giới lần thứ hai Lần nghe cụm từ tình trạng khẩn cấp, cách ly tồn diện, giãn cách xã hội sử dụng cách phổ dụng toàn giới Cuộc sống offline có thay đổi lớn, chắn khơng cịn cũ nữa, đại dịch qua sống trở lại bình thường Sẽ có nhiều quy tắc, chuẩn mực, thói quen, hành vi xem xét dần loại bỏ Và chắn hoạt động online cổ vũ phát triển bời nó: Tiện lợi hơn, hiệu hơn, tiết kiệm an tồn

Vì lẽ đó, Epsilon tiếp tục tự tin đường để tiếp tục diễn đàn trực tuyến tự chất lượng người yêu toán, muốn chia sẻ đến bạn đọc nghiên cứu, khám phá, chiêm nghiệm tốn học vấn đề liên quan

Epsilon17là tuyển tập15bài viết với nội dung thể loại phong phú, dành cho nhiều đối tượng,

kể từ em học sinh tiểu học đến người yêu toán lớn tuổi Từ học sinh chuyên đến học sinh không chuyên, từ sinh viên đại học đến học viên cao học tìm thấy điều bổ ích phù hợp cho số báo Ban biên tập trân trọng cảm ơn tất tác giả đóng góp cho Tạp chí mong tiếp tục nhận cộng tác quý báu số Ban biên tập Epsilon trân trọng cảm ơn GS Đàm Thanh Sơn quan tâm dõi theo, tìm chọn cho chúng tơi báo viết tranh luận Bernoulli D’Alembert hồi cuối kỷ 18 xác suất ứng dụng vào mơ hình nghiên cứu hiệu tiêm chủng, viết phù hợp với bối cảnh ngày

Mặc dù cố gắng tối đa, sai sót khơng tránh khỏi Chúng mong bạn đọc phản hồi cho sai sót để kịp thời chỉnh sửa

(4)

Camila Colombo, Mirko Diamanti

Vắc-xin bệnh đậu mùa: Cuộc tranh luận Bernoulli d’Alembert tính tốn xác suất 5

Nguyễn Lê Anh

Sự tích "Trâu Vàng", "Cáo Chín Đi" Hồ Tây (Phần 2) 20

Võ Bích Khuê, Nguyễn Hùng Sơn

Dạy robot đánh cờ hộp diêm 37

S B Gashkov

Mã kỳ thi Olympic Toán (phần cuối) 42

Benny Lê Văn

Phép tổng hợp biểu thức (resultant) số ứng dụng 50

Lương Văn Khải, Võ Thành Đạt

Bài tốn nhân bền vững 62

Trần Nam Dũng

Phương pháp gián tiếp giải toán định lượng 76

Đào Xuân Luyện

Một số toán dùng tính khơng bị chặn đa thức 87

Lê Phúc Lữ

Một số toán chọn lọc phương trình hàm 99

Ngô Văn Thái

Một số bất đẳng thức kỳ thi Olympic Toán học Quốc tế 111

Nguyễn Tất Thu

Các tốn cực trị khơng gian toạ độ 127

Trần Quang Hùng

Mở rộng bất đẳng thức hình học Finsler – Hadwiger 151

Nguyễn Ngọc Giang, Lê Viết Ân

Một mở rộng đường thẳng Steiner 157

Ban Biên tập

Bài toán hay - Lời giải đẹp 163

BTC kỳ thi Thách Thức Toán Học

(5)

V

ẮC

-

XIN

B

ỆNH

Đ

ẬU

M

ÙA

C

UỘC

T

RANH

L

UẬN

G

IỮA

B

ERNOULLI

V

À

D

ALEMBERT

C

ÙNG

T

ÍNH

T

ỐN

X

ÁC

S

UẤT

Camila Colombo (UK), Mirko Diamanti (Italy)

Người dịch Nguyễn Vũ Duy Linh, Trần Nam Dũng

LỜI

BAN

BIÊN

TẬP

Đại dịch COVID 19vẫn hoành hành thu hút quan tâm toàn nhân loại

Trên tuyến đầu y, bác sĩ chống chọi trực tiếp với virus nguy hiểm để giành giật sống, bảo vệ cho bệnh nhân, quyền lực lượng y tế cộng đồng, quân đội, cảnh sát, triển khai hoạt động cách ly, giãn cách xã hội, nhà khoa học gấp rút nghiên cứu thử nghiệm vắc-xin ngừa virus thuốc chữa virus Và cịn hàng trăm, hàng ngàn cơng việc liên quan khác Các nhà tốn học khơng ngồi cuộc, phương pháp thống kê, họ có phân tích, dự đốn để từ đưa khuyến cáo thích hợp cho nhà hoạch định sách, để lựa chọn phương án chống dịch cách hiệu Được giới thiệu GS.Đàm Thanh Sơn (ĐH Chicago), chọn dịch viết Camila Colombo Mirko Diamanti tranh luận cách hơn200năm Daniel Bernoulli Jean Le Rond d’Alembert

việc tiêm chủng bệnh đậu mùa ứng dụng xác suất nghiên cứu vấn đề thực tế Chúng ta thấy câu chuyện hơn200năm trước có vấn đề

rất gần gũi với thực tiễn chống dịch COVID 19trên giới hôm

1 Tổng quan

Nhu cầu cấp thiết kịch tính nhằm giới thiệu quảng bá vắc-xin chống bệnh đậu mùa, tai họa cho xã hội vào cuối năm1700;đã tạo hội cho tranh luận sôi

(6)

vắc-xin hợp lý d’Alembert, người phản bác lại lập luận Mục đích phân tích cho thấy “nghệ thuật đốn” tìm đối tượng diễn giải khác nhau, từ việc mô tả nguyên tắc lý thuyết nó, khái niệm kỳ vọng, câu hỏi tính hợp pháp ứng dụng “hướng dẫn cách sống thực tế”, sau diễn đạt lại đóng góp hai tác giả

2 Dẫn nhập

Từ nửa đầu kỷ XVII, vấn đề dịch bệnh đậu mùa coi cấp bách kịch tính Ở thành phố có mật độ dân số cao, chẳng hạn London Paris, người ta ước tính khoảng mười phần trăm tất trường hợp tử vong bệnh đậu mùa Hơn hết, trẻ em thiếu niên dễ bị mắc bệnh loại virus khơng khí gây ra, lúc bệnh gây tử vong, thường khiến nạn nhân bị sẹo nặng mù vĩnh viễn Do lây lan rộng rãi tỷ lệ phần trăm dân số bị ảnh hưởng, bệnh đậu mùa coi tai họa xã hội biểu tượng bất lực y học Do đó, phương pháp phịng chữa bệnh thu hút ý tập trung nhà khoa học công chúng

Mặc dù chưa có phương pháp điều trị hiệu biết đến, có thời châu Á, người ta tiến hành kỹ thuật tiêm chủng thô sơ, gọi chủng đậu, cách đưa virus vào người thông qua việc tiêm truyền chất bị nhiễm virus Cách làm giới thiệu Anh vào năm 1718bởi Lady Montague, vợ đại sứ Anh Đế chế Ottoman Tuy nhiên,

rủi ro liên quan đến phương pháp điều trị đáng kể, thường người bị tiêm nhiễm mắc bệnh chết vòng2tháng Hơn nữa, việc tiêm vắc-xin gây sốt thấp

như truyền nhiễm thấp: Do đó, bệnh nhân phải vài tuần sau tiêm, làm cho trở nên thiết thực trừ tầng lớp xã hội giàu có Mặc dù nước Pháp có tỷ lệ mắc bệnh cao tầng lớp xã hội, vắc-xin xem xét với hoài nghi Trên thực tế, người ta lo ngại kỹ thuật góp phần vào việc lây lan bệnh truyền nhiễm mang lại lợi ích đáng ngờ Tóm tắt tình hình Voltaire trình bày Những Lá Thư Triết Học biểu tượng dè dặt chung:

Tại quốc gia Kitô giáo châu Âu, người ta khẳng định vô tư người Anh kẻ ngốc điên Ngốc, họ cho họ nhiễm đậu mùa để ngăn bệnh và điên, họ cố tình gây bất an rối loạn đáng sợ chắn cho họ để ngăn chặn tai họa không chắn xảy ([7], Thư XI)1

1On dit doucement, dans l’Europe chrétienne, que les Anglais sont des fous et des enragés: Des fous, parcequ’ils

(7)

Do đó, bối cảnh sôi người quan tâm cấp bách, giới trí thức tiến hành chiến dịch văn hóa để vận động tiêm chủng, chiến dịch thường xem nội dung trận chiến thực ý thức hệ nhân danh tiến ([2], pp.83 89) Từ

tổ chức đến tờ báo, vấn đề ủng hộ hay chống lại tiêm chủng đối tượng tranh luận thường xuyên từ năm 1750đến1770;với đóng góp đầy nhiệt huyết nhiều trí thức

Pháp Voltaire, người sống sót sau mắc bệnh đậu mùa năm1723;sau quan sát trực tiếp

một số trường hợp tiêm chủng Anh, nhiệt tình tuyên bố ủng hộ Những Lá Thư Triết Học năm 1743:La Condamine đặt câu hỏi chủ đề nói chuyện với Viện Hàn Lâm

Khoa Học Paris vào năm1754:Mục tiêu người ủng hộ tiêm chủng nhà chức

trách phải xem việc tiêm chủng bắt buộc, cơng khai khuyến khích nó, cách vượt qua chống đối kẻ ngu dân thù địch với tiến

Trong bầu khơng khí hào hứng thập tự chinh này, Daniele Bernoulli Jean Baptiste Le Rond d’Alembert giữ vị trí đối lập cân nhắc rủi ro lợi ích việc tiêm phịng Năm1760;một báo Bernoulli xuất tờ Mércure de France,

trong ơng đưa kết thu cách áp dụng giải tích cho xác suất để trả lời câu hỏi vắc-xin Một phân tích định lượng vậy, theo ý kiến tác giả, chứng minh cho “một người có lương tri” cần thiết tiêm chủng Cùng năm đó, Bernoulli trình bày chi tiết tính tốn cho Viện Hàn Lâm Khoa Học, báo có tựa đề “Luận văn cách phân tích tỷ lệ tử vong bệnh đậu mùa lợi ích việc tiêm phịng để ngăn chặn nó”, không công bố năm1766:Trả lời D’Alembert can

thiệp Bernoulli, có “Kỷ yếu thứ mười một, Về việc áp dụng tính tốn xác suất trong tiêm phịng bệnh đậu mùa”: Đề xuất Bernoulli bị trích gay gắt trước xuất văn cuối cùng, dẫn đến tranh luận phàn nàn thiếu tôn trọng đồng nghiệp Bernoulli

Vượt trận bút chiến tranh luận, khẳng định d’Alembert thú vị chúng chuyển câu hỏi sang mức độ tổng quát giải thích khái niệm xác suất tiêu chuẩn có nhược điểm tương đối việc áp dụng phép tính giải tích Chính xác hàm ý lời trích [của d’Alembert] cơng trình Bernoulli đề kiểm tra sâu sắc từ quan điểm lý thuyết d’Alembert Trong nỗ lực đưa đóng góp với tư cách nhà bách khoa toàn thư sở đắc xác suất hệ thống kiến thức nhân loại, dự định lý lẽ nhận thức luận mà ơng ta sử dụng hồn tất việc giới hạn lĩnh vực ứng dụng xác suất bối cảnh hoàn toàn trừu tượng, từ bỏ khả góp phần vào việc kiểm sốt khơng chắn

(8)

trong đời thường

3 Đề xuất Daniel Bernoulli

Năm1759;Maupertuis, người nhiệt tình ủng hộ tiêm chủng, thuyết phục Daniel Bernoulli,

đồng nghiệp ông Basel, dành tâm trí cho phân tích tốn học vấn đề vắc-xin Giải pháp cho vấn đề này, theo Bernoulli, phải có dạng giải đáp cho câu hỏi sau: Chính phủ có nên thúc đẩy việc tiêm chủng cho cá nhân chào đời hay khơng? Mục đích tính tốn là, đưa xác suất bị bệnh tử vong bệnh đậu mùa tiêm vắc-xin, so sánh rủi ro lợi ích hai chiến lược Khi khó khăn thiếu liệu độ tin cậy liệu không cao khắc phục, Bernoulli tin phân tích nghiêm túc giúp phủ xác định nên theo chiến lược Kết luận tác giả giải đáp tích cực cho câu hỏi ban đầu: Nên khuyến khích tiêm chủng dẫn đến tăng tuổi thọ trung bình

Chúng ta tóm tắt lý luận Bernoulli ba bước:

1 Xây dựng mơ hình dân số, nghĩa đường cong mô tả thay đổi dân số theo thời gian (tỷ lệ tử vong)

2 Tính tốn thay đổi tuổi thọ với điều kiện ban đầu bệnh đậu mùa loại bỏ (nghĩa là, tất cá nhân tiêm phòng sinh)

3 Tính tốn thay đổi tuổi thọ xét đến nguy tử vong tiêm chủng

Một cách ngắn gọn, Bernoulli đề nghị so sánh hai “tình trạng nhân loại” khác diện bệnh đậu mùa Tiêu chuẩn để so sánh đường cong dân số mơ tả hai trạng thái tích phân đường cong theo tuổi thọ trung bình Điều kiện cần để thực thao tác so sánh cô lập yếu tố tử vong bệnh đậu mùa với tỷ lệ tử vong tồn phần

4 Mơ hình

(9)

Những người bị nhiễm bệnh đậu mùa lần chết với xác suấtpvà sống sót với xác

suất1 p:

Mỗi cá nhân có xác suất bị nhiễm năm làq:Trong khoảng thời gian vô bé dx;xác suất bị nhiễm tuổix tuổixCdx(vớidx D1cho đơn giản) làqdx:

Những người sống sót sau mắc bệnh đậu mùa miễn dịch phần lại đời họ

Dùng m.x/ tỷ lệ tử vong nguyên nhân khác với bệnh đậu mùa đại lượng tương

ứngm.x/dx;Bernoulli xây dựng mơ hình dân số phù hợp tỷ lệ tử vong bệnh

[đậu mùa] tách biệt ĐặtP0là nhóm cá nhân sinh năm:S.x/là số người,

có nguy nhiễm bệnh, sống chưa bị bệnh đậu mùa tính tuổi x; R.x/

số người sống sót sau mắc bệnh đậu mùa tuổi x; ta có P x/ D S.x/ CR.x/

là số người sống tuổi x:Bernoulli sau chox D tương ứng với lúc chào đời,

đó S.0/ D P 0/ D P0 R.0/ D Giữa x x Cdx, cá nhân có nguy bị nhiễm

qdxvà m.x/dxchết nguyên nhân khác Do đó, vi phân người có nguy dS D S qdx S m.x/dx, dẫn đến phương trình vi phân

dS

dx D qS m.x/S: (1)

Trong đódS vi phân hàmS.x/ Trong khoảng thời gian, số người chết bệnh

đậu mùa pS qdx số người sống sót sau mắc bệnh đậu mùa p/S qdx Số

sau cộng thêm vào lượng người chết nguyên nhân khácRm.x/dx Vì

dR

dx Dq.1 p/S m.x/R (2)

Cộng phương trình (1) (2), ta có

dP

dx D pqS m.x/P (3)

Đến lúc này, Bernoulli suy tỷ lệ cá nhân có nguy [nhiễm bệnh] cộng đồng dân cư ứng với độ tuổi địnhx cách sử dụng phương pháp biết giải

tích phương trình vi phân

S.x/ P x/ D

1

(10)

5 Số liệu tốn

Sau tách biệt nhóm R.x/ cá nhân có nguy nhiễm bệnh cộng đồng dân cư,

Bernoulli có tất yếu tố để tính biến thiên tuổi thọ trung bình Tuy nhiên, mơ hình phải “điền” đầy đủ liệu khoảng từxđếnxC1cùng với tỷ lệ mắc bệnh tử

vong tương ứng bệnh đậu mùa pvà q Những liệu khơng dễ dàng có được: Trong

phần lớn quan đăng ký rửa tội tang lễ thời khơng có tuổi tử vong đưa ra, khiến cho người ta xác định tỷ lệ tử vong ứng với tuổi x:Mặc dù vướng phải

những trở ngại này, năm1660;sự phát triển công ty bảo hiểm nhân

thọ góp phần thúc đẩy kỹ thuật tính tốn tỷ lệ tử vong Giải pháp áp dụng bao gồm sử dụng liệu cộng đồng dân cư ổn định, số ca sinh tử năm nhau, để xây dựng đường cong dân số Năm1693;Halley cơng bố phân tích dân số

thành phố Breslau, Đế chế Hapsburg [6], đáp ứng yêu cầu Kết nghiên cứu, gọi Bảng Nhân Thế Halley, bảng cho biết số1:300cá nhân sinh năm0;

có người sống tuổix

Căn quan sát ngày, Bernoulli sau chọn p D 18 vàq D 18, phép chọn

hóa xác Sử dụng Bảng Halley mơ hình dân số ơng ta, người ta tính đại lượngS.x/các cá nhân có nguy nhiễm bệnh tuổixvà sốR.x/DP x/ S.x/

những người độ tuổi x mắc bệnh đậu mùa sống sót Số ca tử vong bệnh đậu mùa

giữa tuổi x tuổi x C phải pq

xRC1 x

S.x/dx, công thức Bernoulli sử dụng pqŒS.x/CS.xC1/=2, xấp xỉ tốt

6 Phép so sánh tuổi thọ trung bình

Tại đây, Bernoulli xem xét tình tiêm vắc-xin vào cá nhân sinh (đề xuất ông Mércure de France khởi đầu việc tiêm vắc-xin bắt buộc trại trẻ mồ côi), mà không gây trường hợp tử vong Do đó, điều quan tâm tính tốn gia tăng tuổi thọ trung bình bệnh đậu mùa loại bỏ Bắt đầu với số lần sinhP0, gọi

P.x/là số người tuổixkhi bệnh đậu mùa biến mất: dPdx D m.x/P:Như

P D P x/

(11)

trong đóP x/là phận dân cư tuổix trước loại trừ bệnh đậu mùa So sánhP x/

vàP.x/có nghĩa ước lượng tuổi thọ trung bình từ lúc sinh ra, nghĩa tích phân

P0

1

Z

0

P x/dx:

[với bệnh đậu mùa, khơng có bệnh đậu mùa thay bằngP.x/] Bernoulli sử dụng công

thức gần

1

2P 0/CP 1/CP 2/C

P0

:

Kết thu

Tuổi thọ với bệnh đậu mùa là2

E D

1

2 1300C00C C20

1300 Š26:57 năm/

Tuổi thọ khơng có bệnh đậu mùa

E D

1

2 1300C1015C C23

1300 Š29:65 năm/

Do đó, tiêm vắc-xin chào đời đảm bảo tăng hơn3năm tuổi thọ

Tuy nhiên, tiêm vắc-xin phòng bệnh đậu mùa khơng phải quy trình an toàn, Bernoulli xem xétpl;xác suất tử vong vắc-xin, vớipl < ptuổi thọ trung bình pl/Enếu tất tiêm chủng sinh

Để việc tiêm chủng hợp lý, tác giả nhận thấy bất đẳng thức sau phải thỏa mãn

pl < E E:

nghĩa là, pl D 11%:Ngay khơng có liệu xác, Bernoulli ước tính tỷ lệ tử

vong sau tiêm chủng dưới1%:Do đó, việc tiêm chủng phải phủ thúc đẩy tích

cực: “Tơi đơn giản hy vọng rằng, câu hỏi liên quan chặt chẽ đến khỏe mạnh của lồi người, khơng có định đưa mà không xem xét tất thơng tin từ phân tích tính toán khiêm tốn” [1]

(12)

7 Những lời trích D’Alembert

Phục dựng cách mạch lạc lại quan điểm d’Alembert cách nói rõ thơng qua hàng loạt trích mơ hình Bernoulli công việc dễ dàng Khởi đầu từ Kỷ yếu thứ mười mộtvà mở rộng tiếp theo, trích có phạm vi rộng, từ hội trị đến đề xuất sử dụng liệu khác Hơn nữa, phần cuối kỷ yếu d’Alembert tun bố ủng hộ việc tiêm phịng, rõ ràng trái ngược với giọng điệu mà ơng trì toàn văn

Sự nhầm lẫn xuất phát từ điều coi cạnh tranh cá nhân Bernoulli, với mong muốn khơng thể cơng khai chống lại quan điểm (có lợi cho vắc-xin) ủng hộ giới khoa học văn hóa

Với kịch phức tạp tranh luận, hữu ích tập trung vào hai yếu tố có liên quan mặt khái niệm lời trích D’Alembert mà khơng thể bắt nguồn từ cách giải thích yếu tố “cơ hội” cho quan điểm ông

8 Quan điểm cá nhân so với xã hội: Phê phán tiêu

chí tuổi thọ trung bình

Điểm khác biệt hai tác giả liên quan đến hướng mà từ tính hợp lý vắc-xin đánh giá Như trình bày phần trước, Bernoulli đặt câu hỏi liệu có hợp lý khơng phủ thúc đẩy tiêm chủng hệ quả, tối đa hóa tuổi thọ trung bình, điều giúp cho nhà nước có số lượng lớn cá thể làm việc làm việc hữu ích Quan điểm rõ ràng hỗ trợ quan sát Bernoulli, mát mà xã hội phải gánh chịu chết người trẻ, người nuôi dưỡng lớn lên mà chưa đóng góp cho thịnh vượng chung Nói cách khác, Bernoulli đặt câu hỏi chủ đề sức khỏe cộng đồng, tập trung vào hậu xã hội sách có lợi cho vắc-xin

(13)

chấp nhận nguy tử vong trong2tháng tiêm chủng Thứ đến, phân tích Bernoulli đề xuất

là khơng đủ để tính đến “kinh nghiệm đạo đức” mà người ta có chấp nhận rủi ro Bằng thí nghiệm tinh thần xổ số giả định mà định xác suất

2 cho chết

cùng xác suất

2cho sống khỏe mạnh với kỳ vọng100tuổi, d’Alembert cho thấy tâm lý

rủi ro mơ tả thuật ngữ hồn tồn định lượng D’Alembert cho tuổi thọ trung bình số tối đa hóa cho phủ, khơng dành cho cá nhân: Vì lý này, phủ khơng có quyền áp dụng tiêm chủng cho cá nhân, chí với người chọn tiêu chí tối đa hóa khác

Ngồi số quan sát chung tâm lý rủi ro, d’Alembert không xây dựng giải pháp thay cho mơ hình Bernoulli, ông không đề xuất số phù hợp cho tối đa hóa cá nhân Theo ơng, cần so sánh nguy tử vong từ hôm với vài tuần sau tiêm vắc-xin với nguy tử vong tự nhiên bệnh đậu mùa khoảng thời gian Tuy nhiên, khả thức hóa kinh nghiệm đạo đức này, tác giả hồi nghi: Làm so sánh rủi ro với lợi ích khơng xác định từ xa? Liên quan đến vấn đề này, việc phân tích trị chơi may rủi khơng nói cho điều ([3], trang33 34)

Sự khơng tin tưởng vào tính thỏa đáng tính tốn xác suất áp dụng vào vấn đề vắc-xin bệnh đậu mùa điểm bất đồng thứ hai d’Alembert với Bernoulli Trong giai đoạn này, trích d’Alembert, cách chung định hướng chống lại nỗ lực áp dụng phương pháp xác suất cho chủ đề liên quan thực tế Giả thuyết địi hỏi phân tích rộng quan điểm D’Alembert với tư cách nhà lý thuyết xác suất

9 Nghệ thuật đoán

Tập IV Mélanges de littérature, d’histoire et de philosophie năm 1759 có

d’Alembert Essai sur les éléments de philosophie Sau trình bày phác thảo tác phẩm, tác giả đặt để phân định kiến trúc kiến thức người, cho thấy nguyên tắc làm tảng cho lĩnh vực kiến thức khác với phụ thuộc tương quan tương đối chúng, d’Alembert nói logic chương V, trích tun bố nhà triết học cho sử dụng logic kỹ thuật với quy tắc chung, mà tơn trọng chúng đảm bảo tính đắn lý luận:

(14)

đàn ông trung thực Các nhà hình học, vốn khơng bị mệt mỏi với giới luật logic, chỉ dùng suy nghĩ tự nhiên làm kim nam, tìm đường đến chân lý xa trừu tượng nhất, nhiều nhà triết học, hay hơn, người viết về triết học, đặt vào từ ban đầu tác phẩm họ chuyên luận lớn về nghệ thuật lý luận, sau đánh vài phương pháp, tương tự người chơi không may mắn, tính tốn để cuối thua.3

D’Alembert đặt nghệ thuật đoán lĩnh vực logic, giải thích lựa chọn cách lưu ý biết cách đoán tốt phần thiếu lực lý luận

Người nhận thật ảnh hưởng trực tiếp xa người không nhận biết khi nó gần mà dự đốn nhận biết từ xa, với tính cách bí ẩn nó.4

Tập V Mélanges, có Eclairsissements sur les éléments de philosophie, d’Alembert mở rộng phân biệt rõ quan sát mà ông thực Essai Chương VI dành riêng cho nghệ thuật đoán phân biệt ba lĩnh vực mà áp dụng Thứ để “phân tích xác suất trị chơi may rủi tn theo quy tắc đã biết tất định, nhà tốn học nhìn thấy vậy”.5 Lĩnh vực thứ hai

được xác định phần mở rộng phân tích cho câu hỏi liên quan đến sống hàng ngày, chẳng hạn thời gian sống, lương hưu, bảo hiểm hàng hải, tiêm chủng chủ đề tương tự khác

Theo d’Alembert, câu hỏi khác với vấn đề liên quan đến trị chơi may rủi vì:

trong [các vấn đề trò chơi may rủi], quy tắc kết hợp toán học đủ để xác định số lượng tỷ lệ trường hợp có thể, [vấn đề sống hàng ngày], ngược lại, có kinh nghiệm quan sát hướng dẫn số lượng trường hợp, và

3Nous avons sur la logique des écrits sans nombre; mais la science du raisonnement a-t-elle besoin de tant

de règles? Pour y réussir, il est aussi peu nécessaire d’avoir lu tous ces écrits, qu’il l’est d’avoir lu nos grands traités de morale pour être honnête homme Les géomètres, sans s’épuiser en préceptes sur la logique, et n’ayant que le sens naturel pour guide, parviennent par une marche toujours sure aux vérités les plus détournées et les plus abstraites, tandis que tant de philosophes, ou plutôt d’écrivains en philosophie, paraissent n’avoir mis la tête de leurs ouvrages de grands traités sur l’art du raisonnement, que pour s’égarer ensuite avec plus de méthode, semblables ces joueurs malheureux qui calculent longtemps et finissent per perdre ([5], I, p.152)

44 L’esprit qui ne reconnait le vrai que lorsqu’il en est directement frappé, es bien au-dessous de celui qui sait

non-seulement le reconnaitre de près, mais encore le pressentir et le remarquer dans le lointain des caractères fugitifs ([5], I, p.154)

5 l’analyse des probabilités dans les jeux de hasard soumise des règles connues et certaines, ou du moins

(15)

chỉ hướng dẫn cách gần đúng.6

Tuy nhiên, d’Alembert tuyên bố nhánh ứng dụng thứ hai nghệ thuật đoán này, việc truy địi tính tốn xảy “sự khơng chắn, có, liện quan đến các sự kiện đóng vai trị ngun tắc, kiện cho hệ nằm ngồi với”7

Loại nghệ thuật đốn thứ ba liên quan đến mơn khơng chứng minh Ở nghệ thuật đốn cần thiết Những mơn khoa học chia thành quan sát và thực tế Khoa học quan sát bao gồm vật lý lịch sử, khoa học thực tế bao gồm y học, luật học “khoa học giới”, nghĩa khả người sống xã hội, rút từ mối quan hệ với người khác lợi ích tối đa cho họ

10 D’Alembert “lý thuyết thông thường xác suất”

Trong bối cảnh phân tích cấu trúc kiến thức người, d’Alembert chấp nhận khả sử dụng phép tính xác suất đấu trường nghệ thuật đốn cần thiết, hàng loạt tiểu luận xác suất cho thấy điều trái ngược với hoài nghi ông Điều ấn tượng là, thực tế, hai lĩnh vực nghệ thuật đốn địi hỏi tính tốn xác suất lại dẫn đến cách chấp nhận đến thứ ba

Chúng ta thấy trường hợp tiêm chủng, trích dẫn rõ ràng trongEclairsissementnhư ví dụ chủ đề đưa vào lĩnh vực thứ hai nghệ thuật đốn, d’Alembert cho khơng có khả xử lý phân tích tốn học

Thay vào đó, liên quan đến trường hợp phân tích trò chơi may rủi, thật thú vị lưu ý d’Alembert dường đặt nghi ngờ khả áp dụng toán học cho loại câu hỏi D’Alembert dành nhiều tiểu luận khác để trích tảng ông định nghĩa “lý thuyết thông thường xác suất”.8

6 que dans celles-ci, les règles des combinaisons mathématiques suffisent pour déterminer le nombre et le

rapport des cas possibles, au lieu que dans celles-là, l’expérience et l’observation seules peuvent nous instruire de nombre de ces cas, et ne nous en instruisent qu’à peu prés ([5], p.157)

7 l’incertitude, s’il y en a, ne tombe que sur les faits qui servent de principes, ces faits supposés, les

con-séquences sont hors d’atteinte ([5], p.157)

8Bổng sung thêm vào báo Encyclopédie thảo luận chủ đề (Avantage, Croix ou pile, Gageur,

Dé, Loterie, Pari), nhắc lại tiểu luận có tập II (Hồi ký10và11) tập IV (hồi ký23)

(16)

Các lập luận đưa d’Alembert chia thành hai loại, toán học thực nghiệm.9Với lập luận toán học, d’Alembert đưa nghi ngờ khía cạnh lý thuyết của

tính tốn xác suất, chịu trích, số khái niệm khác, có định nghĩa khái niệm kỳ vọng, cách tính xác suất biến cố sở xác định trường hợp đồng khả

Quan điểm mà d’Alembert dường sử dụng lời trích giống nhà triết học, người vạch trần ngụy biện nhà giả hình học, lý thuyết xác suất, chủ đề nghiên cứu toán học, câu hỏi coi logic

Ngồi ra, phân loại Éléments cơng trình liên quanEclairsissementcho thấy vị trí trung gian cho lý thuyết này, nằm đầu tốn học logic Theo nghĩa này, lập luận thú vị phê bình việc ơng buộc tội nhà lý thuyết xác suất việc sử dụng phép ngụy logic để chứng minh kết họ Trong kỷ yếu thứ mười Opuscules, d’Alembert coi lập luận ngụy biện

1 Xác suất mặt sấp lần tung xác suất mặt ngửa lần tung

2 Xác suất mặt ngửa lần tung gấp đôi xác suất mặt ngửa lần tung đầu sấp lần tung thứ hai, hay ngửa lần đầu ngửa lần thứ hai Như xác suất mặt sấp lần tung gấp đôi xác suất mặt ngửa

trong lần tung đầu sấp lần tung thứ hai, hay ngửa lần đầu ngửa lần thứ hai

Và trích d’Alembert lập luận này:

Trong hai mệnh đềŒ12;thuật ngữ trung bình khơng giống nhau, mệnh đề thứ nhất,

thuật ngữ trung bình có nghĩa xác suất để mặt ngửa sau lần tung trước khi

9Sự khác biệt này, không phát biểu cách rõ ràng d’Alembert, toát lên tiểu luận

(17)

thực mệnh đề thứ hai xác suất mặt ngửa lần tung trong sự so sánh với xác suất nhận mặt ngửa mặt sấp lần tung thứ hai Bây giờ, xác suất cuối (tức nhận mặt ngửa mặt sấp lần tung thứ hai) giả định rằng lần tung thực mặt ngửa, đó, xác suất cuối này giả định điều đầu (tức nhận mặt ngửa lần tung đầu tiên) khơng cịn một xác suất, mà chắn.10

Các thuật ngữ logic sử dụng d’Alembert đoạn văn cho thấy rõ ràng giọng điệu phê phán ông Rất thú vị lưu ý phản đối đưa nhà bách khoa toàn thư vào thời điểm này, khơng hồn tồn rõ ràng, dẫn đến khái niệm xác suất có điều kiện, vốn xuất vào cuối năm1700;với cơng trình Bayes

Các lập luận thực nghiệm mà d’Alembert sử dụng trích cho khả áp dụng tính tốn xác suất vào tình cụ thể trị chơi may rủi truy ngun từ định nghĩa phạm trù khả siêu hình khả vật lý:

Cần phân biệt khả siêu hình khả vật lý Lớp bao gồm tất thứ mà tồn khơng có vơ lý, lớp thứ hai, khơng có điều kiện tồn khơng có vơ lý, mà cịn phải khơng có q phi thường và nằm ngồi tiến trình bình thường kiện Một cách siêu hình ta tung hai xúc sắc hai mặt sáu trăm lần liên tiếp, mặt vật lý điều không thể, nó chưa xảy khơng xảy ra.11

Sự khác biệt khả siêu hình khả vật lý có hậu nghiêm trọng từ quan điểm phân tích xác suất tượng thực Một ví dụ quan trọng phân tích lần tung đồng xu Theo d’Alembert, để xác lập khả vật lý cần phải thực thí nghiệm, qua xác định xác suất xảy kết định Liên quan đến vấn đề này, ông lưu ý

10 je dirai que dans cet argument le moyen terme d’est pas le même dans les deux Propositions Car le moyen

terme dans la premiere Proposition, est la probabilité d’amener pile au premier coup, avant d’avoir joué ce premier coup Dans la seconde Proposition, le moyen terme est la probabilité d’amener pile au premier coup, comparée la probabilité d’amener croix ou pile au second coup Or cette derniere probabilité (celle d’amener croix ou pile au second coup) suppose que le premier coup est joué, & qu’il a donné pile, ainsi cette dernier probabilité suppose que la premiere probabilité (celle d’amener pile au premier coup) n’est plus une probabilité mais une certitude ([3], pp.20 21)

11C’est qu’il faut distinguer entre ce qui est métaphysiquement possible, & ce qui est possible physiquement.

(18)

rằng xuất kết định nhiều lần thường làm cho xuất xảy tương lai Điều này, đến lượt dẫn đến dãy lần tung đồng xu khơng cịn đồng khả năng: Một dãy lần tung gồm số lần mặt ngửa mặt sấp gần có khả vật lý cao dãy mà kết kết nhiều lần

11 Kết luận

Như thấy, số lập luận trích lý thuyết xác suất truyền thống việc vạch trần ngụy biện giả định, thực d’Alembert vay mượn ngôn ngữ nhà logic học mà ông phê phán gay gắt Chiến lược giải thích nội hội: Chúng ta thấy cách d’Alembert, phân loại Eléments de philosophie, dẫn dắt lập luận xác suất đến bối cảnh logic đoán, đó, đến hai nhánh logic mà ơng xác định Mặc dù Eclairsissements, ông bày tỏ ủng hộ tính ứng dụng tốn học vào nghệ thuật đốn, lời trích xác suất phân tích cho thấy không tin tưởng khả đưa luận điểm cách cụ thể

D’Alembert miễn cưỡng coi logic cơng cụ chuẩn bị để nói với người cách suy luận tất lĩnh vực kiến thức Việc tính tốn xác suất, xếp cách mơ hồ lĩnh vực logic toán học, phải chịu nghi ngờ giống logic Logic, coi nghệ thuật lý trí, phải giới hạn thân việc cung cấp vài hướng dẫn sơ cách suy luận:

Điều nằm việc quan sát xác phụ thuộc lẫn [của thật], không dùng đến gia phả sai để lấp đầy khoảng trống nơi thiếu hồ sơ, cuối cùng, bắt chước những nhà địa lý, người tỉ mỉ đưa chi tiết khu vực biết, không sợ để lại những khoảng trống tương ứng với vùng đất chưa biết.12

D’Alembert dường ngụ ý nỗ lực áp dụng xác suất vào tự nhiên, giống công việc nhà địa lý học không chuyên nghiệp, không đủ cẩn trọng việc đánh dấu vùng đất chưa biết, nơi mà không chắn thống trị lý luận lấy chút

Lettera Mathematica2; 185 192 2015/;dịch từ tiếng Ý sang tiếng Anh Kim Williams

12Elle consiste observer exactement leur dépendance mutuelle, ne point remplir par une fausse généalogie

(19)

Tài liệu

[1] Bernoulli, D.: Essai d’une nouvelle analyse de la mortalité causée par la petite vérole et des avantages de l’inoculation pour le prévenir Histoire et Mémoires de l’Académie des Sciences,1760;parte2;pp.1 79 1760/:

[2] Daston, L.:Classical Probability in the Enlightenment Princeton University Press, Prince-ton, NJ.1988/:

[3] D’Alembert J.-B L R.Opuscules mathématiques, t II Chez David, Paris.1761/:

[4] D’Alembert J.-B L R.Mélanges de litterature, d’histoire, et de philosophie, t V Chatelain, Amsterdam.1767/:

[5] D’Alembert, J.-B.L.R.:Oeuvres de d’Alembert A Belin, Paris.1821/:

[6] Halley, E An Estimate of the Degrees of the Mortality of Mankind, drawn from the curious Tables of the Births and the Funerals at the City of Breslau, with an Attempt to ascertain the Price of Annuities on Lives.Philosophical Transactions of the Royal Society of London

17W596 610 1693/

[7] Voltaire, F M A de Letters on the English Vol XXXIV, Part2:The Harvard Classics

P.F Collier & Son, New York.1909–14/:Retrieved from:www.bartleby.com/34/2/,

(20)

S

Ự TÍCH

"T

RÂU

V

ÀNG

", "C

ÁO

C

HÍN

Đ

I

"

H

T

ÂY

(P

HẦN

2)

Nguyễn Lê Anh

GIỚI THIỆU

"Để hiểu lịch sử Việt Nam cần phải tìm hiểu tình hình địa chất theo thời gian."Vẫn ln góc nhìn lạ với chuỗi lập luận logic dựa nghiên cứu dày công, từ khảo cứu tài liệu đến thực địa địa phương, tác giả Nguyễn Lê Anh đưa hàng loạt kết nghiên cứu lịch sử, khơng kết đánh đổ nhận định vốn tồn hàng trăm năm

Ở Epsilon số 17 này, trân trọng giới thiệu tới độc giả phần loạt lịch sử với góc nhìn thú vị lạ Hồ Tây tích "trâu vàng" "cáo chín đi"

Thể tích hình chóp bằng

3 tích chiều cao nhân với diện tích đáy.

Hình sau chóp tứ diện vng có đáy tam giác cân Tam giác đáy tam giác cân có cạnh đáy dài150km chiều cao185km Chóp vng đỉnh có chiều cao là30m Thể tích

là139triệu mét khối Con số nói lên điều thế?

1 3×

185km×150km

(21)

Anh Nguyễn Chí Cơng có luận điểm kết dính lớn tạo cộng đồng, cụ thể cộng đồng Việt văn hóa tâm linh, hệ thống đình đền chùa miếu, nơi cư dân đến sinh hoạt Tâm linh diện Ma Quỷ, giới Âm Những thứ khơng có thật, mà hình thành khái niệm, na ná số âm tốn học Trong thực tế khơng có đại lượng tương ứng với kiện số âm, nhiên việc sử dụng số âm mang lại nhiều thuận tiện tính tốn Tâm Linh hồi tưởng cách có hệ thống khứ để tỏ lòng biết ơn tổ tiên

Sau đồ phần trung tâm Hà Nội Người Pháp vẽ vào năm1873.Trên đồ chúng

ta thấy cụm dân cư Hà Nội sống gò đồi xung quanh ruộng lúa nước Mỗi cụm làng Giao thông lại chủ yếu thuyền Ở gò đồi chấm đỏ, đình, đền, chùa làng Các đồng lúa xưa trung tâm Hà Nội, bị thị hóa, đầm hồ bị lấp gần hết Các làng cổ Hà Nội khơng cịn, mà cịn lại đình đền chùa Những di tích kể lại cho câu chuyện sống ông cha

(22)

Qua nghiên cứu đồ Google nhận thấy cư dân Việt định khắp đồng Bắc Bộ cư vùng gò đồi xung quanh nước, vùng gị đồi khơng lớn khơng cao Mỗi gị làng Họ lại chủ yếu thuyền Mỗi làng ln có nhà dựng lên nơi cao gị, sau Đình làng Con cháu người sinh sống gị thường làm nhà gần đấy, mà “Mọi con đường làng tới Đình!” Đình là nhà dựng gị, giếng trở thành Giếng Đình Thành giếng đắp cao để tránh nước lụt tràn vào Thông thường nơi đỉnh gị nơi có cao, xét chất thì, đỉnh gị cao lên nhờ bụi chắn gió bụi quẩn lại mà cao dần lên Những thường họ mà chim hay ăn mà chúng mang vãi hột khắp nơi gò cao chúng đậu Những đa vừa mồm cho chim ăn Và “Cây đa giếng nước sân đình” thứ in đậm vào tâm trí người Việt

Người dân sống gò cao trồng lúa ruộng Ruộng lúa bao quanh làng, có nước tự chảy Đầu làng nơi dòng nước chảy đến cuối làng nơi cuối gò đồi, nơi nước chảy Xét mặt tâm linh, người Việt không quên tổ tiên Họ quan niệm tổ tiên, tức người chết sống cùng, theo dõi phù hộ cho họ Vì người dân có ý thức chơn bảo vệ mồ mả Họ chôn người chết cuối làng, tức cuối nguồn nước Giữa khu vực người sống người chết có nhà mà sau trở thành chùa Chùa nơi giao tiếp người sống với người chết Người ta đến chùa để nhìn lại tổ tiên, để tỏ lịng thành kinh biết ơn Càng sau đất chật người đơng, phía sau chùa cịn dùng để chơn cất đời sư trụ trì, cịn mồ mả thay việc lưu giữ hộp tro, chí bia tưởng niệm lưu phía sau chùa Như phía sau chùa bãi tha ma, phía trước chùa đình làng; tồn chùa Việt cho thấy quan niệm song song tồn hai giới Âm Dương Cũng tiện lợi việc sử dụng việc đồng thời khái niệm số Âm Dương để nghiên cứu ứng dụng khoa học kỹ thuật, việc sử dụng song song hai giới tạo thuận tiện định việc xây dựng bảo tồn văn hóa Việt Ngoài liên quan đến cộng đồng đến tâm linh mà vị trí Chùa thường thay đổi theo thời gian Bởi mà xác định lại vị trí nhiều làng cổ, sơng cổ, hồ cổ, dựa vị trí chùa

(23)

là vị có sức mạnh lịch sử Vậy thật ẩn sau nội dung thờ cúng ấy? Muốn trước tiên phải biết thời điểm người dân tới dựng đình đền chùa vùng đất dựa việc phân tích mật độ di tích phân tích sự liên kết logic nội dung thờ cúng để chắt lọc nội dung lịch sử đựng

Nhiều báo khoa học cho thấy bờ biển lùi xa, ta nghĩ khứ diễn Cách khoảng6nghìn năm bờ biển vào vùng Phú Thọ, lùi

dần180km biển đến vị trí ngày Chúng cần phải xác định xem thời điểm cư dân có

thể đến định cư mô đất Chúng ta tin khống vào khẳng định “biển lùi, biển tiến” cách dễ dàng, để xác định thời điểm cư dân đến định cư vùng đất phải xác định tốc độ lùi biển

Hồn tồn khơng khó khăn để tìm liệu đáng tin cậy cho thấy mực nước biển ln giữ ngun6nghìn năm qua Người ta khoan sâu hàng nghìn mét để lấy mẫu băng

(24)

Từ việc biết tình hình khí hậu năm mà người ta xác định mực nước biển thời kỳ Nhìn vào độ thị mực nước biển theo thời gian nhận thấy có lúc cao mực nước biển tới400m Nếu gặp may tìm thấy vết sóng biển

để lại vách đá nơi độ cao Biển để lại dấu vết sinh vật biển bị mắc kẹt bị hóa đá tầng đất đá Những hang động độ cao vài trăm mét động Người Xưa khu vực Cúc Phương, nhiều hang động Quảng Bình, Hà Tĩnh, dấu vết bàn tay tạo hóa từ nhiều triệu năm trước

Loài người xuất châu Phi, khoảng 1triệu năm trước Trong khoảng thời gian 400

nghìn năm qua khí hậu trái đất thay đổi khơng nhiều, có tính tuần hồn theo chu kỳ khoảng130nghìn năm Mực nước biển mà lên xuống biên độ từ thấp

(25)

Trong vịng 140 nghìn năm qua mực nước biển có hai lần xuống thấp Lần thấp cuối

cùng cách đây20nghìn năm, mực nước biển thấp nay120m

Nước biển xuống thấp nhiệt độ toàn cầu xuống thấp, nước trái đất đóng băng hai cực nhiều tới mức mực nước biển bị thấp Trong khoảng thời gian 20 nghìn năm qua mực

(26)

Như mức tăng mực nước biển tương đối đều, 100m cho khoảng20nghìn năm, tức khoảng 5mm cho năm Tuy nhiên trong6000năm vừa qua mực nước biển gần giữ nguyên

Qua quan sát đồ Google nhận thấy đồng sông Hồng tam giác cân mà đình Phú Thọ, ước lượng chiều cao Phú Thọ khoảng 30m Vậy đồng

bằng sơng Hồng hình thành bờ biển ngày tiến phía biển? Trước nhiều năm tham gia vào dự án tính tốn trữ lượng nước ngầm cho Hà Nội anh Nguyễn Ngọc Kỷ, hiệu trưởng trường đại học Mỏ Địa Chất cho biết cấu trúc đồng sông Hồng vùng Hà Nội Theo anh Kỷ bên Hà Nội 100m tầng đá,

tiếp theo khoảng30m cuội sỏi nơi nước ngầm chảy mạnh Phía bên trên30m cuội sỏi

lớp có độ dày khoảng5m tầng sét không thấm nước, bên tầng đất phù sa

(27)

Sông Đà có lưu vực vùng Tây Bắc có lưu lượng gấp3 lần sơng Lơ, sơng Lơ có lưu vực

là vùng Đơng Bắc có lưu lượng gấp2lần sơng Thao Cả3con sơng nói tển hợp lại thành sơng

Hồng Cách dây 20nghìn năm, mực nước biển thấp nay120m thì, vịnh Hạ

long sâu trung bình 80m mà rừng rậm, sơng Hồng chảy tận phía ngồi đảo Hải

Nam Như khe vĩ đại hai dãy núi phù sa bồi lấp dày tới100m mà thành đồng

bằng Bắc Bộ, lẽ dĩ nhiên sâu100m khe đá

Quay trở lại câu chuyện với anh Kỷ, ngày anh em chúng tơi ngồi phịng tồi tàn tầng3trường đại học Bách Khoa Mọi người đói ấu trĩ Hầu tin

rằng đường khỏi đói việc tính tốn qua số Ngày tơi sĩ quan Học Viện Kỹ Thuật Quân Sự tiến sĩ tốn lý trường MGU, người có khả tính tốn tốt Mọi người đặt niềm tin vào tơi Câu chuyện khoa học chả có đáng để nói, cách đặt vấn đề “dùng tính tốn làm nhanh miếng ăn” chả thể mang lại điều có giá Cái đáng kể lại độ dày tầng phù sa100m, có chuyện đời

thường sản sinh nhiều đại gia trường ĐH Mỏ Địa Chất, mà đến họ chả biết

Ngày anh Kỷ bảo chúng tơi, “Anh ứng cử hiệu trưởng đây, nên làm gì?” Tơi nói với anh Kỷ, “Anh nên lo cho sống anh em” Anh hỏi “Cụ thể nên nào?” Chúng bảo anh xin mua đất cho anh em làm nhà làm ngơ cho họ muốn làm làm Thế anh xin đất Đầu tiên lập mẹo xin làm nhà tập thể5tầng mua đất xung quanh Tơi

cũng mua mua1xuất, nhà Rồi anh giáo

viên nước ngồi vơ tội vạ, miễn để lương lại cho anh em nhà lĩnh Thật may, hệ thống XHCN sụp đổ người dân Đông Âu Như Liên Xô rơi vào hoảng loạn Họ đói rét, họ bị lừa mua từ áo ấm “ma-dê-in” Việt Nam mà bên độn rễ bèo tây Rất nhiều đại gia giáo viên bỏ dạy hợp tác nghiên cứu đánh hàng phất lên ngày Chuyện kỷ niệm anh Kỷ Rất tiếc anh Kỷ ung thư thời gian sau

Quay lại với hình thành đồng sơng Hồng Tơi biết tồn phù sa dày khoảng 70m

là sông Hồng mang tới, rằng6000năm trước biển vào tới tận Phú Thọ Vậy

cớ mà mực nước biển khơng thay đổi mà bờ biển lại tiến tới mức độ vài chục mét năm? Tơi bắt đầu hình dung đồng Bắc Bộ dâng cao lên5mm năm,

nghiêng Phú Thọ nhô lên khỏi mặt nước biển trước vùng đất khác nhô sau Hỏi Google sông Hồng, tơi biết sơng Hồng hình thành dựa cú đâm địa Ấn vào Đại lục địa châu Á Tơi thích đỉnh Everest Tơi dõi theo kiện liên quan đén đỉnh núi ấy, từ thời cịn học phổ thơng, tơi nhớ rõ kiện “Dãy Hymalya đỉnh Everest được hình thành từ cú đâm lục địa Ấn, tận ngày đỉnh núi Everest vẫn được nâng cao5mm năm” Vậy cho đồng sông Hồng nâng lên

mỗi năm5mm Với khái niệm “Lục địa dâng cao 5mm năm” thế, gọi thời

(28)

Nhớ khoảng6nghìn năm trước bờ biển vào vùng Phú Thọ Hỏi anh Google

có thể thấy Phú Thọ đỉnh tam giác cân đồng Bắc Bộ, có đáy dài khoảng150km từ

vùng Hạ Long tới Phát Diệm, cạnh bên dài khoảng200km Cũng hỏi anh Google ta biết Phú

Thọ độ cao mực nước biển khoảng30m

Với mơ hình “Lục địa dâng cao5mm năm”, sau6000 năm, Phú Thọ độ cao 30m

hợp lý Vậy sau6000năm mực nước biển rút xa180km, tức cứ1000năm rút xa

khoảng30km

Áp dụng quan điểm Everest ta để nhận thức lịch sử thấy vùng đất có “đền thờ nhà Trần vùng thành phố Nam Định, nơi cách biển40km”, nhơ lên khỏi mặt nước biển từ khoảng 1200năm trước, tức khoảng vào kỷ thứ8kể từ đầu Cơng Ngun Để đến định cư

(29)

đến600 năm Vậy hợp lý, đền đình chùa nhà Trần Nam Định xây vào khoảng từ

thế kỷ12cho đến14

Tôi tính thời điểm đầu Cơng Ngun người dân dựng đền thờ hai Nữ tướng Bà Trưng Cống Mọc Vậy tơi mừng có phương pháp xác định thời điểm người dân đến định cư khu vực

Sự việc việc nhiều người khẳng định trước vịnh Hạ Long lại Kiến thức có nội dung học phổ thơng Tơi sẵn sàng tin điều ấy, nhiều đảo ngồi vịnh Hạ Long cịn có Vọc sinh sống

Nếu năm thềm lục địa sông Hồng nâng cao5mm có nghĩa lùi

khứ bị thấp đi5mm năm Lùi20nghìn năm trước thấp nay100m, cộng

với lượng phù sa ước tính là20m thấy đồng sông Hồng thấp nay120m Việc

(30)

nước biển nay, khó mà nói qua vịnh đảo Tôi rơi vào thất vọng với mơ hình Everest, thật thềm lục địa đồng sông Hồng nâng lên phải nâng lên nhiều trăm nghìn năm, Hà Nội phải độ cao đỉnh Everest Mà điều khơng đúng!

Tất nhiên tơi cố gắng tìm tài liệu khoa học, tìm thấy tác giả chúng Tơi có hỏi họ câu trả lời không thỏa mãn

Tôi suy nghĩ thấy nến cháy để lại đám sùi phía thân nến, mà nước nến phải chảy xuống tận chân nến Những đám sùi nến bị sức nóng lửa mà chảy nước bị sức hút trái đất mà chảy xuống thấp dọc theo thân nến Nước nến chảy xuống thân nến gặp lạnh đơng lại, thế, hết lớp đè lên lớp khác, mà khối sùi to dần lên Phù sa từ Phú Thọ, ngun lý phải chảy xi theo dịng nước tới nơi thấp nhất, dòng nước nến gặp lạnh đơng lại phù sa đọng lại vận tốc dòng nước phù sa chảy chậm Khi dòng phù sa chảy đủ chậm hạt phù sa kết dính với nhau, hết lớp đến lớp làm cho lịng dịng chảy phía Phú Thọ cao dần lên Lớp phù sa tạo độ nghiêng từ Phú Thọ tới tận ven biển, cho đồng sơng Hồng Như sức kết dính hạt phù sa nguyên nhân làm cho bị đọng lại vùng Phú Thọ trước mà không bị chảy xuống đáy biển sâu Chúng ta gọi mơ hình “nến chảy

Hỏi Google biết tổng lượng phù sa mà sông Hồng mang biển 100 triệu

năm khối lượng riêng đất khoảng1.4tấn/1m3.Vậy năm sông Hồng mang

ra lượng phù sa 75triệu mét khối đất Ước tính tổng thể tích hình khối tứ giác vng

chiều cao30m (ở Phú Thọ) đáy tam giác đồng sông Hồng Đáy tam giác cân

cạnh200km đáy150km bờ biển từ Ninh Bình Hạ Long

185km×150km

2 ×30 = 139000triệu m 3.

Mỗi năm sông Hồng mang biển100triệu phù sa, tức75triệu mét khối đất, thời

gian để có ngầy phù sa 139000

75 = 1800năm Như lượng phù sa khoảng thời

gian4000năm bồi vào đâu?

Hiển nhiên phải coi lượng phù sa bồi chỗ vùng đồng Bắc Bộ, 6000

(31)

sâu trung bình vịnh ước tính từ tổng lượng phù sa bồi lấp trong4000

năm Như vịnh Sông Hồng chừng sâu khoảng20m

1 ×

185km×150km

2 ×30m+

185km×150km

2 ×20m

×75triệu m1 3 = 5500năm

Vậy rõ, lượng phù sa bồi đắp vịnh Sơng Hồng, sơng Hồng mang

5500năm qua Vịnh đáy phẳng, dốc phía biển Vậy

ước tính Phú Thọ vịnh Sơng Hồng sâu khoảng10m ven biển sâu độ40m Nhìn vào biểu

đồ mực nước biển vịng20nghìn năm qua, khoảng10nghìn năm trước mực nước biển

thấp khoảng 40m Vậy là10nghìn năm trước Việt nam chưa bị

ngập mực nước biển

Tổng hợp lại20nghìn năm trước mực nước biển thấp nay120m vùng Hạ

Long rừng rậm Nơi có vọc sinh sống mà kể người tiền sử Mực nước biển dâng cao dần 120m suốt 14nghìn năm Nhiều lồi vọc lại đỉnh

núi cao vịnh Hạ Long, hiên người tiền sử di cư dần lên chỗ cao Những người tiền sử di cư phía bắc vịnh Hạ Long người Bách Việt, người tiền sử di cư phía đồng Bắc tổ tiên trực tiếp người Việt Nam ngày Khoảng10

nghìn năm trước mực nước biển thấp khoảng 40m Việt Nam xưa có hình

thù diện tích ngày Nó đáy vịnh Sông Hồng, bị vùi lấp độ sâu vài chục mét phía bên đồng Bắc Bộ ngày Nước biển tiếp tục dâng dần 4000năm, bờ biển vào tận Phú Thọ sóng biển vỗ bì bọp vào chân núi Ba

Vì Tam Đảo Mực nước biển giữ nguyên suốt 6000năm qua Đồng Bắc

Bộ ngày nay, vịnh nơng, có độ sâu trung bình khoảng20m Trong suốt khoảng

thời gian6000năm qua, phù sa sông Hồng bồi đắp vịnh thành đồng

Những người tiền sử lại vùng cao ven núi người Mường, người di cư vùng đất bồi người Kinh Theo thời gian đất bồi xa, người Kinh đơng dần lên gia sản văn hóa Mường mang theo nhạt dần nhường chỗ cho thể dạng văn hóa mở Và mực nước dâng chậm,100năm lấn sâu vào đất liền được2km việc di cư

rất chậm, người dân có xu sàn nhà cao dần lên để lại Như nhà sàn dân tộc Mường di sản lại 14000năm nước biển dâng, cấu trúc đình làng Việt thừa

(32)

Vậy thềm lục địa khu vực Đông Nam Á vậy, không nâng lên hay hạ xuống, đồng sông Hồng tạo phù sa đọng lại vận tốc dịng chảy giảm 20nghìn năm

trước mực nước biển thấp 120m, người Việt loại Vọc

dễ dàng di chuyển đảo, kể quần đảo Hoàng Sa Trường Sa Khoảng 10nghìn năm

trước đây, nước biển dâng lên, di chuyển vào khỏi khu vực đồng Bắc Bộ trở nên khó khăn hơn, đồng Bắc Bộ bị bao quanh dãy núi cao nhiều nghìn mét rộng hàng trăm kilomet

Điều tạo gianh giới vật lý tách biệt cư dân sống đồng sông Hồng với cư dân phía Nam Trung Quốc Mặc dù có giao lưu đường biển thơng qua vùng Hà Cối Quảng Ninh cư dân đồng sơng Hồng với cư dân phía Nam Trung Quốc có khác biệt văn hóa họ Vậy sử dụng mơ hình “Everest”5mm

một năm để tính tuổi vùng đất nâng lên khoảng6 nghìn năm vừa

qua mà

Dựa vào khái niệm “nến chảy” cố gắng tìm vị trí nơi mà vận tốc dịng nước có phù sa giảm gần tới bằng0.Đấy nơi phù sa đọng lại hình thành

(33)

phía chân núi Tam Đảo nơi để lại vết hồ Đại Lải, qua mũi Thánh Gióng (đền Sóc) tới chân núi Chí Linh men biển phía Quảng Ninh Ra biển bị dịng Hải Lưu đẩy ngược trở lại tạo thành xốy khu vực phía Hải Dương Phù sa bắt đầu đọng lại đáy biển nâng cao thành gò Hải Dương Gò xa ngồi biển gị với vùng Chí Linh có eo biển Theo thời gian phù sa bồi mà bờ biển phía Chí Linh nhơ dần ra, gò Hải Dương lớn dần lên sát vào phía bờ, mà eo biển biến thành sơng Đuống Các sông khác sông Bắc Ninh Cơ sơng Luộc, hình thành từ ngun lý sơng Đuống Nếu để ý sông Đuống theo dạng lực Coriolics, lực sức nước sông Đà mà là lực Coriolics đẩy dịng nước phù sa sông Hồng xa hàng trăm kilomet! Khi xưa nhánh chính, tức sơng Hồng ngày chưa có Thời gian 6000 năm trước cửa sơng Hồng Phú Thọ Theo thời gian cửa “Ba Lạt” chạy dần xa, khoảng45km cho1000năm tính theo chiều dài sông cửa Ba Lạt

Bằng cách đo khoảng tới cửa Ba Lạt, biết tuổi sông chia lưu sông Hồng, từ biết tuổi làng hai ven sông nưu biết tuổi đình đền chùa

Xét đồ cấu tạo thềm đá Cổ Loa phần “sống trâu” cuối dãy núi Tam Đảo, sau sườn núi dốc thẳng xuống vực Hà Nội khơng phải sống trâu dãy núi, sườn dốc núi Ở sâu mặt đất Hà Nội có cấu tạo tầng than bùn Do đồng Bắc Bộ khe núi, tức bị bao bọc vành núi Đơng Triều Hồng Liên Sơn cao hàng nghìn mét, nên thứ gió từ biển thổi vào bị xoáy quẩn lại Hà Nội Hà Nội cao lên thành gò nhờ bụi quẩn lại từ vùng đồng Bắc Bộ xoáy Hồ Tây thực tế khu vực bị lún cục từ đầu kỷ thứ10.Bất chấp lực Coriolics, sau chảy khỏi Hà Nội sơng

Hồng chảy vịng xuống phía Nam Đó chảy khe núi xưa hai dãy núi Khe sâu, phù sa chưa thể bồi lấp

Khi xưa biển vào tới tận chân dãy núi Ba Vì bờ biển sát Hịa Bình Giữa Ba Vì gị Hà Nội eo biển Vào mùa mưa Đông - Bắc, dịng nước sơng Lơ mạnh, đẩy dịng nước sơng Hồng phía eo biển Phù sa sơng hồng bồi dần, bờ biển phía Ba Vì tiến khiến cho eo biển hẹp lại mà thành sơng Đáy

Trong vịng nhiều nghìn năm dịng nước sơng Đà xói dần bờ phía bên Ba Vì thành hõm, nên hướng chảy bị lệch đi30độ

Dịng chảy sơng Đà đẩy dịng nước hướng dần phía Việt Trì bị hắt Miếu Mèn thuộc Sơn Tây Dòng chảy sơng Hồng khơng cịn đâm phía hồ Đại Lải nữa, phù sa bồi lấp dần chân núi Tam Đảo thành vùng đồng có di khảo cổ Đồng Đậu3500năm tuổi Sự

(34)

Dịng chảy sơng Hồng phía Miếu Mèn thuộc Sơn Tây thay đổi dần hướng chảy, hệ lụy cửa sơng Hát Mơn sơng Đáy xưa bị lùi sâu vào4km

(35)

Phía Bắc Hà nội vùng đất cao Xuân Đỉnh, nơi từ nhiều nghìn năm trước người tiền sử di cư từ phía Ba Vì tới Hà Nội xưa đầm lớn với gò đất:

1 Núi Sưa cao 16m vườn Bách Thảo Ở núi có nhiều sưa, đỉnh núi cịn

đền thờ Huyền Thiên Hắc Đế

2 Khu vực Hồng thành Thăng Long cịn có núi Nùng cịn gọi núi Long Đỗ, (có nghĩa rốn rồng) Ngày xưa, Lý Thái Tổ dựng điện núi Nùng Đến đời Lê, vào năm

1430,xây điện Kính Thiên cũ Núi khơng còn, còn4bệ rồng đá

là dấu vết điện Kính Thiên cũ

3 Núi Khán vị trí khoảng trước Phủ Chủ tịch bây giờ, núi đất thấp thời Lê thường dùng làm nơi vua ngự xem duyệt binh, lâu thành tên Núi bị san hồi cuối kỷ XIX

4 Nằm khu vực đường Thụy Khuê đường Hoàng Hoa Thám thuộc quận Ba Đình cịn có núi đất, gồm núi Cung cao 18m, tương truyền cung điện dựng Núi Cột Cờ cao13m, núi Voi (cịn gọi núi Thái Hịa) cao14m phía Đơng núi Cột Cờ

(36)

7 Núi Bò cao8m cạnh hồ Thủ Lệ

8 Gò Đống Đa

9 Vùng đồi cao nhà Bát Cổ

Găn liên với gò đồi dòng nước chảy quanh chúng Nó nhánh sông tạo câu truyện sau

Đi liền với Hồ Tây tích Trâu Vàng - Kim Ngưu: “Sư Không Lộ đánh chuông, Trâu Vàng kho tàng vua Tống nghe tiếng chng đồng, phóng chạy phương Nam Nhà sư e vàng bạc đất Trung Quốc theo Việt Nam, lăn chng xuống Hồ Tây” Sự tích cho thấy hồ Tây xuất có cơng nghệ đúc Đồng, tức không quá3000về trước

Đầm Xác Cáo tên gọi xưa hồ Tây, gắn với tích “Hồ Ly Tinh chín đi” Lịch sử đời Hồ Tây nhắc đến lần Lĩnh Nam chích quái Vũ Quỳnh Kiều Phú soạn vào khoảng năm1492.Ở đây, tác giả kể lai lịch Hồ Tây truyện Hồ

Tinh Theo phía tây thành Long Biên có hịn núi đá nhỏ, có Hồ Tinh (yêu quái cáo, qi sau Ngư Tinh, trước Mộc Tinh) chín sống ngàn năm, biến hóa vạn trạng, thành người thành quỷ khắp dân gian Long Quân lệnh cho sáu đạo quân thủy phủ dâng nước lên công phá bắt cáo mà nuốt ăn Nơi trở thành vũng sâu Hồ Tây hang cáo chín phá hoại dân lành, bị Long Quân dâng nước lên công phá Do đó, hồ có tên đầm Xác Cáo Để giữ kỷ niệm xưa, người ta đặt tên cho cánh đồng phía Tây đầm Hồ Đỗng (hang cáo) thơn xóm cạnh cánh đồng Làng Cáo (Xuân Tảo), làng Hồ Thôn (nay Hồ Khẩu) “Hồ” “con cáo” đồng âm với “hồ” (hồ nước), hòa quyện với địa danh Hồ Khẩu, Cáo Đỉnh, Đối với dân thường, ý nghĩa huyền thoại đời sống hiận thực thật khó lịng tách bạch

Nếu điều mà (và khả lớn) câu chuyện truyền thuyết thật nào? Câu chuyện ghi lại thành Truyện (thay Hồ Cáo) hay vậy? Trên thực tế âm 湖(cái Hồ) âm狐 (con Cáo) gần giống nhau, có lẽ câu truyện

con “Cáo Chín Đi - Khẩu (cửa sơng)” câu chuyện Hồ có nhánh chảy sông chảy Vậy ký mã tên gọi “Hồ Chín cửa Hậu” mà bị sai lệch thành “Cáo Chín Đi” Các em học sinh thân mến, từ việc biết tính thể tích hình chóp hiểu khung cứng tồn lịch sử dân tộc từ cách đây20nghìn năm Chúng ta có

(37)

D

ẠY ROBOT ĐÁNH CỜ BẰNG HỘP DIÊM

Võ Bích Khuê, Nguyễn Hùng Sơn

Tháng 4, 2020

TÓM TẮT

Học máy (machine learning) ngành thú vị tin học Mục đích học máy tìm thuật tốn cho máy tính để giải vấn đề phức tạp cách tự học Chẳng hạn, máy tính sử dụng phương pháp thử nghiệm học cách chơi trị chơi giành chiến thắng cách học hỏi từ sai lầm

1 Giới thiệu trị chơi

Trong báo này, chơi trò chơi hexapawn

Hexapawn trò chơi dành cho hai người, Martin Gardner[1] phát minh vào năm 1962 Trò chơi tổng quát tiến hành bảng hình chữ nhật có kích thướcm×n,

người chơi bắt đầu hàng gồmn tốt, đứng ô vuông hàng

gần với người

Chúng ta minh họa dùng bàn cờ3×3cho đơn giản Trường hợp có ô vuông, người

chơi, người chơi có tốt

Hình 1: Trị chơi Hexapawn bàn cờ3×3và4×4

(38)

Có cách để giành chiến thắng:

1 Con tốt tiến đến dịng cuối phía bên bàn cờ Ăn hết tốt đối phương

3 Đối phương khơng cịn nước (như chiếu hết cờ vua)

2 Phương pháp đào tạo robot chơi Hexapawn

Từ lúc gọi hai đối thủ "người chơi", đối thủ thứ hai "robot" "máy tính" Hexapawn trị chơi mà robot luyện tập (học) để thắng người chơi Chúng ta minh họa phương pháp mà máy tính sử dụng để chơi trị chơi Hexapawn bàn cờ3×3 Người chơi trước (theo tác giả trò chơi) nên dùng quân trắng nước

lẻ1,3,5,7 Máy tính nước thứ2,4,6,8vì ta chứng minh trị chơi

sẽ kết thúc sau nước

Bạn đọc hiểu nguyên lý trò chơi, làm quen với phương pháp huấn luyện máy tính robot chơi cờ Hexapawn

2.1 Chuẩn bị

Đầu tiên nhận thấy bước thứ người chơi phải tiến tốt lên hàng thứ Như vậy, nước thứ 2, robot nhận cấu hình bàn cờ Hình (hai cấu hình có số dưới) Tương tự tiếp tục xét ta có 11 cấu hình nước thứ 11 cấu hình nước thứ Hình mơ tả tất 24 cấu hình mà robot gặp chơi Hexapawn

(39)

Hình 3: Một hộp diêm có cấu hình nước robot chơi trò chơi Hexapawn Trước hết chuẩn bị 24 hôp nhỏ tương ứng với cấu hình có bàn cờ Bạn đọc in cấu hình Hình dán lên 24 hộp diêm

Sau chuẩn bị xong hộp diêm, bạn đọc cần phải đặt viên bi tương ứng với mũi tên vẽ hình chữ nhật (một mũi tên màu xanh viên bi màu xanh hộp, mũi tên đỏ viên bi đỏ, v.v.) Sau bạn chuẩn bị xong, sẵn sàng huấn luyện cho robot chơi cờ

Hình 4: Các hộp diêm để luyện cho robot chơi trò Hexapawn

2.2 Thuật toán học chơi cờ Hexapawn

(40)

Hình 5: Robot dùng hộp diêm để chọn nước trò chơi Hexapawn

ứng với nước sai lầm Bằng cách này, robot khơng thể lặp lại nước sai lầm tương tự trận tình xảy Bằng cách loại bỏ viên bi xấu này, robot học cách chơi cờ ngày tốt có cảm giác ngày thông minh Nếu chơi với robot nhiều trận, nhận thấy chơi nhiều, thuật toán robot hoàn thiện thắng nhiều cuối (người chơi) giành chiến thắng

Nhưng có nhiều nhà tốn học, khoa học máy tính, nhà khoa học liệu, lĩnh vực khác có ý kiến thuật tốn chưa đủ tốt Tất nhiên họ có lý Hy vọng bạn đọc có ý tưởng để làm tốt

Ví dụ, cách học chơi trên, phạt robot bị thua? Việc phạt robot bị thua làm giảm khả robot chọn nước xấu hoàn thiện cách chơi Vấn đề với thuật toán này, robot phải chơi nhiều ván để trở nên thơng minh Nhưng điều xảy thưởng cho bước đúng?

Giả sử sau bước dẫn đến thắng lợi, việc trả lại bi vào hộp, thưởng cho robot cách cho thêm vào hộp diêm viên bi màu với nước Bằng cách này, làm giảm khả thua tăng xác suất để robot chọn viên bi (bước đi) Và với thuật toán này, robot cuối đạt khả chơi hoàn hảo sau số ván cờ giảm tác dụng loại bỏ viên bi tương ứng với bước sai

Đây khơng phải thuật tốn học đánh cờ Chắc hẳn có nhiều thuật tốn khác Nhưng chắn nói q trình học đánh cờ thực sự, máy tính chơi ngày hoàn hảo

(41)

3 Kết luận

Bài báo mang tính chất minh họa phương pháp học chơi cờ tự động với mục đích giúp học sinh thầy cô tiếp cận với môn học máy ngành trí tuệ nhân tạo

Chúng ta thấy 24 hộp diêm tương ứng với 24 cấu hình có bàn cờ Tại cấu hình có từ đến nước Vì tình có đầy đủ thơng tin trị chơi

Với trị chơi khác, số cấu hình lớn nhiều Ví dụ số cấu hình cờ đam (checkers) bàn cờ8×8là khoảng2,3×1021, cịn cờ vua (theo Claude Shannon) thì

có nhất10120cấu hình, có khoảng1040cấu hình có nghĩa Phức tạp cờ vây vì

có khoảng10172 cấu hình Trong trị chơi này, việc ghi nhớ tất cấu hình khơng thể.

Thách thức với chun gia trí tuệ nhân tạo phải (1) nghiên cứu, tìm tịi thuật toán học chơi cờ phức tạp để giải tốn trường hợp khơng thể nhớ tất cấu hình (2) phải trả lời câu hỏi học máy tính đạt trình độ đánh bại đại kiện tướng giới

Các thuật toán tiên tiến mở rộng cải tiến thuật tốn mơ tả báo Trong báo giới thiệu đến bạn đọc phương pháp học đánh cờ đại

Tài liệu

[1] Martin Gardner Mathematical Games, Scientific American, March 1962, reprinted in The Unexpected Hanging and Other Mathematical Diversions, pp 93ff

(42)

M

Ã

V

À

C

ÁC

K

T

HI

O

LYMPIC

T

OÁN

(P

HẦN

C

UỐI

)

S.B.Gashkov

GIỚI

THIỆU

Tiếp theo kỳ trước, xem phần mở đầu phần 1,2,3 Epsilon số 15 4,5 Epsilon số16

6 Mã Reed - Solomon

Trong cánh đồng Galois tràn đầy bơng hoa, các ngun thủy nhảy nhót tiếng đồng hồ32)

S.B Veinstein (IEEE Transactions of Information Theory,1971)

Cho số nguyên tố q > Nhắc lại mã tuyến tínhq−phânC độ dàinchiềuk (viết tắt mã

[n, k]) không gian conkchiềuCbất kỳ không gianGF(q)ntất véc-tơnchiều

trên trườngGF(q) Dễ thấy khoảng cách mã mã tuyến tính trọng lượng nhỏ mà

một véc-tơ khác0có thể có Nếu mã[n, k]có khoảng cách mãd, gọi mã[n, k, d]

Phương án đơn giản để xây dựng mã Reed – Solomon33) trườngGF(q)(viết tắt mã

RS) sau Chok < nq Ta cho tương ứng véc-tơa= (a0, , ak−1)∈ GF(q)k đa

thức

a(x) = a0+a1x+· · ·+ak−1xk−1,

bậc k − trường GF(q) Giả sử x1, x2, , xn ∈ GF(q) phần tử phân biệt

trường Ta xét ánh xạ tuyến tínhl :GF(q)k→GF(q)n,xác định đẳng thức l(a) = (a(x1), , a(xn))∈GF(q)n

Ảnhl(GF(q)k)của ánh xạ – mã tuyến tínhC, gọi mãRS Từ bất đẳng thứcn > k

đa thứca(x)bậck−1có thể xác lập cách theo giá trị tạinđiểm,

vì ánh xạ l : GF(q)k → C tương ứng1−1.Nghĩa lực lượng mãC bằngqk, do

đó chiều bằngk Khoảng cách mãd(C)n−k + 1, với đa thức khác không

a(x), véc-tơ giá trị nó(a(x1), , a(xn)) ∈ C ⊂ GF(q)n có trọng lượng khơng nhỏ

(43)

Thật d(C) = n − k + 1, theo định lý cận Singleton, với mã [n, k] ta có

bất đẳng thức d(C)n−k + 1.Các mã nằm cận gọi mã với khoảng cách lớn

nhất (maximum-distance seperable - MDS) Các mãRS có tính chất Các mã nhị phân

không đạt cận

Định lý tương ứng phát biểu sau (định nghĩamq(n, d)xem phần2)

Định lý (Cận Singleton34, hay cận phép chiếu).

mq(n, d)qn−d+1

Lời giải Chỉ cần chiếu tất cảqktừ mã lên không gian conGF(q)k−1

.Để làm điều ta điền

0vào vàon−k + 1tọa độ cuối véc-tơ mã, giữ nguyênk−1tọa độ đầu Theo nguyên lý

Dirichlet hai véc-tơ mã có ảnh chiếu, tức làk−1tọa độ đầu chúng trùng

nhau Do khoảng cách chúng không vượt quán−k+ 1.Tức làdn−k+ 1(hơn

điều với mã không tuyến tính có lực lượng lớn hơnqk−1).

Bây ta dễ dàng giải tốn28và29

Bài toán 28 Trên bàn cờk−phânn chiều đặtkm+ 1con xe Chứng minh tìm hai

con xe mà tọa độ chúng khác khơng qn−mvị trí Nói riêng, nếum=n−1thì

tồn hai xe ăn nhau.

Bài toán 29 Nếu k là lũy thừa số nguyên tố và n > m+ 1thì tốn 28, có thể

đặtkm+1con xe cho tọa độ hai xe khác khơng qn−mvị trí Nói

riêng, nếu m = n−2thì bàn cờk−phânn chiều đặtkn−1 sao cho chúng đôi một

không ăn Vớin= 2điều hiển nhiên.

Với toán30ta đưa câu trả lời lời giải

Bài toán 30 Bạn vào phịng, có bàn cờ, có số quân cờ Người

ta cho bạn biết tọa độ ô giao nhiệm vụ cho bạn chuyển thông tin ô cho người bạn bạn vừa vào phòng sau bạn Ăn quân cờ hay đặt quân cờ vào ô trống gọi là nước Sau vào phịng, bạn khơng có điều kiện để gặp người bạn báo cho anh ấy điều Cho đến người bạn đến bàn cờ không thay đổi Tuy nhiên trước vào phòng, bạn trao đổi thỏa thuận chiến thuật chung Hỏi dùng bao nhiêu nước để giải toán đặt ra?

Trả lời:Chỉ cần1nước đi

Gợi ý: Tọa độ ô “mã hóa” nhị phân độ dài6.Một cách đặt quân

cờ bàn cờ xác định nhị phân độ dài64.Hãy sử dụng ma trận kiểm tra mã tuyến

tính [64,58], tức mã Hamming, mở rộng cách thêm tọa độ0.Đó ma trận

nhị phân(6,64)mà tất cột khác (có thể xóa cột0từ sử dụng ma trận(6,63)

-ma trận kiểm tra mã Hamming thông thường) Nếu nhân nhị phân độ dài64với ma trận

này, thay đổi vị trí, thu Bộ chọn cho “ hóa” ta cần báo

(44)

Bài toán 31 (Trang 12119). Một điệp viên đượng tung vào quốc gia lạ, liên lạc với

trung tâm cách sử dụng đài phát địa phương, hàng ngày phát thông điệp dài

255bit Điệp viên tiếp cận với tin trước phát lên sóng, anh

ta có quyền sửa nhiều bit Hỏi hàng ngày điệp viên chuyển bao nhiêu bit thông tin cho Trung tâm? (Tất nhiên điệp viên thỏa thuận với Trung tâm về phương pháp mã hóa giải mã trước đó.)

7 Định lý Zarankevic giải mã

Hamilton mở trang sách ngẫu nhiên hiểu rằng cuốn sách, có lẽ là, tuyển tập ý tưởng điên rồ Richard Tierney “Tiếng kêu đêm tối”

MãC với độ dài khốin gọi là(e, l)- giải kiểu danh sách với từy độ

dàintrong hình cầuB(y, e)bán kínhevới tâm tạiy(trong mê-tric Hamming) có khơng ql

từ mã, tức là|B(y, e)∩C|l.Sudan Gurusvami35([18]) chứng minh36được định lý sau

Định lý Mọi mã(n, k, d)đều(e, n(d−e))giải kiểu danh sách vớie < n−pn(n−d).

Nói riêng, mãRS(n, k)qđều đều(e, n(d−e))giải kiểu danh sách với

e < n−pn(k−1) Lời giải Hiển nhiên

n−d=√n−d·√n−d <pn(n−d), n−pn(n−d)d 2,

bởi vìn−d =

n+n−d

p

n(n−d).Theo bất đẳng thức trung bình cộng, trung bình nhân định lý

sẽ suy từ định lý Zarankevic, phát biểu dạng toán

Giả sử cj ∈ B(y, e)∩C với jm, là tất từ khóa nằm hình cầuB(y, e) trong đó

y= (y1, , yn)là từ độ dàintrong bảng chữ mã Cần chứng minh rằng

mn(d−e)

Định nghĩa ma trậnA kích thước(n, m)gồm0và 1sao choaij = 1,hay làyi = cj,i

cj,ilà tọa độ thứicủa véc-tơcj Vì với mọij1, j2 theo định nghĩa khoảng cách mã nhỏ

d(cj1, cj2)dnên số tọa độ trùng véc-tơ khơng qn−d,

cột thứj1 vàj2của ma trậnAcó khơng qn−dsố1chung Nghĩa ma trận không chứa ma

trận kích thước(n−d+ 1,2)gồm tồn số 1.Do đó, số số 1trong ma trận này, gọi làk,

theo định lý Zarankevic, thỏa mãn bất đẳng thức

k n

k n−1

2

m(m−1)(n−d)

(45)

từ đók(k−n)nm(m−1)(n−d)

Các từ y, cj trùng khơng hơnt = n−e >

p

n(n−d)vị trí, theo điều kiện

d(cj, y)e, cột ma trận Acó khơng hơntsố1,từ suy rakmt Ta ln

có thể giả sửk > n, khơng

m <

r

n n−d

nn(d−e)

Do

mt(mt−n)k(k−n)m(m−1)n(n−d)

Từ

m2t2 −mntm(m−1)n(n−d)

Nghĩa

m2(t2−n(n−d))mn(t−n+d), m n(t−n+d)

t2−n(n−d)n(t−n+d) =n(d−e)< n 2.

Định lý chứng minh

8 Mã hóa theo bảng chữ cái

Bảng chữ trở nên quen thuộc hơn, bây giờ, ngoài chữ cái, bắt đầu thấy chữ số, ở dạng thứ tự mà không nhận ngay. Scarlett Thomas, “The End of Mr Y”

Lý thuyết mã hóa lĩnh vực rộng, có mối quan hệ chặt chẽ với đại số, lý thuyết số, tổ hợp, lý thuyết đồ thị, lý thuyết xác suất, lý thuyết thơng tin Nó khơng dừng lại lý thuyết mã sửa sai Một nhánh mã hóa theo bảng chữ Nó khơng dùng với mục đích sửa lỗi, mà, ví dụ đển nén văn bản37), khứ dùng để làm mật mã38).

Mã có tên gọi mã genetic39) coi sơ đồ mã theo bảng chữ cái.

Các toán32, 33, 34liên quan đến lĩnh vực

Bài tốn 32 (Moscow 1962, Vịng 2) Cho2n dãy hữu hạn số 0 1, trong khơng có

dãy phần đầu dãy khác Chứng minh tổng chiều dài dãy khơng nhỏ hơnn·2n.

Bài tốn 33 Trong gia phả Đại công tước Riurik, không hậu duệ của

ơng có qk con trai Trong ngày sinh nhật đại cơng tước có tất cản con cháu tụ tập tính

(46)

Bài toán 34 (Olympic sinh viên khoa Toán-Cơ) Trong danh sách gồm [n từ, tạo thành từ các

chữ bảng chữ gồmk chữ cái, khơng có từ phần đầu từ khác, số

từ có độ dài cho trướcl,vớil= 1,2, , mbằngnl.Chứng minh rằng

n1

k +· · ·+ nm

km1,

dấu xảy ra40.

Trong toán 32thực chất nói độ dài trung bình từ mã sơ cấp mã hóa bảng chữ

cái gồmnchữ bảng chữ gồm hai chữ Quy trình mã hóa từ bảng chữ

cái gồmn chữ cho, ví dụ từ nhị phân, thực cách thay chữ

cái bảng chữ từ mã nhị phân Nếu khơng có từ mã phần đầu từ khóa khác, cách mã hóa gọi tiền tố Điều kiện từ khóa cho phép dễ dàng giải mã41) Trong tốn32thực chất ta cần tìm mã tiền tố với độ dài trung

bình từ mã sơ cấp nhỏ

Bài tốn giải tình tổng quát – chữ bảng chữ cho cho tương ứng với sốpi >0, chop1+· · ·+pn = 1(pi xác suất xuất

hiện chữ này), cịn để mã hóa ta dùng bảng chữ cáik−phân Khi độ dài trung bình từ

mã sơ cấp

lc=p1l1+· · ·+pnln,

trong đóli độ dài từ mã Đối với độ dài trung bình Shannon thu đánh giá sau:

Định lý Ta có bất đẳng thứclcHk(p1, , pn), đó

Hk(p1, , pn) = −(p1logkp1 +· · ·+pnlogkpn),

là entropy Shannon.

Phép chứng minh xuất sau chút, trước hết ta mối liên hệ mã tiền tố bảng chữ cáik−phân câyk−phân có gốc Cây k−phân có gốc định hướng

mà từ gốc (là đỉnh bậc 0) từ đỉnh lớpi xuất

phát không k cạnh hướng tới đỉnh lớpi+

Bằng quy nạp theo thứ tự lớp ta dễ dàng chứng minh số đỉnh bậcikhông ki Theo định nghĩa, đỉnh, trừ gốc, có cạnh vào Các đỉnh mà từ đó

khơng có cạnh nào, gọi lá43) Với tồn đường nối từ gốc

đến Số cạnh đường ta ký hiệu làli, đóilà thứ tự (trong cách

đánh số bất kỳ) Nếu với đỉnh ta cho tương ứng cạnh từ số khác (nhãn) từ tập hợp (bảng chữ cái)Ak ={0,1, , k−1}, đường từ gốc đến lái

có thể cho tương ứng với từwitrong bảng chữ cáiAkbằng cách ghi tất nhãn

các cạnh đường này, cạnh xuất phát từ gốc Tập hợp từwi thu tạo

thành từ mã sơ cấp sơ đồ mã hóa theo bảng chữ cái, xác định cho Hiển nhiên sơ đồ tiền tố

(47)

quy nạp dành lại cho bạn đọc tốn Do có mối liên hệ mã tiền tố cây, khẳng định mã tiền tố phát biểu lại khẳng định Vì từ định lý 6khơng suy đánh giá cho tốn 32,mà cịn tốn 33(được coi mở

rộng tốn32) Cịn tốn34có thể coi hệ định lý7

Định lý (Bất đẳng thức Kraft44). Giả sử câyk−phân có gốc độ dài tất đường

đi từ gốc đến bằngli, i= 1, , n Khi ta có bất đẳng thức

k−l1 +k−l2 +· · ·+k−ln1.

Lời giải Ta tưởng tượng từ lái, từ gốc, mọc thêm nhánh có dạng k−phân đầy đủ chiều caol−li, đól= maxili(ta gọi câyk−phân làđầy đủ chiều caoh,

nếu từ đỉnh có đúngkcạnh ra, cịn tạo thành lớp thứh) Khi nhánh

sẽ cókl−li lá, tất “nở hoa” tạo thành lớp thứlcủa Tất

sẽ

kl−l1 +kl−l2 +· · ·+kl−lnkl,

vì lớp thứlcó khơng qkllá Bây ta cần chia hai vế chokllà xong.

Bây ta chứng minh định lý6.Sử dụng tính chất với hàm sốlogkxvới sốK

thích hợp có bất đẳng thứclogkxK(x−1), có ý nghĩa hình học đơn giản: Đồ thị hàm số

này (ngoại trừ điểm tiếp xúc) nằm tiếp tuyến kẻ điểm (x = 1, y = 0) Thay

vào bất đẳng thức nàyx= k−pli

i , ta thu −li+ logk

pi K

k−li

pi −

,

Từ

liK

1− k− li

pi

+ logk

pi Tiếp theo

piliK pi−k−li

+pilogk

1

pi

Cộng bất đẳng thức lại, ta

p1l1 +· · ·+pnlnK(p1+· · ·+pn−k−l1 − · · · −k−ln) +p1logk

1

p1

+· · ·+pnlogk

1

pn

=K(1−k−l1 − · · · −k−ln) +Hk(p

1, , pn)

Hk(p1, , pn)

theo định lý7.Như vậy, định lý6được chứng minh

Ghi chú:

32) In Galois Fields, full of flowers, primitive elements dance for hours

33) Irving Reed(1923−2012)và Gustav Solomon(1930−1996)– chuyên gia người Mỹ

(48)

34) Richard Collom Singleton(1928−2007)– chuyên gia người Mỹ lý thuyết mã hóa

35) Madlu Sudan (sinh năm 1966) Venkatesan Gurusvami (sinh năm1976) chuyên gia

người Mỹ gốc Ấn tin học

36) Thực chất họ chứng minh điều mạnh nhiều, cụ thể họ đề xuất thuật toán liệt kê danh sách tất từ có khoảng cách khơng vượt q e từ mã 37) Có lẽ với mục đích bảng chữ Morse ví dụ Bảng chữ Morse

là sơ đồ mã hóa theo bảng chữ cái, nhiên khơng tiền tố khơng có tính chất giải mã

38) Ví dụ mật mã thay thế, mật mã Polybia, mật mã Bacon coi sơ đồ mã hóa theo bảng chữ

39) Cho tương ứng axit amin3trong4chữ cáiA, G, S, T

40) Bài toán đề xuất thi olympic vào năm1980,khi chưa có khóa

học sách giáo khoa toán rời rạc

41) Tất nhiên, thay sử dụng mã tiền tố ta sử dụng mã hậu tố, khơng có từ mã sơ cấp đoạn cuối từ mã khác Một điều khác không hiển nhiên tồn mã không tiền tố, khơng hậu tố thỏa mãn tính chất giải mã 42) Bài tốn33sẽ nói khái niệm

43) Người ta sử dụng thuật ngữ “đỉnh treo

44) Bất đẳng thức dành cho mã với tính chất giải mã có phép chứng minh phức tạp gọi bất đẳng thức McMillan

Tài liệu

[1] Берлекэмп Э Р Алгебраическая теория кодирования М.: Мир, 1971

[2] Васильев Н Б., Егоров А А Задачи Всесоюзных математических олимпиад М.: Наука, 1988

[3] Гальперин Г А., Толпыго А К Московские математические олимпиады М.: Про-свещение, 1986

[4] Гашков С Б Разностные множества, конечные геометрии, матрицы Заранкеви-ча и экстремальные графы // Математическое просвещение Сер Вып 21 М.: МЦНМО, 2017 С 145–185

(49)

[6] Левенштейн В И Элементы теории кодирования // Дискретная математика и математическая кибернетика Т М.: Наука, 1974

[7] Мак-Вильямс Ф Дж., Слоэн Н Дж А Теория кодов, исправляющих ошибки М.: Связь, 1979

[8] Прасолов В В и др Московские математические олимпиады 1935–1957 гг М.: МЦНМО, 2010 Коды и олимпиады 173

[9] Прасолов В В и др Московские математические олимпиады 1958–1967 гг М.: МЦНМО, 2013 [10] Бегунц А В и др Московские математические олимпиады 1981–1992 гг М.: МЦНМО, 2017 [11] Фёдоров Р М и др Московские математи-ческие олимпиады 1993–2005 гг / 3-е изд М.: МЦНМО, 2017

[12] Садовничий В А., Григорьян А А., Конягин С В Задачи студенческих мате-матических олимпиад М.: МГУ, 1987 [13] Сидельников В М Теория кодирования М.: Физматлит, 2008

[14] Таранников Ю В Комбинаторные свойства дискретных структур и приложения к криптологии М.: МЦНМО, 2011

[15] Фейеш Тот Л Расположения на плоскости, на сфере и в пространстве М.: Физ-матгиз, 1958 [16] Холл М Комбинаторика М.: Мир, 1970

[17] Чашкин А В Дискретная математика М.: Академия, 2012

[18] Guruswami V., Sudan M Improved decoding of Reed — Solomon and algebraicgeometric codes // IEEE Trans Inform Theory 1999 Vol 45 P 1757–1767

(50)

P

HÉP

T

ỔNG

H

ỢP

B

IỂU

T

HỨC

(R

ESULTANT

)

V

À

M

ỘT

S

NG

D

ỤNG

Benny Lê Văn

G

IỚI

T

HIỆU

Bài viết trình bày sở lý thuyết phép tổng hợp biểu thức (resultant), ứng dụng việc giải toán số thảo luận liên quan theo ánh nhìn tác giả

1 Dẫn nhập

Phép tổng hợp biểu thức (resultant) khái niệm thuộc chuyên ngành đại số trừu tượng (abstract algebra) có nhiều ứng dụng thực tiễn Phép tốn đề cập mơn học đại số đại cương, đại số trừu tượng hay đại số đại Với ứng dụng giải toán mà phép tổng hợp biểu thức mang lại, viết thực nhằm trình bày định nghĩa phép tốn số ví dụ minh họa liên quan Bên cạnh giáo trình đại số trừu tượng, sở lý thuyết phép tổng hợp biểu thức trình bày tương đối đầy đủ Woody [4], ứng dụng liên quan diễn giải chi tiết giảng Lemmermeyer [1] Như bước tiếp nối, viết làm rõ số áp dụng phép tổng hợp biểu thức việc giải toán, đặc biệt tốn sơ cấp Theo đó, phạm trù lý thuyết phép tổng hợp biểu thức giới thiệu phần2;những ứng dụng thực

tiễn trình bày Phần3và Phần4;và Phần5cung cấp số thảo luận dựa

(51)

2 Phép tổng hợp biểu thức

Như diễn giải Woody [4], với hai đa thứcPn.x/vàQm.x/:

8 ˆ ˆ ˆ ˆ ˆ < ˆ ˆ ˆ ˆ ˆ :

Pn.x/D n

Y

jD0

.x ˛j/D n

X

jD0

ajxj

Qm.x/D m

Y

kD0

.x ˇj/D m

X

kD0

bkxk

Phép tổng hợp biểu thức (resultant) hai đa thức ký hiệu Res.Pn; Qm; x/và

xác định sau:

Res.Pn; Qm; x/Damnb n m

16j6n; 1Y6k6m i;j

.˛j ˇk/:

Từ định nghĩa này, ta có nhận xét rằng:

(i) Phép tổng hợp biểu thức có tính giao hoán, tức Res.Pn; Qm; x/ DRes.Qm; Pn; x/:

(ii) Nếu Pn.x/ vàQm.x/ có nhân tử chung phép tổng hợp biểu thức chúng mang

giá trị0, tức Res.Pn; Qm; x/ D0:

Đây hai tính chất quan trọng dẫn đến nhiều áp dụng phép tốn

Bên cạnh đó, người ta [5] chứng minh phép tổng hợp biểu thức hai đa thức biểu diễn thông qua định thức ma trận Sylvester Đây ma trận vuông cấpnCm

được ký hiệu Syl.Pn; Qm; x/và xây dựng sau:

Syl.Pn; Qm; x/D fsc;rg.nCm/.nCm/:

(52)

Như

Res.Pn; Qm; x/ D jSyl.Pn; Qm; x/j D jSyl.Pn; Qm; x/Tj:

Từ định nghĩa phép tổng hợp biểu thức thông qua ma trận Sylvester, xây dựng khái niệm biệt thức (discriminant) đa thức Theo đó, biệt thức đa thức mang giá trị

0thì đa thức có nghiệm trùng Khi đa thức có nghiệm trùng, giá trị đa thức

như đạo hàm đa thức nghiệm trùng là0 Như vậy, biệt thức đa thứcPn.x/với

ký hiệu Disc.Pn/được định nghĩa sau:

Disc.Pn/D

1/n.n21/

an Res

.Pn; P

0

n; x/:

Trong đó, ma trận vng Sylvester có kích thước lànC.n 1/D2n 1:

Chẳng hạn, với tam thức bậc haiP2.x/Dax2CbxCc;ta cóP20.x/D2axCb biệt thức

Disc.P2/D

1/2.221/

a Res.P2; P

0

2; x/ D

1 a ˇˇ ˇˇ ˇˇ ˇˇ

a 2a

b b 2a

c b

ˇˇ ˇˇ ˇˇ ˇˇD

ab2C4a2c 2ab2

a Db

2

4ac:

Một ví dụ khác, với đa thức bậc ba dạng rút gọnP3.y/ Dy3CpyCq;ta cóP

0

3.y/ D3y2Cp

và biệt thức

Disc.P3/D

1/3.321/

a Res.P3; P

0

3; x/D

ˇˇ ˇˇ ˇˇ ˇˇ ˇˇ ˇˇ ˇ

1 p q

0 p q

3 p 0

0 p

0 p

ˇˇ ˇˇ ˇˇ ˇˇ ˇˇ ˇˇ ˇ D ˇˇ ˇˇ ˇˇ ˇˇ ˇˇ ˇˇ ˇ

1 p q

0 p q

0 2p 3q

0 p

0 p

(53)

D ˇˇ ˇˇ ˇˇ ˇˇ ˇˇ

1 p q

0 2p 3q

3 p

0 p

ˇˇ ˇˇ ˇˇ ˇˇ ˇˇ D ˇˇ ˇˇ ˇˇ ˇˇ ˇˇ

1 p q

0 2p 3q

0 2p 3q

0 p

ˇˇ ˇˇ ˇˇ ˇˇ ˇˇ D ˇˇ ˇˇ ˇˇ ˇˇ

2p 3q

0 2p 3q

3 p

ˇˇ ˇˇ ˇˇ ˇˇ D 4p3C27q2/:

Ngoài việc xác định biệt thức, phép tổng hợp biểu thức cịn áp dụng để giải toán liên quan đến phương trình – hệ phương trình, biến đổi đa thức, khử biến (elimination) minh họa phần sau

3 Ứng dụng phép khử biến tìm đa thức hữu tỷ

3.1 Phép khử biến việc giải hệ phương trình

Như tổng hợp Woody [4], phép khử biến (elimination) ứng dụng phép tổng hợp biểu thức việc giải hệ phương trình đại số Ví dụ minh họa sau làm rõ điều

Ví dụ Giải hệ phương trình tập số thực

8 < :

x2y2 25x2C9D0 4xCy D0

Lời giải Để giải hệ trên, ta xác định đa thứcf x/vàg.x/như sau:

8 < :

f x/Dx2y2 25x2C9D0Dx2.y2 25/C9 g.x/D4xCy D0

Theo định nghĩa phép tổng hợp biểu thức thìf x/ Dg.x/D0nếu Res.f; g; x/D 0:

Khử biếnxtrong hệ phương trình trên, ta có

Res.f; g; x/D

ˇˇ ˇˇ ˇˇ ˇˇ

y2 25 4 0

0 y

9 y

ˇˇ ˇˇ ˇˇ ˇˇDy

4 25y2

C144

Như vậy, để giải hệ phương trình ta cần tìmysao cho: y4 25y2C144D0:

Phương trình trùng phương có 4nghiệm y f4; 4; 3; 3g:Từ hệ phương trình có

(54)

Khi giải hệ phương trình dựa vào phép khử, ta cần lưu ý số điểm:

(i) Có thể khử biến giúp cho việc giải thuận tiện ii) Cần thử lại nghiệm tìm để đến đáp án cuối

3.2 Phép khử biến hình học giải tích

Trong hình học giải tích, đơi quan hệ biến biểu thị qua hàm tham số có dạng:

x D p.t /

q.t /; y D s.t / r.t /:

Theo giảng Lemmermeyer [1], phép khử biến t giúp ta tìm mối liên hệ trực

tiếp giữaxvày:Thật vậy, xét hàm số sau

8 < :

F t / Dxq.t / p.t / G.t / Dyr.t / s.t /

Theo đó, biếnt khử ta tìm mối liên hệ giữaxvày choF t /DG.t / D0:

Tức ta cần tìmx vày cho Res.F; G; t / D 0:Ví dụ sau minh họa rõ cho ứng

dụng phép tổng hợp biểu thức

Ví dụ Tìm mối liên hệ trực tiếp giữaxy, biết rằng x D t

2 1

t2C1; y D

2t

t2C1; t R:

Lời giải Xét hàm sốF t /vàG.t /được xác định sau

8 < :

F t /Dx.t2C1/ t2 1/D.x 1/t2CxC1 G.t /Dy.t2C1/ 2t Dyt2 2t Cy

Tiếp đến, ta cần tìmx vàysao cho Res.F; G; t / D0;trong

Res.F; G; t /D

ˇˇ ˇˇ ˇˇ ˇˇ ˇˇ

x y

0 x y

xC1 y

0 xC1 y

ˇˇ ˇˇ ˇˇ ˇˇ ˇˇ

D x 1 ˇˇ ˇˇ ˇˇ ˇˇ ˇˇ

x y

0 x y

0 2y 2.x 1/

0 xC1 y

ˇˇ ˇˇ ˇˇ ˇˇ ˇˇ D ˇˇ ˇˇ ˇˇ ˇˇ

x y

0 2y 2.x 1/

xC1 y

ˇˇ ˇˇ ˇˇ ˇˇD x ˇˇ ˇˇ ˇˇ ˇˇ

x y

0 2y 2.x 1/

0 2.xC1/ 2y

ˇˇ ˇˇ ˇˇ ˇˇD4

ˇˇ ˇˇ ˇ

y x 1/

xC1 y

(55)

D4.x2Cy2 1/:

Như vậy,x2Cy2 D1chính mối liên hệ giữax vàymà ta cần tìm.

Ngồi ra, ví dụ thay thết Dtan với 2Œ0; 2thì ta cóxD cos.2 /và y Dsin.2 /;điều dẫn đếnx2Cy2 D1:

3.3 Đa thức hữu tỷ có nghiệm vơ tỷ

Phép tổng hợp biểu thức áp dụng để tìm đa thức hữu tỷ mà ta biết trước nghiệm vơ tỷ Chúng ta xem xét ví dụ sau

Ví dụ Tìm đa thức hữu tỷh.x/có nghiệm làx0D

p

2Cp3 3:

Lời giải Theo diễn giải Lemmermeyer [1], với đa thức hữu tỷf x/vàg.x/lần lượt có

các nghiệm là˛ vàˇthì đa thức hữu tỷh.x/có nghiệm là˛Cˇđược xác định sau

h.x/DResŒf x y/; g.y/; y: (1)

Trong ví dụ này, ta có 8

< :

f x/Dx2 2; ˛ Dp2 g.x/Dx3 3; ˇ D p3

Theo (1), đa thức hữu tỷh.x/đươc xác định sau

h.x/DResŒf x y/; g.y/; yDResŒ.x y/2 2; y3 3; yDResŒy2 2xyCx2 2; y3 3; y

D ˇˇ ˇˇ ˇˇ ˇˇ ˇˇ ˇˇ ˇ

1 0

0 2x

0 x2 2x

3 0 x2 2x

0 0 x2

ˇˇ ˇˇ ˇˇ ˇˇ ˇˇ ˇˇ ˇ D ˇˇ ˇˇ ˇˇ ˇˇ ˇˇ ˇˇ ˇ

1 0

0 2x

0 x2 2x

0 x2 2x

0 0 x2

ˇˇ ˇˇ ˇˇ ˇˇ ˇˇ ˇˇ ˇ D ˇˇ ˇˇ ˇˇ ˇˇ ˇˇ

1 2x

0 x2 2 2x 1

0 x2 2x

3 0 x2

ˇˇ ˇˇ ˇˇ ˇˇ ˇˇ D ˇˇ ˇˇ ˇˇ ˇˇ ˇˇ

1 2x

0 x2 2 2x 1

0 x2 2x

0 6x x2

ˇˇ ˇˇ ˇˇ ˇˇ ˇˇ D ˇˇ ˇˇ ˇˇ ˇˇ

x2 2 2x 1

3 x2 2x

6x x2

ˇˇ ˇˇ ˇˇ ˇˇ Dx6 6x4 6x3C12x2 36xC1:

(56)

Một cách tương tự

Đa thức hữu tỷ.x/có nghiệm là ˛ˇ được xác định theo công thức

.x/DResŒf xy/; g.y/; y: (2)

Như ví dụ

Ví dụ Tìm đa thức hữu tỷ.x/có nghiệm làx1 D

p

2

3

p

3:

Lời giải Áp dụng công thức (2), ta có

.x/DResŒf xy/; g.y/; yDResŒx2y2 2; y3 3; yD9x6 8:

Thật vậy, dox61 D 89 nên.x1/D0:

4 Ứng dụng phương trình đại số

4.1 Hệ thức liên quan đến nghiệm phương trình đại số

Phần trình bày ứng dụng phép tổng hợp biểu thức việc chứng minh hệ thức liên quan đến nghiệm phương trình đại số Ta xem xét ví dụ sau

Ví dụ Cho số thựcathỏa điều kiện.a 1/3 D9a;đặtb Da2Ca:Chứng minh rằng bC1/3 D27b2:

Lời giải Ví dụ giải nhiều cách biến đổi hệ thức giải trực tiếp

phương trình bậc ba Với phép tổng hợp biểu thức, đặtxDa 1;ta có

8 < :

x3 9x 9D0

bDa.aC1/D.xC1/.xC2/Dx2C3xC2

Xây dựng hàm số sau

8 < :

f x/Dx3 9x

(57)

Khử biếnxtrong hệf x/Dg.x/D0;hệ thức độc lập củab xác định

Res.f; g; x/D

ˇˇ ˇˇ ˇˇ ˇˇ ˇˇ ˇˇ ˇ

1 9

0 9

1 b 0

0 b

0 b

ˇˇ ˇˇ ˇˇ ˇˇ ˇˇ ˇˇ ˇ D ˇˇ ˇˇ ˇˇ ˇˇ ˇˇ ˇˇ ˇ

1 9

0 9

0 b 11

0 b

0 b

ˇˇ ˇˇ ˇˇ ˇˇ ˇˇ ˇˇ ˇ D ˇˇ ˇˇ ˇˇ ˇˇ ˇˇ

1 9

3 b 11

1 b

0 b

ˇˇ ˇˇ ˇˇ ˇˇ ˇˇ D ˇˇ ˇˇ ˇˇ ˇˇ ˇˇ

1 9

0 b 11 36 27

0 b 11

0 b

ˇˇ ˇˇ ˇˇ ˇˇ ˇˇ D ˇˇ ˇˇ ˇˇ ˇˇ

b 11 36 27

3 b 11

1 b

ˇˇ ˇˇ ˇˇ ˇˇ Db3 24b2C3bC1:

Như vậy, hệf x/Dg.x/D0có nghiệm khib3 24b2C3bC1D0;tức là.bC1/3 D27b2;

đây điều phải chứng minh

Ví dụ thực chất phép biến đổi Tschirnhaus bậc hai phương trình bậc ba Phần khảo luận rõ vấn đề

4.2 Giải phương trình bậc ba theo phương pháp Tschirnhaus

Bên cạnh phương pháp giải phương trình bậc ba phổ biến [6], Tschirnhaus [3] đề xuất cách tiếp cận dựa vào phép biến đổi đa thức (Tschirnhaus transform) Theo đó, ứng dụng khử biến (elimination) phép tổng hợp đa thức công cụ đắc lực cho cách tiếp cận Xét phương trình bậc ba tổng quát có dạng

f x/Dx3C3ax2CbxCc D0:

Tschirnhaus đề xuất phép đổi biến x Dpn.y/vào phương trình trên, đópn.y/

một đa thức bậcn < 3;với mục tiêu loại bỏ số hạng bậc hai bậc đa thức bậc ba

tổng quát Vớix Dy a;phương trình bậc ba tổng quát trở thành hệ phương trình sau

8 < :

(58)

Vận dụng phép tổng hợp biểu thức f x/và g.x/liên quan đến việc khử biến x hệ

trên, ta cần có Res.f; g; x/D0;với

Res.f; g; x/D

ˇˇ ˇˇ ˇˇ ˇˇ ˇˇ

1 3a b c

1 y a 0

0 y a

0 y a

ˇˇ ˇˇ ˇˇ ˇˇ ˇˇ

Dy3CpyCq:

Trong đóp vàqđược xác định sau

8 < :

pDb 3a2 qD2a3 abCc

Với phép biến đổi Tschirnhaus bậc trên, ta loại hệ số bậc hai phương trình cần tìm nghiệm cho phương trình bậc ba dạng rút gọny3CpyCq D0:

Ý tưởng loại bỏ hệ số bậc Đổi biếnz D y2CuyCv;đây phép biến

đổi Tschirnhaus bậc hai Ta xét hệ phương trình sau

8 < :

h.y/Dy3CpyCq D0 k.y/Dz y2Cuy Cv/D0

Hệ phương trình dẫn đến Res.h; k; y/D0;trong

Res.h; k; y/ D

ˇˇ ˇˇ ˇˇ ˇˇ ˇˇ ˇˇ ˇ

1 0

0 u

p z v u

q p z u u

0 q 0 z v

ˇˇ ˇˇ ˇˇ ˇˇ ˇˇ ˇˇ ˇ

Dz3Crz2Csz Ct:

Vậyz3Crz2CszCt D0;và để loại bỏ hệ số bậc hai bậc phương trình này,

ta cần tìmuvà v chor Ds D 0:Tất nhiên vớip D 0;phép biến đổi Tschirnhaus bậc hai

này khơng cần thiết Do đó, ta gi s rngp Ô0:

Trong Res.h; k; y/;ta cúr D2p 3v;đểr D0thì ta cầnv D 2p3 ;khi s Dpu2C3qu p

2

3 :

(59)

tức

u D 6p

h

9q ˙p3.4p3C27q2/i:

Trong công thức trên, việc u số thực hay số phức phụ thuộc vào biểu thức 4p3C27q2/;đây biệt thức phương trình bậc ba dạng rút gọn trình

bày phần2;như

t Dqu3 2p

2

3 u

2 pqu

2p3 27 Cq

2

:

Theo đó, tập số phức C;ta tìm đượcz D p3

t ; từ ta giải đượcy thơng qua phương

trình bậc hai

y2Cuy Cv z D0;

hay

y D 2Œ u˙

p

u2 4.v z/:

Và cuối cùng,x Dy a:

Với cách tiếp cận phép đổi biến Tschirnhaus bậc hai phương trình bậc ba, ta biến đổi trực tiếp dạng tổng quát dạngz3Ct D0:Tuy nhiên, phép đổi biến Tschirnhaus bậc

nhất bước trung gian giúp làm giảm khối lượng tính tốn đáng kể

Ngồi ta, với cách tiếp cận này, từ ba giá trị củaz;ta tìm sáu giá trị củay tương ứng

sáu giá trị củax:Vì thế, việc thử lại phương trình dạng rút gọn dạng tổng quát

là cần thiết để có tập nghiệm xác

5 Một số thảo luận

Xuyên suốt viết này, khảo luận sở lý thuyết số ứng dụng phép tổng hợp biểu thức (resultant) Thay lời kết, tác giả thảo luận thêm hai vấn đề,

(i) Thuật ngữ“resultant”trong tiếng Việt

(ii) Quan điểm tác giả việc sử dụng phép tổng hợp biểu thức giải toán sơ cấp

(60)

biểu đạt danh từ “resultant” đại số trừu tượng Trong trình tìm cách tốt để biên dịch thuật ngữ “resultant” sang tiếng Việt, tác giả có tham khảo số tài liệu viết ngơn ngữ có văn hóa gần với Việt Nam Theo đó, chữ “resultant” người Trung Quốc gọi “kết thức” (結式) người Nhật Bản gọi “chung kết thức” (終結式) Thiết nghĩ, thật không phù hợp ta mượn phiên âm chữ Trung Quốc hay Nhật Bản để dịch chữ “resultant” sang tiếng Việt Như vậy, “phép tổng hợp biếu thức” kỳ vọng đóng góp nhỏ nhoi xây dựng cho sáng mẻ tiếng Việt, theo tinh thần ông Ngơ Quang Châu [2]

Thứ nhì, việc áp dụng phép tổng hợp biểu thức vào giải toán sơ cấp Có thể nói khái niệm thuộc đại số trừu tượng phép tổng hợp biểu thức cơng cụ mạnh mẽ để giải tốn sơ cấp Do đó, việc vận dụng gặp phải số phản biện cách giải tốn khơng tương thích với phạm vi, khn khổ chương trình học hay kỳ thi Về vấn đề này, tác giả quan niệm việc áp dụng công cụ cho “ngoài giới hạn” nâng cao tinh thần sáng tạo chia sẻ kiến thức tốn học Bởi lẽ, khơng nên suy nghĩ bên “chiếc hộp”, khơng nên suy nghĩ ngồi “chiếc hộp”, mà nghĩ chẳng có “chiếc hộp

6 Lời cám ơn

Tác giả xin gửi lời cám ơn chân thành đến Tiến sĩ Trần Nam Dũng lời khuyến khích, động viên ý kiến quý báu Thầy viết hoàn thành

Tài liệu

[1] Lemmermeyer F.,Resultants, Department of Mathematics – Faculty of Science – University of Bilkent (2005), 41-46

[2] Ngô Quang Châu,Luận tiếng Nam, Nhà xuất Đời (1943)

[3] Tschirnhaus E W v.,A method for removing all intermediate terms from a given equation, This Bulletin, Vol 37 (1), No 143 (2003), 1-3 [Nguyên bản: Methodus auferendi omnes terminus intermedios ex data equation, Acta Eruditorum, (1683), 204-207]

(61)

[5] https://math.stackexchange.com/q/2268111

(62)

BÀI TỐN HƠN NHÂN BỀN VỮNG

Lương Văn Khải - Võ Thành Đạt

(SV trường ĐH Khoa học tự nhiên TPHCM)

1 Giới thiệu toán

1.1 Bài tốn nhân bền vững gì?

Trong tốn học khoa học máy tính, tốn nhân bền vững (SMP) yêu cầu tìm cặp ghép bền vững phần tử hai tập hợp theo thứ tự ưu tiên phần tử Một cặp ghép ánh xạ từ phần tử tập hợp tới phần tử tập hợp Một cặp ghép bền vững hai điều kiện sau không đồng thời xảy ra:

Một phần tửAcủa tập hợp thứ thích phần tửBcủa tập hợp thứ hai phần tử ghép

vớiA, vàB thíchAhơn phần tử ghép vớiB Nói cách khác, tổ hợp ghép bền

vững không tồn cặp(A, B)trong cảAvàB thích phần tử phần tử

ghép với chúng

Bài tốn ”hơn nhân bền vững” phát biểu đơn giản sau:

Hãy tìm cách ghép cặp vợ-chồng cho n chàng trai vàm cô gái cho

hôn nhân "bền vững" Tức không tồn hai cặp mà người chồng bên cặp lại thích người vợ bên cặp vợ mình, đồng thời người vợ bên cặp thích người chồng bên cặp chồng mình, hai người ngoại tình

(63)

Năm 1960, hai nhà toán học người Mỹ Lloyd Stowel Shapleyl1và David Gale2quan sát thực

tiễn nhận thấy nhiều hoạt động liên quan đến việc ghép đơi, tạo mối quan hệ, ví dụ gặp gỡ kí kết hợp đồng người mua người bán hay tuyển sinh, v.v

Hình 1: Lloyd Stowel Shapleyl (trái) David Gale

Để giải vấn đề này, họ tìm thuật tốn DAA (Deferred Acceptance Algorithm) hay cịn gọi thuật tốn chấp nhận trì hỗn cơng bố vào năm 1962 báo mang tên "Tuyển sinh đại học ổn định hôn nhân (College Admissions and the Stability of Marriage)" Đến năm 1980, nghiên cứu tắc nghẽn việc phân bổ lao động số thị trường Mỹ, Alvin Eliot Roth3đã nhận thấy tương đồng thị trường với mơ

hình tốn học D Gale L Shapley nghiên cứu trước đó, ơng nghiên cứu thiết kế lại thị trường dựa thuật toán DAA

Năm 2012, Alvivin E.Roth Lloyd S.Shapley trao giải Nobel Kinh tế cho cống hiến “Lý thuyết phân phối ổn định thực tiễn tạo lập thị trường” (The theory of stable allocations and the practice of market design)

Ngày nay, tốn nhân bền vững có nhiều ứng dụng sống (y tế, giáo duc, ) Trong phần cuối, nói ứng dụng tốn nhân bền vững cách xét tuyển vào đại học Việt Nam nói riêng giới nói chung

1Lloyd Stowell Shapley (2/6/1923 - 12/3/2016) nhà toán học kinh tế học người Mỹ Ông Giáo sư

danh dự đại học California, Los Angele Ơng đóng góp vào lĩnh vực kinh tế toán học đặc biệt lý thuyết trị chơi Năm 2012, ơng giải Nobel kinh tế Alvin Elliot Roth

2David Gale (13/12/1921 – 7/3/2008) nhà toán học kinh tế học người Mỹ Ông giáo sư danh dự tại

Đại học California, Berkeley Ơng đóng góp cho lĩnh vực kinh tế tốn học, lý thuyết trị chơi phân tích lồi

3Alvin Eliot Roth nhà kinh tế Mỹ, giáo sư thỉnh giảng đại học Stanford giáo sư

(64)

Hình 2: Alvin Eliot Roth

2 Tìm hiểu tốn

2.1 Cơ sở lý thuyết

2.1.1 Một số khái niệm

Định nghĩa 1. Một cách ghép cặp chàng trai cô gái gọi không bền vững có hai cặp vợ chồng(B1, G1)(B2, G2)sao cho chàng traiB1 thích gáiG2 hơn

cơ vợG1 của mình, đồng thời gáiG2cũng thích chàng traiB1 hơn anh chồngB2của mình.

Định nghĩa 2. Một cách ghép cặp bền vững gọi tối ưu chàng trai lấy được người vợ ưng ý so với cách phân bố bền vững khác.

2.1.2 Các vấn đề đặt ra

Vấn đề 1(Marriage problem). Có tồn hay khơng cách ghép cặp chonchàng trai vớin

gái cho lấy người có cảm tình (một người có cảm tình với nhiều người khác giới)? Nếu tồn cách ghép.

Lời giải. Câu trả lời có Vấn đề giải nhà Tốn học Philip Hall vào năm 1935 Ơng tìm điều kiện cần đủ để tồn cách ghép thỏa mãn u cầu, nội dung "định lý hôn nhân" (Hall’s marriage theorem) Xem [8]

Vấn đề 2 (Stable Marriage Theorem). Có thể ghép cặp hôn nhân bền vững hay không?

(65)(66)

Hình 4: Hai cặp nhân khơng bền vững

2.2 Thuật toán

Định lý 1. Nếu số chàng trai số gái ln tồn cách ghép cặp vợ-chồng cho họ cho hôn nhân bền vững.

Chứng minh. D Gale L Shapley chứng minh định lý cách đưa thuật toán DAA Thuật toán mơ tả vịng lặp sau:

Bước 1:Các chàng trai gái lên danh sách thứ tự cô gái chàng trai mà muốn lấy (theo thứ tự thích nhất, thích nhì, thích ba, )

Bước 2:Lần lượt chàng trai cầu gái đứng đầu danh sách Cơ gái nhận nhiều lời cầu chọn người mà thích (dựa theo danh sách mình)

◦ Nếu người chọn người đứng đầu danh sách gái kết hôn với người từ chối lời cầu hôn khác lượt

◦ Cịn khơng phải gái tìm cách “trì hỗn” việc tạm nhận lời,

coi có “hơn phu tạm thời” chờ đợi lời cầu từ người có thứ hạng cao Nếu người xuất cầu gái (ở lượt sau) dĩ nhiên cô chia tay người "hôn phu tạm thời" mà chọn người • Bước 3:Các chàng trai xóa tên gái từ chối chia tay với khỏi

danh sách

Bước 4:Lặp lạibước 2

(67)

Hình 5: Một cách ghép cặp nhân bền vững

Nhận xét.

1 Thuật toán DAA thực hữu hạn vòng lặp.

Chứng minh. Ở lượt, chàng trai cầu hôn với cô gái, gái từ chối chia tay với khơng cầu lại mà phải cầu hôn cô gái khác lượt sau, số vịng lặp khơng thể vượt quán2.

2 Sau thuật toán kết thúc, người kết (khơng cịn độc thân).

Chứng minh. Điều dễ dàng thấy do:

• Sau thuật tốn kết thúc, chàng trai khơng cịn phải cầu nữa, tức anh làm chồng "hôn phu tạm thời"

• Mỗi gái nhận lời cầu khơng có gái có chồng "hơn phu tạm thời"

Tóm lại gái ngỏ lời chàng trai nhận lời nên người kết

3 Bằng việc thực thuật tốn trên, hôn nhân thiết lập bền vững.

Chứng minh. Giả sử tồn hai cặp vợ chồng(B1, G1)và (B2, G2)màB1 thíchG2

cơ vợ G1 mình, đồng thờiG2 thíchB1 anh chồngB2 Khi rõ

ràngB1phải cầu hơnG2 trướcG1 Xét thời điểmB1cầu hơnG2:

• Nếu trước đóB2 cầu hôn "hôn phu tạm thời" củaG2 lúc

(68)

• NếuB2 cầu hơnG2 sauB1 chắnB2 bị từ chối vìG2 thíchB1

B2

Khi đó, B2 luôn bị từ chối chắn chồng củaG2 được, mâu

thuẫn

4 Thuật tốn DAA áp dụng trường hợp số chàng trai không số cô gái. Cụ thể:

Nếun < mthì thuật tốn dừng khincơ gái cầu hơn.

Nếu m < nthì thuật tốn dừng chàng trai rơi vào hai trạng

thái sau:

Là hôn phu tạm thời cô gái đó.

Bị tất gái từ chối.

5 Nếu cô gái cầu chàng trai áp dụng thuật toán DAA để cho ra cách ghép cặp bền vững Hai cách ghép cặp hai trường hợp khác nhau Ngồi ra, bên bên cầu kết thuật tốn tối ưu bên đó.

Chứng minh. Xem phần

2.3 Tính tối ưu

Định lý 2. Trong cách ghép cặp bền vững, kết thuật toán DAA tối ưu nhất, tức là các chàng trai lấy người vợ mà ưng ý nhất, so với cách ghép bền vững khác.

Chứng minh. Xét tất cách ghép cặp bền vững Đối với chàng trai, gọi vùng khả thi tập hợp tất gái mà cưới cách ghép đôi bền vững Định nghĩa tương tự với cô gái Ta chứng minh phản chứng

Gọi B chàng trai bị gáiGmà thích vùng khả thi từ

chối Điều chứng tỏGđã có phu tạm thời làB0 vàGthíchB0 hơnB

GọiG0là gái màB0thích vùng khả thi mình.

• TH1:GkhácG0.

Xét danh sách (lập bước thuật tốn) củaB0, ta thấyGphải có thứ hạng cao

hơnG0, khơng thìB0phải cầu hơnG0 trướcGvà bị từ chối, tức làB0sẽ bị người thích

nhất vùng khả thi từ chối Điều vơ lý kiệnB0 bịG0 từ chối xảy

ra trước kiện B bịG từ chối, hayB chàng trai bị từ chối

người thích vùng khả thi minh (trình tự cụ thể:B0 cầu hơnG0 từ

chối, tiếp theoB0 cầu hônG, đượcGcho làm hôn phu tạm thời, sau đóBmới cầu hơnG

và bịGtừ chối) Tóm lại, danh sách củaB0 thìGcó thứ hạng cao hơnG0.

Mặt khác, ta xét cách ghép cặp bền vững mà đóB vàGkết với nhau, cịnB0

(69)

• TH2:Gchính làG0.

Tương tự TH1

Nhận xét.

1 Các cô gái lấy chàng trai có thứ hạng thấp vùng khả thi Như vậy, bên cầu lợi bên cầu hôn.

Chứng minh. Giả sử tồn gái G kết với chàng trai B

chàng trai xếp cuối vùng khả thi cô ta Theo định lý 2, ta cóGchính gái đứng

đầu vùng khả thi củaB Xét cách ghép cặp bền vững màGkết hôn vớiB0 là chàng trai

xếp cuối vùng khả thi ta Khi ta thấy GthíchB hơnB0, khiB phải kết

hơn với gái có xếp hạng thấp hơnG, mâu thuẫn với tính bền vững

2 Nếu có cách ghép cặp bền vững tính tối ưu chàng trai các cô gái nhau.

Chứng minh. Vì vùng khả thi người lúc có người

2.4 Mơ hình Code

2.4.1 Mơ hình

INPUT:

• Tập chàng traiB ={b1, b2, , bn}

• Tập cô gáiG={g1, g2, , gn}

• Danh sách xếp hạng u thích người, gọiPbi(gj)(Pgi(bj)) thứ

hạng mà bạn nam (nữ) thứiđánh giá bạn nữ (nam) thứj danh sách xếp hạng

u thích

OUTPUT: Tp hụn nhõn bn vngSBìG

ã |S|=n

ã ∀b∈B,∃!g ∈G,(b, g)∈S

• ∀g ∈G,∃!b ∈B,(g, h)∈S

ã ơ(b, g)/ S : ((b, g0)S)((b0, g)S)(P

(70)

2.4.2 Code (Python)

def deferred_acceptance_algorithm(man_pref, woman_pref, n):

"""

Thuật tốn chấp nhận trì hỗn (Gale-Shapley Algorithm) # LƯU Ý: thuật toán cài đặt mã nguồn đánh index từ 0 :param man_pref: list(list)

- danh sách yêu thích m thứ i man_pref[i] :param woman_pref: list(list)

- danh sách yêu thích w thứ i woman_pref[i] :param n: int - số lượng cặp đôi

:return stable_marriage: list(tuple)

- tuple(m, w) cặp hôn nhân """

# Người mà w hẹn hò: fall_in_love[w] = m # Nếu w độc thân: fall_in_love[w] = -1

fall_in_love = [-1] * n

# Những người đàn ông độc thân

free_man = list(range(n))

# Lưu thứ hạng w thích m

ranking = [[0] * n for i in range(n)]

for i in range(n):

for j in range(n):

ranking[i][woman_pref[i][j]] = j

# chiến thuật tỏ tình bắt đầu

while free_man:

# xét m độc thân

m = free_man[0]

# tỏ tình với w mà m thích nhất

w = man_pref[m].pop(0)

# w chưa hẹn hò

if fall_in_love[w] == -1:

# hẹn hị m, m khơng độc thân

fall_in_love[w] = m

free_man.pop(0)

else:

# w thích m m'

if ranking[w][m] < ranking[w][fall_in_love[w]]:

# m hết độc thân, m' độc thân

(71)

free_man.append(fall_in_love[w]) # m w hẹn hò

fall_in_love[w] = m

# Danh sách [(m,w), ] hôn nhân bền vững

stable_marriage = [(fall_in_love[i], i) for i in range(n)]

# trả tập hôn nhân bền vững

return stable_marriage # Chạy thử code

n =

man_pref = [[1,0,2,3],

[3,0,1,2], [0,2,1,3], [1,2,0,3]]

woman_pref = [[0,2,1,3],

[2,3,0,1], [3,1,2,0], [2,1,0,3]]

result = deferred_acceptance_algorithm(man_pref, woman_pref, n)

print(result) # Kết quả

[(0, 0), (3, 1), (2, 2), (1, 3)]

3 ỨNG DỤNG

Thuật toán DAA có nhiều ứng dụng thực tế:

• Y tế: phân phối nội tạng hiến tặng đến bệnh nhân để đảm bảo tương đối tính cơng mang lại lợi ích nhiều (trao đổi thận thân nhân Hoa Kì) • Công việc: Phân công tân bác sĩ đến bệnh viện (Chương trình quốc gia phân

bổ bác sĩ nội trú Hoa Kì NRMP áp dụng)

• Giáo dục: Phân bổ học sinh trung học New York nhiều thành phố khác Chúng ta tìm hiểu ứng dụng toán

3.1 Bài toán cho nhận ghép tạng

(72)

được coi thành cơng thận người hiến tương thích với thể người nhận Nếu thận khơng tương thích, bệnh nhân đưa vào "danh sách chờ".4

Theo Alvin, có nhiều thân nhân muốn hiến thận khơng thể ghép cho người nhà khơng tương thích Điều tạo thực tế phải thiết kế chế cho phép người có thân nhân cần thận muốn hiến thận để cứu người nhà nối kết với nhằm hiến thận tạo điều kiện cho người thân ghép thận

Với tảng tốn nhân bền vững, Alvin Roth cộng thiết kế chế giúp bệnh nhân hốn đổi người hiến thận cho nhau, qua làm tăng số lượng ca ghép thận thành công, cứu sống thêm nhiều bệnh nhân Xét hai cặp vợ chồng (H, W)và (H0, W0)với hai người vợW, W0 cần ghép thận Lý tưởng ông chồng hiến thận

cho vợ mình:Hhiến thận choW,H0hiến thận choW0 Tuy nhiên, thận màH lại khơng

tương thích với vợ mìnhW, tương tự với H0 vàW0 Nếu may mắn,H có thể hiến thận choW0

cịnH0 sẽ hiến thận choW Nếu khơng may mắn, bác sĩ phải tìm cặp vợ chồng khác

(H00, W00) để tạo thành chu trình H hiến thận choW0, H0 hiến thận choW00 H00 hiên

thận choW Chu trình tiếp tục giúp hình thành chuỗi, ví dụW0 nhận

từHcịnW nhận từ người khác danh sách

3.2 Bài tốn ghép đơi bác sĩ thực tập bệnh viện

Hãy tưởng tượng có tình cảnh sau:

• Bác sĩAvừa chấp nhận hợp đồng thực tập phòng khámX

• Sau tháng, bệnh việnY gọi điện mời bác sĩAtới kí hợp đồng khơng qua thực tập,

với lí "đã biết thành tích bác sĩ Aở trường" Hợp đồng bệnh viện Y cung

cấp mức lương gấp đôi hợp đồng thức bệnh việnX, kèm theo phụ cấp hậu

hĩnh khác Bác sĩAbỏ qua hợp đồng phịng khámX đầu qn cho bệnh việnY

• Bị "nẫng tay trên", phòng khám X xem lại danh sách ứng viên chọn ứng viên

khác, ứng viên thấy phịng khámX chọn bỏ việc bệnh việnZ sang làm

cho phòng khámX

• Bạn bác sĩ Alà bác sĩB, làm bệnh việnZ, nghe chuyện bác sĩAkể lại, nộp

đơn vào bệnh việnY Hồ sơ bác sĩB hố lại bệnh việnY cần,

để nhận bác sĩB, họ sa thải bác sĩ trẻ khác họ

Rất hỗn loạn Đó mà bệnh viện nói riêng cơng ty nói chung phải đối mặt khứ Trước đây, "cuộc chiến" bệnh viện để chọn sinh viên

4Theo luật pháp Hoa Kỳ, việc mua bán thận bị coi dạng tội hình Vì thế, người cần ghép thận

(73)

phù hợp bệnh viện ln "khốc liệt" Sinh viên Y Dược trường khó để xác định trình độ đâu so với đối thủ cạnh tranh, khó xác định đâu bệnh viện phù hợp với lực Điều thường dẫn tới định sai lầm hai phía: bệnh viên chọn sinh viên có lực khơng đáp ứng nhu cầu mình, sinh viên vào bệnh viện q sức với Chưa kể cịn vài vấn đề phát sinh: cặp bác sĩ cưới muốn vào chung bệnh viện, bác sĩ muốn bệnh viện gần nhà,

Hình 6: Ghép đơi bác sĩ với bệnh viện

Chính Alvin Roth, với Elliott Peranson, thiết kế chương trình National Resident Match-ing Program (NRMP) dựa sở tốn nhân bền vững, giúp “gán ghép” 20.000 bác sĩ tập với sở y tế Xét mơ hình NRMP với ba bác sĩ1,2,3và ba bệnh viện

a, b, c(hình trang bên) xếp theo thứ tự chất lượng từ xuống

• Khi bác sĩ chọn bệnh viện, trước hết họ chọn bệnh viện a với mức đãi ngộ cao

nhất, môi trường làm việc tốt nhất,

• Trong số3ứng viên nộp vào, bệnh việnachọn bác sĩ1, ưu tiên số bệnh

việna

• Với hai ứng viên2,3và hai bệnh việnb, ccịn lại, trước hết bác sĩ2nộp hồ sơ vào bệnh

việnb (ưu tiên số hai) bác sĩ3 nộp vào bệnh việnc(ưu tiên số hai) Tới đây, NRMP

xuất tìm giải pháp tối ưu: bác sĩ 2được chuyển sang bệnh viện cvà bác sĩ

được chuyển sang bệnh việnb

Thuật toán áp dụng với số bệnh viện số bác sĩ

(74)

3.3.1 Mơ hình

Cómtrường đại học, cao đẳng Trường thứituyểnai học sinh Tổng cộng cóS ≥a1 +· · ·+

ai+· · ·+anhọc sinh tham gia đợt tuyển sinh Giả sử trường sử dụng tiêu chuẩn

nào cho lập danh sách thứ tự học sinh theo tiêu chuẩn khơng có học sinh mức độ Khi trường bảng xếp hạng học sinh Trong trường hợp này, ta áp dụng thuật toán Gale – Shapley sau:

1 Nếu có học sinh chưa xếp vào trường danh sách người khác rỗng thì: • Xét trường đứng đầu danh sách học sinh đó, trường chưa có đủ học sinh

theo tiêu, ta xếp học sinh xét vào trường

• Ngược lại, trường đủ học sinh, ta so sánh hạng học sinh xét bảng xếp hạng trường với học sinh có học sinh danh sách học sinh trường, học sinh xét có thứ hạng cao người xếp hạng cuối danh sách học sinh trường ta loại học sinh ra, đưa học sinh xét vào Cuối ta xóa trường khỏi danh sách xếp hạng xét Nếu khơng cịn tồn học sinh có danh sách xếp hạng khác rỗng, ta kết thúc thuật

toán, danh sách học sinh trường trở thành danh sách thức

Thuật tốn ln đảm bảo học sinh có điểm số cao vào trường đó, học sinh top đầu vào trường, ngành yêu thích ngành, trường tốt chọn học sinh giỏi Nói cách khác, thuật tốn đảm bảo tính cơng cho học sinh nhiều lợi ích

Một đặc tính ưu việt DAA (student-proposer) việc thí sinh khơng thể can thiệp vào kết Đây tính chất đáng cân nhắc Hiện nhiều thí sinh Việt Nam lựa chọn trường dựa vào khả trường nhận học (thi đỗ) mà khơng quan tâm nhiều đến giá trị ngành học, trường học Hệ thống xét tuyển sử dụng DAA hiệu thí sinh nộp đơn với danh sách nguyện vọng thực tâm

3.3.2 Một số vấn đề phát sinh

Có thật là, mơ hình ghép cặp bác sĩ với bệnh viện có điểm khác với mơ hình tuyển sinh đại học Có thể kể đến vài điểm sau:

(75)

2 Người trưởng thành, tìm việc tâm lý thái độ khác với học sinh học đại học Thay vào đó, mơ hình sinh viên sư phạm trường xin việc giống với mơ hình ghép cặp bác sĩ - bệnh viện

3 DAA hạn chế quyền độc lập tuyển sinh trường

Ba điều dẫn tới hệ là, DAA tạo số khơng hài lịng cho nhiều bên, thí sinh, gia đình, trường đại học, Điều dẫn tới việc, muốn áp dụng DAA vào việc tuyển sinh đại học Việt Nam, cần có cân nhắc nhiều yếu tố Theo thành viên nhóm Đối thoại giáo dục, yếu tố bao gồm:

1 Sự khác giáo dục DAA thực thành công Mỹ trường đại học Mỹ tuyển sinh dựa nhiều yếu tố: điểm số, hoạt động giờ, luận, dẫn tới độ xác việc chọn trường cho sinh viên tốt Việt Nam xét tuyển sinh viên dựa điểm thi, nên khách quan giảm nhiều

2 Giảm chất lượng sinh viên giỏi trường đại học nhỏ trường đại học thành lập, qua vơ tình giảm hội phát triển họ

Tài liệu

[1] D Gale and L S Shapley,College admissions and the stability of marriage.

https://www.maa.org/sites/default/files/pdf/pubs/monthly_ jan1962-StabilityofMarriage.pdf

[2] Alvin E Roth, Lloyd S Shapley,Stable matching: Theory, evidence, and practical design,

https://mk0nrmp3oyqui6wqfm.kinstacdn.com/wp-content/uploads/ 2013/08/popular-economicsciences2012.pdf

[3] Blog Toán học cho người,Vài nét tốn nhân bền vững,

https://blogm4e.wordpress.com/2016/09/09/ vai-net-ve-bai-toan-hon-nhan-ben-vung/

[4] Wikipedia,Bài tốn nhân bền vững,

https://vi.wikipedia.org/wiki/B%C3%A0i_to%C3%A1n_h%C3%B4n_nh% C3%A2n_b%E1%BB%81n_v%E1%BB%AFng

[5] Vũ Thị Nhinh,Bài toán hôn nhân bền vững ứng dụng thực tế,

https://katatunix.wordpress.com/2008/09/14/ bai-toan-hon-nhan-b%E1%BB%81n-v%E1%BB%AFng/

[6] Trần Vinh Dự,Khi kinh tế học nghiên cứu cách "ghép đôi",

tuoitre.vn/khi-kinh-te-hoc-nghien-cuu-ve-cach-ghep-doi-516607

[7] Thetalog,Deferred-Acceptance Algorithm tốn nhân bền vững

https://thetalog.com/algorithm/deferred-acceptance/

[8] Wikipedia,Hall’s marriage theorem,

(76)

P

HƯƠNG

P

HÁP

G

IÁN

T

IẾP

G

IẢI

C

ÁC

B

ÀI

T

OÁN

Đ

ỊNH

L

ƯỢNG

Trần Nam Dũng

Thành phố Hồ Chí Minh

There are direct paths to a successful goal. But there are plenty of indirect paths, too.

Định lý Vi-et cho phương trình bậc hai nói riêng cho phương trình đa thức bậc n nói chung cho ví dụ việc tính tốn biểu thức liên quan đến nghiệm mà không cần biết đến giá trị nghiệm Trong viết này, mở rộng ý tưởng để nghiên cứu tính chất đối tượng mà không cần biết công thức tường minh đối tượng

Chúng ta toán lấy từ đề thi tuyển sinh trường Đại học Độc lập Moscow (IUM), năm1998:

Bài tốn Tính tích phân từ0đến2của hàm sốg.x/;biếtg.a/là nghiệm phương trình x5CxDa:

Lời giải Ta biết làg.a/xác định cách theo a, ví dụ g.0/ D 0; g.2/ D 1: Thế

nhưng ta khơng có cơng thức chog.x/:Khơng có cơng thứcg.x/thì tính

2

Z

0

g.x/dx ‹

Nếu bó tư vào cơng thức tínhg.x/thì chắn ta khơng giải Ta phải

khai thác định nghĩa củag.x/, theo định nghĩa,g.x/chính hàm ngược hàm f x/Dx5Cx:

Hai hàm ngược có tính chất gì? Tính chất nếuf x/Dythìg.y/Dx, tức

(77)

Từ dễ dàng suy

2

Z

0

g.y/dyC

1

Z

0

f x/dxDS.OCAB/D2:

Nhưng

1

Z

0

f x/dx D

1

Z

0

.x5Cx/dxD x

6

6 C x2

2

ˇˇ ˇˇ10 D 23:

Suy

2

Z

0

g.x/dx D

2

Z

0

g.y/dy D2 D

4 3:

Bài tập Cho hàm sốf x/liên tục trênRvà thỏa mãnf3.x/C2f x/ D1 xvới mọiR:

Tính tích phân

1

Z

2

f x/dx:

Như vậy, toán1;mặc dù ta khơng có cơng thức cho hàm g.x/;nhưng nhờ vào mối quan

hệ với hàm biết f x/, ta thực hành tính tốn g.x/ Áp dụng ý

(78)

Bài toán (VMO 2002) Cho n số nguyên dương Chứng minh phương trình

x C

1

4x 1C C

1 n2x 1 D

1

2: (1)

chỉ có nghiệm thuộc.1;C1/;ký hiệu làxn:Chứng minh khindần đến vô cùng,xndần đến4:

Lời giải Đặt

fn.x/D

1

x 1C

1

4x C C

1 n2x 1

1 2;

thìf hàm liên tục trên.1;C1/và có đạo hàm âm Hơn

lim

x!1Cfn.x/D C1;x!C1lim fn.x/D

1 2;

nên phương trìnhfn.x/D0có nghiệm nhấtxn:

Ta lưu ý

fnC1.x/Dfn.x/C

1

.nC1/2x 1:

Cho nên

fnC1.xn/Dfn.xn/C

1

.nC1/2xn

> 0:

Từ suy raxnC1 > xn:Tiếp theo, ta để ý

fn.4/D

1

4 1C

1

44 C C 4n2 1

1

D 12 1 C12

C C 12

1

2n

1 2nC1

1

D

4nC2:

Suy raxn< Như dãy số.xn/tăng bị chặn 4, suy ra.xn/có giới hạn hữu hạn

Để chứng minh lim

n!1xnD4, ta dùng định lý Lagrange để đánh giá khoảng cách giữaxnvà

Cụ thể, áp dụng định lý Lagrange, ta có

1

4nC2 D jfn.xn/ fn.4/j D jxn 4jjf

0

n.c/j;

trong đóc nằm giữaxnvà4:Nhưng

jfn0.c/j D

1 c 1/2 C

4

.4c 1/2 C C

n2

.n2c 1/2 >

1 9:

Suy

1 4nC2 >

1

9jxn 4j:

(79)

Bài tập (IMU 1999) Gọixnlà nghiệm phương trình

1 x C

1

x C C

x n D0:

nằm khoảng.0; 1/:

a) Chứng minh dãyfxnghội tụ

b) Tìm giới hạn củaxnkhindần đến vơ

Bài tập (VMO 2007) Cho số thựca > 2và

fn.x/Da10xnC10CxnC CxC1:

a) Chứng minh với số ngun dươngn; phương trìnhfn.x/ D achỉ có

một nghiệm dương

b) Ký hiệu nghiệm dương làxn;chứng minh dãy sốfxngcó giới hạn hữu

hạn khindần đến vô

Bài tập (Ninh Bình 2013) Cho phương trình (ẩnx;tham sốnnguyên dương) xC2x2C Cnxn

4 D0:

a) Chứng minh với số ngun dươngnphương trình có1nghiệm dương nhất,

kí hiệu nghiệm làxn:

b) Chứng minh lim

n!1xnD 3:

Để viết phương trình đường thẳng qua điểm hai phương trình đường tròn qua 3điểm,

theo phương pháp trực tiếp ta phải tìm tọa độ điểm đó, từ cách cách khác thực thi nhiệm vụ Cách làm này, mặt dẫn đến tính tốn phức tạp (đặc biệt trường hợp đường trịn), mặt bất khả thi (nếu phương trình để tìm tọa độ điểm có bậc cao khơng giải nghiệm đẹp) Trong tình đó, ta lại dùng định nghĩa điểm (thường điểm định nghĩa cách quỹ tích tương giao) để phương trình đường thẳng hay đường trịn chứa điểm cho, từ tìm lời giải toán

(80)

Bài toán Cho ellip.E/W x9 Cy2D1;và parabol.P /Wy Dx2 2x:

a) Chứng minh rằng.E/.P /cắt tại4điểm phân biệtA; B; C; D:

b) Chứng minh rằngA; B; C; Dcùng nằm đường trịn Tìm tâm bán kính đường

trịn đó.

Lời giải a) Phương trình hồnh độ giao điểm của.E/và.P /có dạng x2

9 C.x

2

2x/2 1D0:

Đặt

f x/D x

2

9 C.x

2

2x/2 1;

thì ta có

f 1/D8C

9 > 0; f 0/D < 0; f 1/D

9 > 0; f 2/D

9 < 0; f 3/D9 > 0:

Do phương trình hồnh độ giao điểm có nghiệm phân biệt

1 < x1 < < x2 < < x3 < < x4 < 3:

Suy cắt tại4điểm phân biệtA; B; C; D:

b) Để chứng minh điểmA; B; C; D nằm đường tròn, làm theo phương

pháp trực tiếp, ta phải tìm tọa độ4điểmA; B; C; Drồi viết phương trình đường trịn

quaA; B; C cuối chứng minhD nằm đường tròn vừa viết Để tìm tọa độ

điểmA; B; C; Dta phải giải phương trình hồnh độ giao điểm Đó phương trình bậc4

mà nói chung ta giải Thực trường hợp lại điều may mắn, biết (mà có cách giải thật), ta bị sa đà vào hướng vô mờ mịt với tính tốn khủng khiếp (kiểu tính

1

p

5

0

@ 1Cp5

2

!10

1 p5

!101

A;

bằng phương pháp khai triển theo nhị thức Newton)

Ta không theo hướng mà dựa vào định nghĩa hình học củaA; B; C; Dđể tìm phương

(81)

Cụ thể, ta thấy tọa độ củaAthỏa mãn hệ phương trình ˆ < ˆ :

xA2 Cy

2

A 1D0

yA xA2 C2xAD0

Nếu ta nhân phương trình thứ hai với

9 cộng với phương trình đầu ta

xA2 Cy

2 A

8 9yAC

8 9x

2 A

16

9 xA D0:

Hay xA C yA

D 16181 :

Các đẳng thức tương tự, hiển nhiên với B; C; D: Như bốn điểm A; B; C; D

cùng nằm đường trịn có phương trình

x C y

D 16181 :

Tức đường trịn tâm điểmI 89I49bán kính p

161 :

Bài tốn Tìm tất giá trị tham sốmđể đồ thị hàm sốy Dx3C2x2Cmx 3 cắt paraboly Dx2Cx 1tại ba điểmA; B; C phân biệt cho bán kính đường trịn ngoại

tiếp tam giácABC bằng p3

2:

(82)

mà tìm phương trình đường trịn chứa 3giao điểm Đầu tiên ta thấy xA; xB, xC nghiệm

của phương trình hồnh độ giao điểm

x3Cx2C.m 1/x 2D0:

Ta cần mối quan hệ x y khơng cịn chứax3:Làm nào? Ý tưởng ta lấy

y2 D.x2Cx 1/2và chia chox3Cx2C.m 1/x 2sẽ phần dư có dạngax2CbxCc:

Như

y2D.x3Cx2C.m 1/x 2/Q.x/Cax2CbxCc:

Thay tọa độ điểmAvào

yA2 DaxA2 CbxACc:

Cuối dùng phương trìnhyA DxA2CxA 1để điều chỉnh cho hệ số củax2A; yA2

Đi vào chi tiết

.x2Cx 1/2 D.x3Cx2C.m 1/x 2/.xC1/ mC1/x2 m 1/x C3:

Từ suy

yA2 D mC1/x

A m 1/xAC3:

Nhân đẳng thứcyADxA2 CxA 1vớimrồi cộng với đẳng thức vế theo vế

yA2 CmyA D xA2 CxAC3 m;

Suy

yAC

m

2

C

xAC

1

2

D m

2 4mC13

4 :

Đây phương trình đường trịn qua A; B; C; hiển nhiên, tọa độB; C thỏa

mãn phương trình

Để đường trịn có bán kính p3

2;thì ta cần có

m2 4mC13

4 D

9 ,m

2

4m 5D0:

Giải ta m D m D 1: Với m D phương trình hồnh độ giao điểm x3Cx2C4x 2D0chỉ có nghiệm thực, loại, vớimD 1phương trình hồnh độ giao

điểm

x3Cx2 2x 2D0,.xC1/.x2 2/;

(83)

Bài tập Cho hai hàm sốy Dx2 2xvày Dx3 x2 mC4/x Cm 1;vớimlà tham

số Có giá trị củam để đồ thị hai hàm số cho cắt ba điểm phân biệt

và ba điểm nằm đường trịn bán kính bằngp5:

Bài toán Chứng minh với số thực m, đồ thị hàm sốy Df x/D xx2CCm1;có ba điểm

uốn thẳng hàng.

Lời giải Tính đạo hàm bậc hai củaf x/, ta có (bỏ qua phép chứng minh phương trìnhx3 C 3mx2 3x mD0ln có ba nghiệm phân biệt)

f00.x/D 2.x

3C3mx2 3x m/

.x2C1/2 :

Ta chứng minh ba điểm uốn thẳng cách phương trình đường thẳng chứa ba điểm uốn Gọi ba điểm uốn làA; B; C:Xét điểmAcó tọa độ.xA; yA/, với

xA3 C3mxA2 3xA mD0;

yAD

xACm

x2 AC1

:

Với˛ số đó, ta có

xAC˛yA DxAC

˛.xACm/

x2AC1 D

xA3 CxAC˛xAC˛m

xA2 C1 D

3mx2AC.˛C4/xACmC˛m

xA2 C1 :

Ta muốn vế phải số nên chọn˛ D 4và đượcxA 4yAC3m D0:Đây

phương trình đường thẳng chứaA; B; C:Nghĩa làA; B; C:thẳng hàng Bài tập Cho hàm sốf x/D x3 3xx2CC13xC5;xác định trênRn f 1g:

a) Chứng minh hàm sốy Df x/có3điểm cực trị khơng thẳng hàng

b) Gọi A; B; C ba điểm cực trị Tính diện tích tam giác ABC tính bán kính đường

trịn ngoại tiếp tam giácABC:

Trong hình học giải tích phương pháp phân đơi tọa độ cho cách giải toán tiếp tuyến, tiếp diện nhanh chóng tiện lợi, tránh tính tốn trực tiếp

Bài tốn (Định lý cực – đối cực ellip) Cho ellip.E/W xa22C

y2

b2 D1:Một đường thẳng

(84)

Lời giải Giả sử.xi; yi/là tọa độ Ti với i D 1; 2:Phương trình tiếp tuyến của.E/tại T1

có dạng

x1x

a2 C

y1y

b2 D1:

Tiếp tuyến quaA.xA; yA/nên ta có

x1xA

a2 C

y1yA

b2 D1:

Hoàn toàn tương tự ta có

x2xA

a2 C

y2yA

b2 D1:

Nhưng điều có nghĩa gì?

Điều có nghĩa điểmT1; T2đều nằm đường thẳng

xAx

a2 C

yAy

b2 D1Š

Tức

xAx

a2 C

yAy

b2 D1;

chính phương trình đường thẳng quaT1; T2! Thật ảo thuật! Ta viết phương

trình đường thẳngT1T2mà khơng cần tìm tọa độ củaT1; T2:

Và nữa, đường thẳngd có phương trìnhAxCByCC D0thì viết lạid dạng ˛xCˇy D1;ta thấy tất đường thẳngT1T2sẽ qua điểm.˛a2; ˇb2/:

Điểm P ˛a2; ˇb2/được gọi cực đường thẳng d có phương trình ˛x Cˇy D 1: Còn

đường thẳngd gọi đối cực điểmP: Bài tập Cho ellip.E/ W xa22 C

y2

b2 D 1:Tìm quỹ tích điểmM mặt phẳng ellip

mà từ kẻ hai tiếp tuyến vng góc đến.E/:

Một cách tổng quát, phương pháp phân đôi tọa độ cho công thức sau để viết phương trình tiếp tuyến đường cong bậc hai Xét đường cong

.C /Wax2C2bxy Ccy2C2dx C2eyCf D0:

Giả sửA.xA; yA/là điểm thuộc.C / Khi phương trình tiếp tuyến của.C /tạiAcó dạng

axAxCb.xAyCyAx/CcyAyCd.xCxA/Ce.yCyA/Cf D0:

(85)

Từ phương trình tiếp tuyến có dạng

.axACbyACd /.x xA/C.bxACcyACe/.y yA/D0:

Từ, đây, với ý

axA2 C2bxAyACcyA2 C2d xAC2eyACf D0;

ta có công thức

Phương pháp phân đôi tọa độ tiếp tục không gian để giúp viết phương trình tiếp diện đường cong bậc hai điểm thuộc đường cong

Bài toán Trong không gian với hệ tọa độOxyz, cho đường thẳng d W x 131 D yC11 D z4

mặt cầu.S /W x2Cy2Cz2 2x 4y 6z 67D0:Quad dựng tiếp diện với.S /tiếp

xúc với.S /lần lượt tạiT1; T2 Viết phương trình đường thẳngT1T2

Lời giải Giả sửT1có tọa độ là.x1; y1; z1/ Phương trình tiếp diện của.S /tạiT1có dạng

x1xCy1yCz1z x x1 2y 2y1 3z 3z1 67D0:

Vì tiếp diện quad nên ta có

x1.13 t /Cy1 1Ct /Cz14t 13 t / x1 1Ct / 2y1 34t 3z1 67D0; 8t;

hay

12x1 3y1 3z1 78Ct x1Cy1C4z1 13/D0; 8t

Từ suy raT1 D.x1; y1; z1/nằm đường thẳng cho phương trình

8 < :

12x 3y 3z 78D0

xCyC4z 13D0

Tương tự,T2cũng nằm đường thẳng Từ đó, phương trình đường thẳngT1T2là (sau

rút gọn) 8

< :

4x y z 26D0

xCyC4z 13D0

Chuyển dạng tắc x D

y D

z :

Bài tập Trong không gianOxyz cho đường thẳngd W x 31 D y1 D z 31 mặt cầu S / W x2Cy2Cz2 D 4:Hai mặt phẳng phân biệt qua d;tiếp xúc S /tạiA; B:Viết phương trình

(86)

Bài tập Cho hai hàm sốy Df x/vày Df f x//có đồ thị là.C /và.C0/:Đường

thẳngx D2cắt.C /và.C0/lần lượt tạiM vàN:Biết phương trình tiếp tuyến của.C /tại điểm M lày D2x 2:Viết phương trình tiếp tuyến của.C0/tại điểmN:

Bài tập 10 Cho số thựcathỏa điều kiện.a 1/3 D9a;đặtb Da2Ca:Chứng minh bC1/3 D27b2:

(87)

MỘT SỐ BÀI TOÁN

DÙNG TÍNH KHƠNG BỊ CHẶN CỦA ĐA THỨC

Đào Xn Luyện

(Trường THPT chun Lê Q Đơn ,Bình Định)

TĨM TẮT

Các tốn đa thức nói chung nói riêng tốn đa thức sử dụng tính khơng bị chặn đa thức có vị trí quan trọng nhiều phân mơn tốn học giải tích, số học, biểu diễn xấp xỉ, Các toán liên quan đến đa thức thường xuất đề thi học sinh giỏi cấp, thi olympic thi IMO Các tài liệu viết đa thức tương đối nhiều, tác giả đề cập việc sử dụng tính khơng bị chặn đa thức (khác hằng) số dạng toán

1 Một số mệnh đề bản

Các đa thức nói tính khơng bị chặn viết khơng nói ta hiểu đa thức có bậc dương

Ngoài phần lý thuyết đa thức định nghĩa, khái niệm bậc, nghiệm, định lý Viet, định lý Berzout, định lý đại số, tính liên tục, tính khả vi , bổ sung thêm số kết sau

Mệnh đề Cho số nguyên dươngnvà đa thức hệ số thực

f(x) = anxn+an−1xn−1+ .+a1x+a0, an 6=

1 Khinlẻ

a) Nếuan>0thì lim

x→−∞f(x) = −∞,x→lim+∞f(x) = +∞

b) Nếuan<0thì lim

x→−∞f(x) = +∞,x→lim+∞f(x) = −∞

2 Khinchẵn

a) Nếuan>0thì lim

x→−∞f(x) = limx→+∞f(x) = +∞

a) Nếuan<0thì lim

(88)

Hệ quảCho số nguyên dươngnvà đa thức hệ số thực

f(x) = anxn+an−1xn−1+ .+a1x+a0, an 6=

1 Khinlẻ

a) Nếuan >0thì tồn tạiα >0đểf(α)>0và tồn tạiβ <0đểf(β)<0

b) Nếuan <0thì tồn tạiα >0đểf(α)<0và tồn tạiβ <0đểf(β)>0

2 Khinchẵn

a) Nếuan >0thì tồn tạiα >0đểf(α)>0và tồn tạiβ <0đểf(β)>0

a) Nếuan <0thì tồn tạiα >0đểf(α)<0và tồn tạiβ <0đểf(β)<0

Mệnh đề Cho số nguyên dươngm, nvà đa thức hệ số thực f(x) = anxn+an−1xn−1+ .+a1x+a0, an 6=

g(x) = bmxm+bm−1xm−1+ .+b1x+b0, bm 6=

1 Khin > m

a) lim x→+∞

f(x)

g(x) = +∞nếuan·bm >0

b) lim x→+∞

f(x)

g(x) =−∞nếuan·bm<0

2 Khin=mthì lim x→−∞

f(x)

g(x) = limx→+∞ f(x)

g(x) = an bn

3 Khin < mthì lim x→−∞

f(x)

g(x) = limx→+∞ f(x)

g(x) =

Mệnh đề Nếu hàm số đa thức hệ số thựcy=f(x)là hàm số tuần hồn thìf(x)là số.

Chứng minh mệnh đề. Rõ ràng nếuf(x) =C vớiC số thìf(x)là hàm số tuần hồn

Giả sửf(x)có bậc dương tuần hồn GọiT > 0là chu kỳ củaf(x) Xét đoạn[0;T],

do đa thứcf(x)liên tục nên nhận gíá trị nhỏ nhấtmvà giá trị lớn nhấtM đoạn[0;T]

Mặt khác,f(x)có bậc dương tuần hồn chu kỳT nên nhận giá trị đoạn[m;M]với

(89)

2 Sử dụng tính khơng bị chặn đa thức các

bài toán liên quan đến số học đa thức nhận giá trị

nguyên

Các toán đa thức liên quan đến số học thường khó Chúng ta xét sau số toán liên quan đến vấn đề

Bài toán Cho số nguyên không âmm, nthỏam+n >0và hai đa thức

f(x) = xn+a1xn−1+ .+an,

g(x) =xm+b1xm−1+ .+bm,

sao cho với số thực x, ta có f(x) là bình phương số nguyên khi g(x)

là bình phương số nguyên Chứng minh tồn đa thức Φ(x)hệ số thực cho

f(x)·g(x) = Φ2(x).

Lời giải. Nếun >0, m = 0thìg(x) =a2 vàf(x)là số phương∀x ∈R, điều vơ lý

vìf(x)liên tục trênR

Nếum >0, n >0, tồn khoảng[N,+∞)để cácf, gtăng thực Gọia2, b2là số

chính phương lớn hay bằngf(N), g(N) Vìf liên tục nên tồn dãy N ≤x0 < x1 < < xn,

sao cho f(xn) = (a+n)2, ∀n ≥ Ta chứng minhg(xn) = (b+n)2, ∀n ≥ bằng

quy nạp

Với n = 0, khẳng định Giả sử khẳng định vớin ≥ 0, ta chứng minhg(xn+1) = (b+n+ 1)2 Giả sử trái lại g(xn+1) = (b+m)2, m 6= n + 1, tồn x0 sao cho g(x0) =

(b+n+ 1)2 Ta có

g(xn)< g(x0)< g(xn+1)⇒xn< x0 < xn+1,

suy

f(xn)< f(x0)< f(xn+1)⇒(a+n)2 < f(x0)<(a+n+ 1)2,

điều vơ lý vìf(x0)là số phương Vậyg(xn+1) = (b+n+ 1)2, phép quy nạp hoàn tất.

Từ đó, ta có

p

f(xn)−pg(xn) =a−b ⇒f(xn) +g(xn)−2·pf(xn)·pg(xn) = (a−b)2,

suy

f(xn)·g(xn) =

"

f(xn) +g(xn)−(a−b)2

#2

Như vậy, đặt

Φ(x) =

"

f(xn) +g(xn)−(a−b)2

#

(90)

Nhận xét Để giải toán cần nhiều kiến thức liên quan tính liên tục,

tính khơng bị chặn đa thức, lý thuyết dãy số, để xây dựng dãy giá trị hàm Đây bài tốn khó hay.

Bài toán (USAMO)Chứng minh đa thức monic (đa thức có hệ số cao bằng

1) bậcnhệ số thực trung bình cộng hai đa thức monic bậcnnnghiệm thực.

Lời giải. Gọif(x)là đa thức monic bậcnhệ số thực

Nếun = 1thìf(x) =x+a, vớia ∈ R Đặt P(x) = x, Q(x) =x+ 2athìP(x), Q(x)có1

nghiệm thực vàf(x) = P(x) +Q(x)

Nếun >1, xét

g(x) = (x−2)(x−4) .(x−2(n−1))

có bậcn−1 Đặt P(x) =xn−kg(x)và Q(x) = 2f(x)−xn+kg(x)thìP(x), Q(x)là các

đa thức monic bậc n hệ số thực f(x) = P(x) +Q(x)

2 Ta chứng minh vớik đủ lớn

P(x), Q(x)cónnghiệm thực

Xét giá trị củag(x)tạinđiểm1,3, ,2n−1, giá trị đan dấu có giá trị nhỏ

nhất bằng1(vì nhiều hai thừa số có giá trị tuyệt đối bằng1và thừa số cịn lại

có giá trị tuyệt đối lớn hơn2)

Mặt khác, tồn sốc >0sao cho với0≤x≤2n, ta có

|x|n< c, |2f(x)−xn|< c

Lấyk > cthìP(x), Q(x)có giá trị đan dấu tạinđiểmx= 1,3,5, ,2n−1, nênP(x),Q(x)

đều có nhấtn−1nghiệm thực Mà chúng đa thức bậcnnên đa thức có đủn

nghiệm thực Bài toán giải

Nhận xét Đây toán biểu diễn đa thức monic bậc nbất kỳ qua hai đa thức cùng

bậc có đủnnghiệm thực Việc xây dựng hai đa thứcP(x), Q(x)rất hay kỹ thuật có thể

nghĩ đến gặp toán biểu diễn đa thức.

Bài toán Cho f(x) là đa thức bậc không bé hơn 2với hệ số hữu tỷ dãy dãy số hữu tỷ

a1, a2, thỏa mãn f(an+1) = an, với số nguyên dương n Chứng minh tồn số

nguyên dươngN để với số nguyên dươngnthìN anlà số nguyên.

Lời giải. Trước tiên, ta chứng minh dãy(an)bị chặn

Dodegf >1nên lim x→∞

|f(x)|

|x| = +∞,

∃M ≥ |a1|:|f(x)| ≥ |x|,∀ |x| ≥M

Ta chứng minh|an| ≤M,∀n≥1 Thật vậy, giả sử∃an: |an|> M,

(91)

trái với cách chọnM Như vậy(an)bị chặn

Dof(x)có hệ số hữu tỉ nên ta viếtf(x)dưới dạng

f(x) = bmx

m+ .+b

c ,

trong đób0, b1, , bm, clà số nguyên Giả sửa1 =

p

q, vớip, qnguyên

Ta đặt N = qbm, đóN a1 = pbm nguyên Giả sử N an nguyên, ta chứng minh N an+1

nguyên Xét đa thức

P(x) = cN m

bm ·

fx N

−an

,

thìP(x)là đa thức monic hệ số nguyên Màf(an+1) =annênP(N an+1) = Như vậyN an+1

là nghiệm hữu tỉ củaP(x), nghiệm nguyên Bài toán giải

Bài toán Cho đa thức

P(x) = x2017−18x2016+ 11x2015+ 6x+

Vớia0 là số nguyên khác0cho trước, xác định dãy(an)như sauan+1 =P(an) Chứng

minh với cặp số nguyên dươngm, nphân biệt thìam, annguyên tố nhau.

Lời giải. Dễ thấy(an)là dãy số nguyên khác0vìP(x)khơng có nghiệm ngun

DoP(0) =P(1) = 1nênP(x) =x(x−1)Q(x) + Ta chứng minh quy nạp

an≡1 (mod a0)a1 an−1

◦Vớin= 1, ta có

a1 =P(a0) = a0(a0−1)Q(a0) + 1≡1 (mod a0)

◦Giả sử vớin, tức làan ≡1 (mod a0)a1 an−1, hayan = ka0a1 an−1 + 1, k ∈Z

Khi

an+1 =P(an) = an(an−1)Q(an) + =ka0a1 an−1anQ(an) + 1,

suy raan ≡1 (mod a0a1 an−1an)

Không tính tổng quát, giả sửm < n đặtd = (am, an) Khi đó, tícha0a1 an−1 chứa

thừa sốamnên chia hết chod, suy raan−1chia hết chod Nói cách khác,an, an−1đều chia

hết chodnênd= Vậy(am, an) = 1, ∀m6=n

Bài toán Cho đa thức

P(x) =x2018 +a20x20+a11x11+a10x10+ .+a1x+ 2017,

trong hệ sốaklà số nguyên có giá trị tuyệt đối không lớn hơn100 Chứng minh rằng

(92)

Lời giải. Theo định lý đại số,P(x)có đủ 2018 nghiệm (thực hay phức) Gọiz

một nghiệm củaP(x), ta chứng minh|z|>1

Thật vậy, giả sử|z| ≤1 Ta có

P(z) =z2018+a20z20+a11z11+a10z10+ .+a1z+ 2017 = 0,

suy

2017 =z2018+a20z20+a11z11+a10z10+ .+a1z

≤z2018+a20z20

+a11z11

+ .+|a1z|

≤1 +|a20|+|a11|+ .+|a1| ≤1 + 12·100 = 1201,

vô lý, đó|z|>1

Giả sửP(x) =g(x)·h(x), đóg(x), h(x)là đa thức hệ số ngun có bậc dương

|P(0)|=|g(0)| · |h(0)|= 2017

Mà2017là số nguyên tố nên|g(0)|= 1hoặc|h(0)|=

Khơng tính tổng qt, giả sử |g(0)| = 1, với ≤ degg = m ≤ 2017 hệ số cao

củag(x), h(x)đều bằng1 Gọiz1, z2, , zm nghiệm phức củag(x)thì chúng

nghiệm phức củaP(x), đó|zk|>1,∀k= 1,2, , m Nhưng theo định lý Viet, ta lại có

|z1z2 zm|=|g(0)|= 1,

vơ lý VậyP(x)thành tích hai đa thức hệ số nguyên có bậc dương

Bài tốn Ký hiệupklà số nguyên tố thứk Chứng minh với số nguyên dươngnthì

phương trình

2018x4+ 108n3 = 15nx3+ 4n(pn+1+pn+2)x2 + 18n2(pn+2+pn+3−3n)x+ 1925n4

có nghiệm khơng bé hơnn

Lời giải. Trước tiến, quy nạp, ta chứng minhpn+1+pn+2 ≥6n

Với n = 1,2, kiểm tra thấy Giả sử vớin = k, tức làpk+1 +pk+2 ≥ 6k Ta chứng

minh vớin=k+ 1, haypk+2+pk+3 ≥6k+

Theo giả thiết quy nạp, ta cần chứng minhpk+2+pk+3≥pk+1+pk+2+ 6, haypk+3−pk+1 ≥6

Thật vậy, giả sửpk+3−pk+1 <6 Vìpk+3, pk+1là số lẻ nênpk+3 =pk+1+4, haypk+1, pk+2, pk+3

là ba số lẻ liên tiếp Do đó, ba số phải có số chia hết cho3, vơ lý vìpk+1 >3

Ta đặt

f(x) = 2018x4−15nx3−4n(pn+1+pn+2)x2−18n2(pn+2+pn+3−3n)x−1925n4+ 108n3

Vìf(x)là đa thức bậc bốn có hệ số cao dương nên lim

x→+∞f(x) = +∞ Do đó, tồn tạiα > n

đểf(α)>0 Mặt khác, ta có

f(n) = 78n3−4n3(pn+1+pn+2)−18n3(pn+2+pn+3−3n) + 108n3 ≤0,

dopn+1 +pn+2 ≥ 6n vàpn+2+pn+3 ≥6n+ Như vậyf(n)f(α) ≤0, tồn tạix0 ≥n

(93)

3 Sử dụng tính khơng bị chặn số dạng toán về

xác định đa thức

Mệnh đề 3, xét lớp hàm đa thức tuần hồn, tốn sau xét xem hợp hàm tuần hoàn với hàm đa thứclà hàm tuần hồn hàm đa thức hàm

Bài tốn Tìm tất đa thứcf(x)hệ số thực cho hàm sốy= sinf(x)là hàm số

tuần hoàn.

Lời giải. Trước tiên, ta chứng minh hàmf tuần hồn, xác định trênR có đạo hàm Rthì hàmf0cũng tuần hồn Thật vậy, giả sửT >0là chu kỳ hàmf, đó

f(x+T) =f(x), ∀x∈R, ⇒ f0(x+T) = f0(x), ∀x∈R,

hayf0cũng hàm tuần hồn.

◦Nếuf(x)≡choặcf(x) =ax+b, vớia, b, c∈R, a6= 0thì dễ thấyy= sinf(x)tuần hồn

◦Nếudegf ≥2, đóy0 =f0(x) cosf(x)tuần hồn nên bị chặn.

Mặt khác, dof(x)liên tục không bị chặn nên tồn dãyxnsao cho|f(xn)|=|nπ|, ∀n≥1

Khi lim

n→+∞|xn|= +∞vàn→lim+∞|f

0(x) cosf(x)|= +∞ Điều trái với tính bị chặny0 Do

đó nếudegf ≥2thìy= sinf(x)khơng tuần hồn

Vậyf(x)≡choặcf(x) = ax+b, vớia, b, c∈R, a6= 0là tất đa thức cần tìm

Bài tốn (Greece 2014)Tìm tất đa thức hệ số thựcP(x)thoả mãn

(x2 −6x+ 8)P(x) = (x2+ 2x)P(x−2), ∀x∈R (∗)

Lời giải. Ta có(∗)tương đương với

(x−2)(x−4)P(x) =x(x+ 2)P(x−2), ∀x∈R (1)

Lần lượt thay x = 0,−2,4 vào (1), ta thu P(0) = P(−2) = P(2) = 0, đặt

P(x) =x(x−2)(x+ 2)Q(x) Thay vào (1), ta

(x−2)Q(x) = xQ(x−2), ∀x∈R\ {0;±2; 4}

MàQ(x)là đa thức nên

(x−2)Q(x) =xQ(x−2), ∀x∈R (2)

Thayx = 0vào (2), ta cóQ(0) = 0, đóQ(x) =xR(x) Tương tự trên, thay vào (2), ta

được

R(x) =R(x−2), ∀x∈R,

hayR(x)tuần hồn Do theo mệnh đề thìR(x)≡c

(94)

Bài tốn (Việt Nam 2003)Tìm tất đa thức hệ số thựcP(x)thỏa

(x3+ 3x2+ 3x+ 2)P(x−1) = (x3−3x2+ 3x−2)P(x), ∀x∈R (∗)

Lời giải. Ta có(∗)tương đương với

(x+ 2)(x2+x+ 1)P(x−1) = (x−2)(x2−x+ 1)P(x), ∀x∈R (1)

Lần lượt thayx =−2; 2;−1; 1vào (1), ta thu đượcP(−2) = P(−1) = P(0) = P(1) = 0,

đó đặtP(x) = (x−1)x(x+ 1)(x+ 2)Q(x) Thay vào (1), ta

(x2+x+ 1)Q(x−1) = (x2−x+ 1)Q(x), ∀x∈R\ {0;±1;±2}

MàQ(x)là đa thức nên

(x2+x+ 1)Q(x−1) = (x2−x+ 1)Q(x), ∀x∈R (2)

Do(x2 +x+ 1, x2 −x+ 1) = 1nên

Q(x) = (x2+x+ 1)R(x), ∀x∈R ⇒ Q(x−1) = (x2−x+ 1)R(x−1)

Từ đó, (2) trở thành

R(x−1) =R(x), ∀x∈R,

hayR(x)tuần hồn Do theo mệnh đề thìR(x)≡c

Như vậyP(x) =c(x−1)x(x+ 1)(x+ 2)(x2+x+ 1), ∀x∈R Thử lại thoả mãn.

4 Sử dụng tính khơng bị chặn đa thức số

dạng toán khác

Bài toán 10 Cho số nguyên dươngn ≥ và đa thức hệ số thựcP(x)bậcn và cón nghiệm

thực đơi phân biệt.

(a) Chứng minh đa thức Q(x) = P(x) +P0(x) cũng cón nghiệm thực đơi phân

biệt.

(b) Gọia1 < a2 < < anlà nghiệm củaP(x) b1 < b2 < < bn là nghiệm

củaQ(x) Chứng minh rằng

1≤i≤n−1{ai+1−ai} ≤1≤mini≤n−1{bi+1−bi}

Lời giải. (a) Khơng tính tổng qt, giả sử hệ số cao củaP(x)bằng1 Vìa1, a2, , an

là nghiệm củaP(x)nên

P(x) = (x−a1)(x−a2) .(x−an)

Do

(95)

Màa1 < a2 < < an nên từ suy dãyQ(a1), Q(a2), , Q(an)là dãy đan dấu Vì

giữaai, ai+1, i= 1, n−1có nghiệm củaQ(x)

Hơn nữa, ta có

◦Nếunchẵn thìQ(a1)<0, lim

x→−∞Q(x) = +∞, đóQ(x)có nghiệm thuộc khoảng

(−∞;a1)

◦Nếunlẻ thìQ(a1)>0, lim

x→−∞Q(x) = −∞, đóQ(x)có nghiệm thuộc khoảng

(−∞;a1)

Như vậy,Q(x)cónnghiệm thực phân biệt

(b) Phản chứng Giả sử tồn tạijsao chobj+1−bj <

1≤i≤n−1{ai+1−ai}

Xét hàm số

f(x) = P

0(x) P(x) =

n

X

i=1

x−ai

, x∈R\ {a1, a2, , an}

Ta có

f0(x) = −

n

X

i=1

(x−ai)2 <0,

nênf nghịch biến khoảng xác định Màbj, bj+1là nghiệm củaQ(x)nên

P0(bj) P(bj)

= P0(bj+1)

P(bj+1)

=−1,

do tồn tạik ∈ {1; 2; .;n}sao chobj < ak< bj+1

Ta có

ai+1−ai > bj+1−bj ⇒bj−ai > bj+1−ai+1, ∀i= 1,2, , n−1

Nhận thấy với mọi1≤i≤k−1, hai vế bất đẳng thức dương; với mọik ≤i≤n−1,

cả hai vế bất đẳng thức âm,

1

bj−ai

<

bj+1−ai+1

, ∀i= 1,2, , n−1

Chú ý

bj−an

<0<

bj+1−a1, ta có

P0(bj)

P(bj) =

n−1

X

i=1

bj −ai

+

bj −an

<

n−1

X

i=1

1

bj+1−ai+1

+

bj+1−a1 = P

0(bj+1) P(bj+1),

mẫu thuẫn Vì điều phản chứng sai, ta thu điều phải chứng minh

Bài toán 11 Cho đa thức hệ số thựcP(x)bậcnvà số thựca≥3 Chứng minh rằng max

0≤i≤n+1|a

i −P(i)| ≥

a−1

n

(96)

Lời giải. Ta chứng minh phương pháp quy nạp Vớin = 0thìP(x)≡c, doa >3nên

max 0≤i≤1|a

i−P(i)|= max{|1−c|, |a−c|} ≥ |1−c|+|a−c|

2 ≥

|a−1| ≥1

Khẳng định toán vớin=

Giả sử khẳng định toán cho đa thức hệ số thực bậcn−1 Xét đa thức hệ số thực

P(x)bậcn,

P(x+ 1)−P(x)

a−1

là đa thức hệ số thực bậcn−1 Do theo giả thiết quy nạp

∃0≤k ≤n:

ak− P(k+ 1)a−−1 P(k)

a−1

n−1

,

tương đương với

|(ak+1−P(k+ 1))−(ak−P(k))| ≥ (a−1) n

2n−1

Suy

max{|ak+1−P(k+ 1)|,|ak−P(k)|} ≥ |a

k+1−P(k+ 1)|+|ak−P(k)|

≥ |(a

k+1−P(k+ 1))−(ak−P(k))|

2 =

a−1

n

Do hiển nhiên

max 0≤i≤n+1|a

i

−P(i)| ≥

a−1

n

Phép quy nạp hồn tât, tốn giải

Bài toán 12 Cho tam thức bậc hệ số thựcf(x) = ax2 +bx+csao chof(x)không âm với

mọi số thựcxkhông âm Chứng minh tồn đa thứcP(x)sao cho đa thứcf(x)P(x)

tất hệ số không âm.

Lời giải. Dof(x)≥0, ∀x≥0nênc=f(0) ≥0vàa >0, suy ra∆f =b2−4ac≤0.

Nếub ≥0thì chọnP(x) = 1, ta thu điều phải chứng minh

Nếub <0thì ta tìm số nguyênn ≥2đểP(x) = (x+ 1)nthỏa yêu cầu tốn Ta có

f(x)P(x) = (ax2+bx+c) n

X

k=0

Cnkxk

=axn+2+ (b+na)xn+1+ n

X

k=2

(97)

Ta cần chọnnsao chob+na > 0, b+nc >0vàaCk−2

n +bCnk−1+cCnk, ∀k≥2 (∗)

Ta xét

h(k) = aCnk−2+bCnk−1+cCnk

= (a−b+c)k2−[a−(n+ 2)b+ (2n+ 3)c]k+ (n+ 1)(n+ 2)c

Vìa, c > 0, b <0nênh(k)là tam thức bậc có hệ số cao nhấta−b+c > Do để

h(k)>0với mọik,ta cần chọnnsao cho∆h <0

Chú ý rằng∆h <0là tam thức bậc theon, có hệ số cao

(b−2c)2−4c(a−b+c) = b2−4ac≤0

Do lim

n→+∞∆h = −∞ Như vậy, vớin đủ lớn thìn > max

−b a ;

−b c

và ∆h <0, thỏa

điều kiện của(∗) Khi đóP(x)thỏa u cầu tốn

Bài toán 13 Cho số nguyên dươngn ≥3và đa thức

P(x) = anxn+an−1xn−1+ .+a1x+a0

thỏa mãnP(x)dương với số thựcxdương Chứng minh tồn số tự nhiêns

để đa thứcQ(x) = P(x)(x+ 1)scó hệ số không âm.

Lời giải. Ta xét hai trường hợp

◦Trường hợp : Khin = 2mthì P(x) =

m

Y

k=1

(akx2+bkx+ck),

trong đóakx2+bkx+ck>0, ∀x >0 Theo tốn 12, với tam thứcakx2+bkx+ckđều

tồn số tự nhiênsksao cho đa thức

Qk(x) = (akx2+bkx+ck)(x+ 1)sk

có hệ số khơng âm Từ

Q(x) = m

Y

k=1

Qk(x) = m

Y

k=1

(akx2+bkx+ck)(x+ 1)sk =P(x)(x+ 1)s1+ +sm,

cũng có hệ số khơng âm

◦Trường hợp : Khin = 2m+ 1thìP(x)có nghiệm khơng dương là−a Do P(x) = (x+a)R(x), vớiR(x)>0, ∀x >0

DodegR = 2mnên theo trường hợp tồn số tự nhiênssao choR(x)(x+ 1)scó hệ số

đều khơng âm Do

Q(x) = P(x)(x+ 1)s = (x+a)R(x)(x+ 1)s,

cũng có hệ số khơng âm

(98)

Bài tốn 14 Cho đa thức hệ số nguyênf(x) = x4+ax3+bx2+cx+d Biết hệ số nguyên

akhông chia hết cho4 Xác định lim n→∞

n

4

p

|f(n)|o

Lời giải. Đặta= 4k+r, vớir ∈ {1; 2; 3} Ta có

f(n) = n4+an3+bn2+cn+d= (n+k)4+ (a−4k)n3+ (b−6k2)n2+ (c−4k3)n+ (d−k4) = (n+k)4+rn3+ (b−6k2)n2+ (c−4k3)n+ (d−k4)

Vớinđủ lớn

rn3+ (b−6k2)n2 + (c−4k3)n+d−k4 >0,

nênf(n)>(n+k)4, đó|f(n)|=f(n)

Tương tự trên, ta có

f(n) =n4+an3+bn2+cn+d= (n+k+ 1)4+ [a−4(k+ 1)]n3+

= (n+k+ 1)3+ (r−4)n3+

Vớinđủ lớn

(r−4)n3+ < 0,

nênf(n)<(n+k+ 1)4

Do vớinđủ lớn

(n+k)4 < f(n)<(n+k+ 1)4,

suy rajp4 f(n)k=n+kvànp4 |f(n)|o=np4 f(n)o=p4 f(n)−(n+k) Từ

lim n→∞

n

4

p

f(n)o= lim n→∞[

4

p

f(n)−(n+k)] = lim

n→∞

f(n)−(n+k)4

h

4

p

f(n) + (n+k)i hpf(n) + (n+k)2i

= lim n→∞

(a−4k)n3+ .

n3(1 + 1)(0 + 1)

= r = na o

Tài liệu

[1] Hà Huy Khoái,Số học, NXB Giáo dục

[2] Nguyễn Văn Mậu,Đa thức đại số phân thức hữu tỉ, NXB Giáo dục

[3] Trần Nam Dũng (chủ biên),Các phương pháp giải toán qua kỳ thi Olympic - 2014 [4] Tạp chí Tốn học tuổi trẻ

[5] Tạp chí Pi

(99)

MỘT SỐ BÀI TOÁN CHỌN LỌC VỀ

PHƯƠNG TRÌNH HÀM

Lê Phúc Lữ

(Lớp Cao học Khoa học tự nhiên TPHCM)

TÓM TẮT

Các tập trích phần giảng Phương trình hàm tác giả cho đội tuyển HSG, hướng tới kỳ thi VMO năm học 2019-2020 Trong phần Ví dụ tốn Phương trình hàm tập số thực thơng thường, phần Bài tập bổ sung tập số thực dương lớp hàm liên tục để bạn đọc rèn luyện thêm

1 Kiến thức cần nhớ

Đối với phương trình hàm tập số thực, ta cần ý

• Phép thế, đổi biến thích hợp: x → 0, x ↔ ±y, x → f(x), biến để triệt tiêu f(u), f(v)ở hai vế nhằm tìm quan hệ

• Đặta=f(0)để dễ biến đổi “Đơn ánh điểm”, tức làf(a) = 0→a=

• Xét tính đơn ánh / tồn ánh hàm số Nếuf(f(x)) =x+ 2019thì suy điều gì?

• Chú ý nên liệt kê đẳng thức liên hệ thu thơng qua phép để liên kết chúng với tạo điều kiện

• Hàm sốf(x)cộng tính suy đượcf(x) = axnếu thêm điều kiện

f(x)đơn điệu (sử dụng dãy kẹp, chuyển từ hữu tỷ sang vô tỷ)

f(x)≥0,∀x≥0(đưa hàm đồng biến)

f(xn) =fn(x),∀n≥1(sử dụng đa thức để đồng hệ số).

f x1 = f(x)1 ,∀x6= 0(tính hai cách)

(100)

2 Các toán chọn lọc

Bài tốn Tìm tất hàm sốf :R→Rthỏa mãn đồng thời điều kiện:

i) f(xy+f(x)) =xf(y) +f(x)với mọix, y ∈R

ii) Phương trìnhf(x) = 0có khơng q nghiệm.

Lời giải. Thayy = thìf(f(x)) = xf(0) +f(x).Nếuf(0) 6= kéo theof đơn ánh Lại

thayx= 0thìf(f(0)) =f(0)⇒f(0) = 0,vơ lý Do đóf(0) = 0vàf(f(x)) = f(x)

Giả sử tồn tạia6=bsao chof(a) =f(b)6= Suy

af(b) +f(a) =f(ab+f(a)) = f(ab+f(b)) =bf(a) +f(b)

Do đó(a+ 1)f(a) = (b+ 1)f(a)⇒a =bnênf đơn ánh Thayy= 0,ta cóf(f(x)) =f(x)

nênf(x) =x

Bài toán Xét hàm sốf :R→Rthỏa mãn

f(f(x)f(y)) = f(x+y) +f(xy)

với mọix, y ∈R.GọiS là tập hợp số thựcx0 sao chof(x0) = f(−x0)

1 Chứng minh nếuu, v ∈S thìu+v ∈S

2 Chứng minh nếuu, v ∈Rf(u) =f(v)thìu−v ∈S

Lời giải. 1) Ta cóu∈S→ −u∈S.Do

f(u+v) +f(uv) =f(f(u)f(v)) = f(f(−u)f(−v)) =f(−u−v) +f(uv)

Suy raf(u+v) = f(−(u+v))nênu+v ∈S

2) Thay(x, y) = (u,1),(v,1)ta có

f(u+ 1) +f(u) =f(f(u)f(1)) =f(f(v)f(1)) =f(v+ 1) +f(v)

Suy raf(u+ 1) =f(v+ 1) Thay(x, y) = (u+ 1,−1),(v+ 1,−1)ta có

f(f(u+ 1)f(−1)) =f(−u−1) +f(u)vàf(f(v+ 1)f(−1)) =f(−v−1) +f(v)

Suy f(−u−1) = f(−v−1)nênf(−u) = f(−v).Thay (x, y) = (u,−u),(v,−v), ta có

f(u2) =f(v2)nênf(−u2) =f(−v2).Thay(x, y) = (u,−v),(v,−v), ta có

f(f(u)f(−v)) =f(u−v) +f(−uv)vàf(f(v)f(−v)) =f(0) +f(−v2)

Suy raf(u−v) +f(−uv) = f(0) +f(−v2).Tương tự thì

f(v−u) +f(−uv) =f(0) +f(−u2).

(101)

Bài toán Tìm tất hàm sốf, g:R→Rsao chog(1) =−24

f(x−5f(y)) =xf(y)−yf(x) +g(x)

với mọix, y ∈R

Lời giải. Nếuf(x)≡0thì kéo theog(x)≡0,vơ lý nên tồn tạiađểf(a)6=

Giả sử f(y1) = f(y2)thì dễ thấyy1 = y2 nênf đơn ánh Nếuf(0) = 0thì thay y = 0, ta có

f(x) =g(x),thay tiếpx= 0thì cóf(−5f(y)) =g(0) =f(0)nênf(y) = 0,∀y, vơ lý

Do đó,f(0) 6= 0,thayx= 0thì có ngayf tồn ánh Giả sửf(u) = 0thì thayx=u,ta có f(u−5f(y)) =uf(y) +g(u)

Điều chứng tỏf tuyến tính Đặtf(x) =ax+bthì thay vào, ta có f(1−5(ay+b)) =ay+b−y(a+b) +g(1)

⇔a(1−5ay−5b) +b=b−yb−12

(

5a2 =b

a−5ab=−24 ⇔

(

a=

b=

Do đóf(x) = x+ Bài tốn

1 Tìm tất hàm sốf :R→Rsao cho với mọix, y ∈Rthì

f(x2−xy+f(y)) =f2(x)−xf(y) +y

2 Tìm tất hàm sốf :R→Rsao cho với mọix, y ∈Rthì f(xf(x) +f(y)) =f2(x) + 4y

Lời giải. a) Thayx= 0,ta thấyf(f(y)) =f2(0) +ynên dễ thấyf là song ánh trênR.Giả sử

alà số thỏa mãnf(a) = 0.Thayx=y=a,ta cóf(0) =a Suy f(f(0)) = =f2(0)nênf(0) =

Tiếp theo, thayy= 0,ta cóf(x2) = f2(x).Lại thayx=y,ta có

f(f(x)) =f2(x)−xf(x) +xhay =f2(x) = xf(x)

Suy raf(x)(x−f(x)) = 0.Màf(0) = 0vàf song ánh nênf(x)6= 0,∀x6= 0, nên ta có

(102)

2) Thayx= 0,ta cóf(f(y)) =f2(0) + 4ynênf là song ánh Giả sửf(u) = 0thì thayx=u,

ta cóf(f(y)) = 4ynênf(4x) = 4f(x).Do đóf(0) = 0.Thayy= 0,ta cóf(xf(x)) =f2(x).

Thay(x, y)→(f(x),0),ta có

f(4xf(x)) = 16x2 nênf(xf(x)) = 4x2

Do đóf2(x) = 4x2, kéo theof(x) = 2x∨ −2x,∀x ∈ R.Giả sử cóx, y 6= 0sao chof(x) = 2x, f(y) =−2y,thay vào đề bài, ta cóf(2x2−2y) = 4x2+ 4y.Do đó, ta phải có

4x2+ 4y= 2(2x2−2y)∨ −2(2x2−2y)

nênxy= 0,vô lý

Bài tốn Tìm tất hàm sốf :R→Rthỏa mãn

f(x)f(yf(x)−1) = x2f(y)−f(x)

với mọix, y ∈R

Lời giải. Nếu tồn tạiu6= 0đểf(u) = 0thì thayx=u,ta có ngayf(x) = 0,∀x

Nếuf(u) = 0kéo theou= 0thì thayx=y= 1,ta có

f(1)f(f(1)−1) = 0nênf(1) = 1∨f(0) =

Thayx= 1,suy raf(y−1) =f(y)−1.Từ suy

f(x) (f(yf(x)−1) =x2f(y)−f(x)nênf(x)f(yf(x)) =x2f(y)

Thayy= 1thì

f(x)f(f(x)) = x2

Thayx→x+ 1thì(f(x) + 1) (f(f(x)) + 1) = (x+ 1)2nên

f(x) +f(f(x)) = 2x

Giải hệ tổng-tích này, ta cóf(x) = f(f(x)) =x.Thử lại ta thấy thỏa Bài toán Xét hàm sốf :R→Rthỏa mãn

f(x2) +f(xy) =f(f(x)) +yf(x) +xf(y)với mọix, y.

1 Chứng minh rằngf là hàm hằng.

2 Khẳng định không điều kiện đề đổi thành

f(x2) +f(xy) = f(f(x)) +yf(x) +|x|f(y)?

Lời giải. 1) Thayx= 0, ta có2f(0) =f(f(0)) +yf(0); nếuf(0) 6= 0thìy = 2− f(ff(0)(0))

hằng số, vơ lý Do đóf(0) = 0.Thayy= 0,ta có

f(x2) =f(f(x))với mọix (∗)

(103)

• Thayx=y= 1thìf(1) = 2f(1)nênf(1) =

• Thayx=y=−1thìf(1) =−2f(−1)nênf(−1) =

• Thayy=−1thìf(−x) =−f(x)nênf hàm số lẻ

Cuối cùng, trong(∗)thayx→ −xthì

f(f(x)) =f(x2) =f(f(−x)) =f(−f(x)) =−f(f(x))

Suy raf(f(x)) = f(x2) = 0,∀xnênf(x) = 0,∀x > 0; mà f(−x) = −f(x)nên ta có

f(x) = 0,∀x <0.Điều chứng tỏf(x)≡0là hàm

2) Câu trả lời phủ định Thật vậy, xét hàm số:

f(x) =

(

0, ∀x≥0

−x,∀x <0

Khi đó, f(x2) = 0và f(f(x)) = f(0) = 0,∀x ≥ 0và f(f(x)) = f(−x) = 0,∀x < 0nên ta

luôn cóf(f(x)) = 0,∀x, điều kiện ban đầu trở thành

f(xy) = yf(x) +|x|f(y)

Dễ dàng kiểm tra nếux= 0hoặcy = 0thì đẳng thức Ta xét trường hợp sau:

• Nếux >0, y >0thì dễ thấy tất số hạng bằng0,thỏa mãn

• Nếux >0, y <0thì−xy= +x(−y),

• Nếux <0, y <0thì0 =y(−x) + (−x)(−y),

• Nếux <0, y >0thì−xy=y(−x) + (−x)·0,

Do đó, hàm số nêu thỏa mãn điều kiện đề

Bài toán Với số thựcm, n, giả sử tồn hàm sốf :R→Rthỏa mãn

f(x+m·f(xy)) = f(x) +n·xf(y)

với mọix, y ∈R

1 Chứng minh nếuf là hàm toàn ánh vàf(1) = 1thì phải cóm=n.

(104)

Lời giải. 1) Trước hết, ta thấy nếum = 0thìn·xf(y) = 0,∀x, y ∈ R,kéo theon = Cịn

nếun= 0thì

f(x+m·f(x)f(y)) =f(x)với mọix, y ∈R

Nếum 6= 0thì xét x0 chof(x0) 6= 0, chọn yđể f(y) = −f(xx00)m thay vào, ta có f(0) =

f(x0), kéo theof(x)là hàm hằng, khơng thỏa mãn tính chất tồn ánh Điều cho ta thấy

m = 0vàn=

Cuối cùng, xétmn6= 0.Gọix0 số thực thỏa mãnf(x0) = 0thì chox=x0, ta có

f(x0+m·f(x0)) = f(x0) +n·x0f(1)⇒x0 =

hay f(0) = 0.Thayx = 1vào đề bài, ta cóf(1 +mf(y)) = +nf(y)nênf tuyến tính, mà f(0) = 0, f(1) = 1kéo theof(x) = x, thay vào có ngaym=n

2) Ta có

f(x+f(xy)) =f(x) +xf(y)

Chox=y=−1, ta cóf(−1 +f(1)) = 0nên tồn tạiuđểf(u) = (*) Thayx=u, y = 1,

ta có0 = muf(1) ⇒uf(1) = 0.Ta xét hai trường hợp:

1 Nếuf(1) = 0thì thayx= 1y, y 6= 0, ta cóf(y) = 0,∀y6= Tiếp tục thayy= 0thì

f(x+f(0)) =f(x) +xf(0)

Chọnx6= 0, x6=−f(0)thì dễ dàng có đượcf(0) = 0nên ta cóf(x) = 0,∀x∈R

2 Nếuf(1) 6= 0thìu = 0hay f(0) = 0.Thay vào (*), ta cóf(1) = 1.Với x, y 6= 0,đặt z =x+yfx

y

thì đề bài, thayx→ x

y, y →1, ta có

f

x y +f

x y =f x y + x 2y hay f z y =f x y + x 2y

Thayx→y, y→ x y, ta có

f(y+f(x)) =f(y) +yf

x y

Cuối cùng, thayx→y, y→ z y ta có

f(y+mf(z)) = f(y) +yf

z y

=f(y) +yf

x y

+x

Do đóf(y+f(z)) =f(y+f(x)) +x Thayy =−f(x)thìf(−f(x) +f(z)) =xnên f tồn ánh Thayx→1, y →xthì

f(1 +f(x)) =f(1) +f(x)

Vìf tồn ánh nên đặtt =f(x) + 1, ta cóf(t) =t+f(1)−1với mọit ∈ R.Thay vào

(105)

Vậy tất hàm cần tìm làf(x) = 0, f(x) = x Bài toán Cho hàm sốf :R→R+thỏa mãn

f(x−y)f(y−z)f(z−x) = 8f(x)≥2,∀x≥0

Xác địnhf(x)

Đặtf(x) = 2g(x)+1thì cóg(x)≥0,∀x≥0và

g(x−y) +g(y−z) +g(z−x) =

Suy rag(x) +g(y) +g(z) = 0với mọix+y+z = Suy rag hàm lẻ cộng tính Suy g(x) = axnênf(x) = 2ax+1 vớia ≥0.

Bài tốn Tìm tất hàm sốf :R→Rsao cho

f(x+y2+z3) =f(x) +f(y)2+f(z)3

với mọix, y, z

Lời giải. Thayx=y=z = 0cóf(0)2+f(0)3 = 0nênf(0) = 0, f(0) =−1

1 Nếuf(0) = 0thì thayz = 0là cóf(x+y2) = f(x) +f(y)2

≥ f(x)nênf đồng biến

Ngồi ra, ta có

f(x+z3) = f(x) +f(z)3

Suy raf(z3) =f(z)3nên

f(x+z3) =f(x) +f(z3), chứng tỏfcộng tính vàf(x) = ax.

2 Nếuf(0) =−1thì thay vào có

f(y2) =f(y)2 −2nên suy raf(x)2 =f(−x)2

Thayy= 0,dễ dàng cóf(x) +f(−x) = −2.Giải cóf(x) = f(−x) = −1,∀x

Vậy có hai hàm số thỏa mãn đề làf(x) = xvàf(x) =−1

Bài tốn 10 Tìm tất hàm sốf :R→Rthỏa mãn điều kiệnf(0) = 0

f(x+y3) +f(x2+y) = xf(x) +x+f(f(y+y3))

với mọix, y ∈R

Lời giải. Thếx= 0,ta cóf(y) +f(y3) =f(f(y+y3))nên viết lại đề thành

f(x+y3) +f(x2+y) =xf(x) +x+f(y) +f(y3) (∗)

Thayx→x3,−x3, ta có

(106)

f(−x3+y3) +f(x6+y) =−x3f(−x3)−x3+f(y) +f(y3)

Suy

f(x3+y3)−f(−x3 +y3) =x3 f(x3) +f(−x3) + 2.

Đổix3, y3 →x, yta có

f(x+y)−f(y−x) =x(f(x) +f(−x) + 2) (∗∗)

Thayy= 0,ta cóf(x)−f(−x) = x(f(x) +f(−x) + 2)nên(∗∗)viết lại thành

f(x+y)−f(y−x) = f(x)−f(−x)

Thayx =y,ta cóf(x)−f(−x) = f(2x)nênf(x+y)−f(y−x) =f(2x)với mọix, y.Từ

đây suy raf cộng tính Từ cóf hàm lẻ Thay lại vào (*), ta có f(x) +f(x2) = xf(x) +x

Thayx→ −x,ta có−f(x) +f(x2) = xf(x)−xnênf(x) = x.

Thử lại thấy thỏa Vậy tất hàm số cần tìm làf(x) =x

3 Bài tập tự luyện

Bài 1. Tìm tất hàm sốf :R+ →R+ thỏa mãn

1 f(xy) +f(x+y) = x+y+xy,∀x, y ∈R+.

2 x2(f(x) +f(y)) = (x+y)f(yf(x)),∀x, y >0.

Bài 2. Tìm tất hàmf, g :R+→R+sao chof(1) =g(1)và với mọix, y >0thì

f(g(x) +y) =f(x) +g(y)

g(f(x) +y) =g(x) +f(y)

Chứng minh rằngf(x) =g(x),∀x∈R+.

Gợi ý.Chứng tỏ rằngf(x)≥x, g(x)≥xvới mọix >0

Bài 3. Tìm tất hàm sốf :R+ → R+sao cho f(f(x) + 2y) = f(2x+y) + 2yvới mọi

x, y >0

Gợi ý.Nếu tồn tạia >0, b∈Rthỏaf(x+a) = f(x) +bvới mọix >0thìa=b

Bài 4. Tìm tất hàm sốf :R+ →R+ thỏa mãn

f 3f2(xy) + (xy)2= (xf(y) +yf(x))2

(107)

Gợi ý.Biến đổi thích hợp đưa vềxf(y) +yf(x) =cxy+f(xy)

Bài 5. Tìm tất hàmf :R+ →R+sao cho

f(x+f(y)) = f(x)−x+f(x+y)

với mọix, y >0

Gợi ý.Dùng phương pháp thêm biến

(x, y)→

z,f(y)

2 +x

,

y,f(z)

2 +x

để đưa vềf(x+u) = f(x+v) +whay f(x+u−v) = f(x) +w Từ chứng minh w= 2(u−v)để suy raf(x)−2x=const

Bài 6. Tìm tất hàmf :R+→R+sao cho

f(f(xy)−xy) +xf(y) +yf(x) =f(xy) +f(x)f(y),∀x, y >0

Gợi ý.Sử dụng phép

(x, y)→(x+f(z), y),(x, z),(x+y, z)

để thu đẳng thức cộng lại, từ chứng minh rằngf(x)−x=const

Bài 7. Tìm tất hàm sốf :R+ →R+ thỏa mãn

f(x)2−f(y)f(z) =f(xy)f(y)f(z)[f(yz)−f(zx)]

với số thựcx, y, z >0phân biệt

Gợi ý.Đổi vai trò(x, y, z)cho cộng lại, dùng bất đẳng thứcAM −GM

Bài 8. Cho hàm sốf :R→Rthỏa mãnf(x+y) = f(x) +f(y),∀x, y ∈R

1 Cho biết rằngf(x2020) =f(x)2020

.Chứng minh rằngf(x) =ax,∀x

2 Cho biết rằngf(x2019) = f(x)2019 Chứng minh rằng

f(x) =ax,∀x

Gợi ý.1) Từ giả thiết suy raf(x)≥0,∀x≥0nênf đồng biến Ta cóf(x) =ax,∀x ∈Qnên xét dãy để raf(x) = ax,∀x∈R

2) Tính hai cách biểu thứcf(x+y)2019

Bài 9. Cho hàm sốf :R→Rthỏa mãn:

f(x+y) = f(x) +f(y),∀x, y |f(x)−x| ≤2019

(108)

Gợi ý.Ta cóf(2x) = 2f(x)nênf(2nx) = 2nf(x).Giả sửf(x0) =x

0+εthì

f(2nx0) = 2nx0+ 2nε

vớiε6= 0.Thay vào đề, ta có|2nε| ≤2019,∀n,vơ lý.

Bài 10. Cho hàm sốf : R → Rliên tục thỏa mãnf(x+f(y+z)) +f(y+f(z +x)) +

f(z+f(x+y)) = 0với mọix, y, z ∈R

1 Chứng minh rằngf(0) =

2 Cho biết rằngf(2019)6= 0,chứng minh rằngf hàm số tồn ánh Từ tìm tất

hàm số thỏa mãn đề

Gợi ý.1) Chox=y=z,ta có

f(x+f(2x)) = 0nênf−x

2 +f(x)

=

Lại thayx→f(x)−x thìf

x −

1 2f(x)

= 0,cứ cóf(0) =

2) Ta có

f(f(2x)−x) =−2f(x)

nên cóf(x0)6= 0thì xét dãy để cóf(x)→ ±∞, tính liên tục nên tồn ánh Cho

y=z = 0thìf(x) + 2f(f(x)) = 0nênf(x) = −x2

Bài 11. Xét hàm sốf :R+ →Rsao cho với mọix, y >0thì

i) f(x) +f(y)≤ f(x+y)2

ii) (x+y) [yf(x) +xf(y)]≥xyf(x+y)

1 Chứng minh rằngf(2nx) = 4nf(x),∀x >0.

2 Đặtg(x) = f(x)x , chứng minh rằngg(nx) = ng(x),∀n∈Z+, x >0.

3 Tìm tất hàm sốf(x)thỏa mãn đề

Bài 12. Tìm tất song ánhf :R→Rliên tục cho

i) f(f(x)) = 4f(x)−3xvới số thựcx

ii) lim

x→−∞ (3x−f(x)) = −∞vàx→lim+∞ (3x−f(x)) = +∞

Bài 13. Tìm tất hàm sốf :R→Rliên tục thỏa mãn:

1 f(f(f(x))) = 2x−f(x)với mọix

(109)

Bài 14. Tìm tất hàm số liên tụcf : [−1; 1]→[−1; 1]sao cho:

1 f(x) =f(4x3−3x)với mọi−1≤x≤1.

2 2f(x) = f x2+f x+12 với mọi−1≤x≤1

Bài 15.

1 Cho hàm số liên tục f, g : R → R thỏa mãn f(x) ∈ Q khig(x) ∈/ Q

Chứng minh rằngf, glà hàm

2 Giả sửf hàm số liên tục thỏa mãn

f(x) +f

x+2019

∈Q⇔f(x+ 11) +f(x+ 4) +f(x+ 2019)∈/ Q

Chứng minh phương trìnhx11−4x+ 2019 =f(x)có nghiệm.

Bài 16. (PTH sau phép biến đổi) Tìm tất hàm sốf :R→Rliên tục cho:

1 f(x3)−f(x) =x(x2−1).

2 f(x)−x=f(f(x)−x)vàf(0) =

3 f(x) +f(x2+ 2x) =x2+ 3x+ 4026với mọix. Bài 17. Tìm tất hàm sốf :R+ →R+ liên tục cho:

i) f(2x) = 2f(x)

ii) f f3(x) ef(x)−1=x2(ex−1)f(x)với mọix >0.

iii) f(e−1) = (e−1)f(1)

iv) f(k)∈Z+,∀k ∈Z+.

Bài 18. Tìm tất hàm số liên tụcf :R→Rthỏa mãn

f(x3+y3+xf2(x) +yf2(y)) = f(x) +f(y)

với mọix, y ∈R

Gợi ý.Đặtg(x) =x(x2+f(x2))thìgcũng liên tục, lẻ vàg(0) = 0.Ta viết lại thành 2f(g(x) +g(y)) =f(x) +f(y)

Ta chứng minh nếug(x)không bị chặn để suy rag(x)tồn ánh trênR.Từ tìm hai

(110)

Tài liệu

[1] Phạm Văn Ninh, Các tốn phương trình hàm liên tục, 2019 [2] Group"Hướng tới Olympic Toán VN"trên facebook

[3] Group"Bài toán hay - Lời giải đẹp"trên facebook

[4] Võ Quốc Bá Cẩn, Xây dựng dãy số giải phương trình hàm (tạp chí Epsilon) [5] artofproblemsolving.com

(111)

M

ỘT

S

B

ẤT

Đ

ẲNG

T

HỨC

T

RONG

C

ÁC

K

T

HI

O

LYMPIC

T

ỐN

H

ỌC

Q

UỐC

T

Ngơ Văn Thái

(Thái Bình)

GIỚI THIỆU

Kỳ thi Olympic Toán học Quốc tế (IMO) tổ chức năm lần năm 1959 đến Kỳ thi dành cho học sinh THPT, nội dung đề thi gồm có tốn thuộc phân mơn khác chương trình Tốn học sơ cấp Đây kỳ thi danh giá hành tinh, qui mô kỳ thi ngày phát triển không ngừng, chất lượng kỳ thi nâng lên ngang tầm thời đại Ảnh hưởng tích thi lớn, tạo động lực thúc đẩy phong trào phát bồi dưỡng nhân tài Toán học Thế giới cách rộng khắp mạnh mẽ Những thí sinh đạt giải cao kỳ thi IMO, phần lớn em sau theo nghiệp làm toán trở thành nhà toán học, nhà khoa học, nhà giáo có sử dụng nhiều đến toán Sau năm ban tổ chức kỳ thi Quốc tế thường thay đổi dạng toán nâng dần độ khó đề thi Bởi quốc gia có thí sinh tham dự kỳ thi, phải nắm bắt qui luật để đổi công việc tuyển học sinh khiếu, đổi nội dung định thời gian giảng dậy cho phù hợp đạt hiệu cao Trong đề thi IMO năm gần không thấy xuất câu chứng minh bất đẳng thức, lý có ban giám khảo Quốc tế hiểu rõ hết Song có điều dễ nhận thấy, câu bất đẳng thức kỳ thi Olympic trước thường câu khó, cản trở nhiều sỹ tử muốn dành điểm thi tuyệt đối câu đem lại quan tâm đặc biệt người đam mê Toán học sơ cấp

(112)

1 IMO 1961, Hungary

Bài toán Gọia; b; c là độ dài ba cạnh tam giác có diện tích làS:Chứng minh rằng a2Cb2Cc2>4p3S:

Lời giải Vìa; b; c độ dài ba cạnh tam giác nênaCb c > 0; b Cc a > 0; cCa b > 0:Ap dụng bất đẳng thức AM-GM ta có

aCbCc >3p3

.aCb c/ bCc a/ cCa b/;

hay

3 q

.aCbCc/4 >33 s

16

aCbCc

aCb c

bCc a

cCa b

3 q

.aCbCc/4 >3p3 16S2;

Do

.aCbCc/2 >3p34S:

Kết hợp với đánh giá quen thuộc3 a2Cb2Cc2>.aCbCc/2

;ta a2Cb2Cc2>4p3S:

Bài toán chứng minh

Lời giải Ta có đẳng thức (bạn đọc tự chứng minh)

2 a2b2Cb2c2Cc2a2 a4Cb4Cc4D16S2:

Kết hợp với đánh giá

a4Cb4Cc4> a

2

Cb2Cc22 >a2b2Cb2c2Cc2a2;

ta

a4Cb4Cc4C a

2

Cb2Cc22 >16S2C a4Cb4Cc4; a2Cb2Cc2>4p3S:

(113)

2 IMO 1961, Hungary

Bài toán Gọi P điểm tùy ý nằm tam giác ABC, PA cắt BC D, PB cắt AC E, PC cắt

AB F Chứng minh có tỷ số sau không lớn 2: AP PD;

BP PE;

CP PF và có tỷ số không nhỏ thua 2.

Lời giải Theo định lý Gergonne PD

AD C

PE BE C

PF CF D1;

suy ba tỷ số PD AD;

PE BE;

PF

CF có tỷ số không nhỏ 3:

Không tính tổng quát ta giả sử tỷ số PD AD >

1 3:Từ

PD AD >

1

3 ,

AD

PD 63,

AP CPD

PD 63,

AP PD 62

Chứng minh tương tự ba tỷ số AP PD;

BP PE;

CP

PF có tỷ số không nhỏ

thua2:Khi điểmP trùng với trọng tâmGcủa tam giác ABC tỷ số bằng2:

Bài toán chứng minh

Nhận xét Năm1818nhà toán học Gergonne người Pháp chứng minh định lý sau:

Nếu điểmP nằm tam giácABC, đường thẳngAP; BP; CP cắt cạnhBC; AC; AB

theo thứ tự tạiD; E; F;thì PD

AD C

PE BE C

PF CF D1:

Cách chứng minh định lý Gergonne đơn giản xin dành lại cho bạn đọc Như thực chất toán hệ định lý Gergonne mà (lời giải khác với lời giải đưa đáp án kỳ thi)

3 IMO 1964, Liên Xô

(114)

Lời giải Vìa; b; c độ dài ba cạnh tam giác nên a b/2.aCb c/>0; b c/2.bCc a/>0; c a/2.cCa b/>0;

Cộng vế theo vế ba bất đẳng thức rút gọn

3abc >a2.bCc a/Cb2.cCa b/Cc2.aCb c/

Dấu xảy tam giác cho Bài tốn chứng minh

Lời giải Khơng tính tổng quát giả sử

a>b>c > )a2Cbc >b2Cca>c2Cab:

Theo bất đẳng thức hoán vị

a a2CbcCb b2Cca>a b2CcaCb a2Cbc; a a2CbcCc c2Cab>a c2CabCc a2Cbc; b b2CcaCc c2Cab>b c2CabCc b2Cca;

Cộng vế với vế ba bất đẳng thức ta có điều phải chứng minh

Nhận xét Với giải thiết tương tự, mời bạn đọc thử sức với bất đẳng thức sau a2.2b c/Cb2.2c a/Cc2.2a b/>3abc:

a2.2c b/Cb2.2a c/Cc2.2b a/>3abc:

4 IMO 1969, Rumania

Bài toán Chox1; x2> 0; x1y1 > z12; x2y2 > z22:Chứng minh rằng

1 x1y1 z12

C x

2y2 z22

>

.x1Cx2/ y1Cy2/ z1Cz2/2

(115)

Lời giải Từ giả thiết ta đượcy1; y2 > 0:Đặt

x1y1 z12Dk1; x2y2 z22 Dk2; k1; k2 > 0:

.x1Cx2/ y1Cy2/ z1Cz2/2 D x1y1 z12

C x2y2 z22

Cx1y2Cx2y1 2z1z2

Dk1Ck2C

k

1Cz12

y1

y2C

k

2Cz22

y2

y1 2z1z2

Dk1Ck2C

k

1y2

y1 C

k2y1

y2

C

z2 1y2

y1 C

z22y1

y2

2z1z2:

Theo bất đẳng thức Cauchy-Schwarz hai số ta

k1y2

y1 C

k2y1

y2

>2pk1k2;

z12y2

y1 C

z22y1

y2

>2jz1z2j:

Do

.x1Cx2/ y1Cy2/ z1Cz2/2 >k1Ck2C2

p

k1k2C2jz1z2j 2z1z2 >

p

k1C

p k2 : Suy p

k1Cpk2

2 >

8

.x1Cx2/ y1Cy2/ z1Cz2/2

:

Lại sử dụng bất đẳng thức Cauchy-Schwarz ta dễ dàng chứng minh

1 k1 C

1 k2 >

8

p

k1Cpk2

2:

Vậy

1 x1y1 z12

C

x2y2 z22

>

.x1Cx2/ y1Cy2/ z1Cz2/2

:

Đẳng thức xảy khix1 Dx2; y1 Dy2; z1 Dz2:Bài toán chứng minh

Nhận xét Khái quát tốn ta dễ dàng có mở rộng sau:

Chox1; x2; : : : ; xn2 RC; y1; y2; : : : ; yn; z1; z2; : : : ; zn2R;thỏa mãn

xkyk zk2 > 0; 8kD1; n: Chứng minh

n

X

kD1

1

xkyk zk2 >

n3

Pn kD1

xk n

P

kD1

yk

n

P

kD1

zk

2:

(116)

Cho

xi k; zk > 0; m

Y

iD1

xi k zkm > 0; 8i D1; m; 8k D1; n; m; n2N: Chứng minh rằng

n

X

kD1

1

Qm iD1

xi k zkm

> Qm nmC1

iD1

Pn

kD1

xi k

n

P

kD1

zk

m:

5 IMO 1975, Bungaria

Bài toán Cho (

a1 >a2 > >an

b1 >b2> >bn

gọi.c1; c2; : : : ; cn/là hoán vị tùy ý của.b1; b2; : : : ; bn/ Chứng minh rằng

.a1 b1/2C.a2 b2/2C C.an bn/2 6.a1 c1/2C.a2 c2/2C C.an cn/2:

Lời giải Vì.c1; c2; : : : ; cn/là hoán vị tùy ý của.b1; b2; : : : ; bn/nên

b21Cb

2 C Cb n Dc

2 1Cc

2

2C Cc n:

Theo bất đẳng thức hốn vị

a1c1Ca2c2C Cancn6a1b1Ca2b2C Canbn;

tương đương với

2a1b1 2a2b2 2anbn6 2a1c1 2a2c2 2ancn;

.a1 b1/2C.a2 b2/2C C.an bn/2 6.a1 c1/2C.a2 c2/2C C.an cn/2:

Dấu xảy

a1Da2 D Dan b1 Db2D Dbn:

(117)

Nhận xét Dùng phép biến đổi tương đương toán ta toán sau:

Choa1 > a2 > > an; b1 > b2 > > bn Gọi.c1; c2; : : : ; cn/là hoán vị tùy ý của

.b1; b2; : : : ; bn/ :Chứng minh rằng

.a1Cb1/2C.a2Cb2/2C C.anCbn/2 >.a1Cc1/2C.a2Cc2/2C C.anCcn/2:

Khái quát cách giải, áp dụng nhị thức Newton lại cho ta toán mở rộng đẹp sau:

Cho hai dãy số thực không âma1; a2; : : : ; an; b1; b2; : : : ; bn.x1; x2; : : : ; xn/là hoán vị tùy ý của.b1; b2; : : : ; bn/,m 2N:

a) Nếua1; a2; : : : ; anb1; b2; : : : ; bnsắp thứ tự chiều, thì

.a1Cb1/mC.a2Cb2/mC C.anCbn/m >.a1Cx1/mC.a2Cx2/mC C.anCxn/m:

b) Nếua1; a2; : : : ; anb1; b2; : : : ; bnsắp thứ tự ngược chiều, thì

.a1Cb1/mC.a2Cb2/mC C.anCbn/m 6.a1Cx1/mC.a2Cx2/mC C.anCxn/m:

6 IMO 1983, Pháp

Bài toán Choa; b; c là độ dài ba cạnh tam giác Chứng minh rằng a2b a b/Cb2c b c/Cc2a c a/>0: Lời giải Thay

aDyCz; b DzCx; cDxCy phép Ravi/:

Bất đẳng thức cho viết lại thành

.yCz/2.zCx/.y x/C.zCx/2.x Cy/.z y/C.xCy/2.yCz/.x z/>0:

hay

x3z Cy3xCz3y >x2yz Cy2zxCz2xy:

Bất đẳng thức theo bất đẳng thức Cauchy-Schwarz

.x3z Cy3xCz3y/.y2zxCz2xyCx2yz/>.x2yzCy2zxCz2xy/2:

(118)

Lời giải (Leeb Bernhard) Khơng tính tổng qt, giả sửaDmax.a; b; c/ ;thì a2b a b/Cb2c b c/Cc2a c a/

Da bCc a/ b c/2Cb a b/ a c/ aCb c/>0:

Chứng minh hoàn tất

Nhận xét Bạn đọc thử sức cách thay điều kiện toán thànha>b >c > 0:

Bài toán mở rộng:

Gọia; b; c là độ dài ba cạnh tam giác vàn2 N; n>2:Chứng minh rằng anb a b/Cbnc b c/Ccna c a/>0:

7 IMO 1984, Tiệp Khắc

Bài toán Cho ba số thực không âma; b; c thỏa mãnaCbCcD1:Chứng minh rằng 06abCbcCca 2abc6

27: Lời giải VìaCbCc D1;nên

abCbcCca 2abcDab c/Cbc a/Cca>0:

Đẳng thức xảy khi.a; b; c/là bộ.1; 0; 0/; 0; 1; 0/; 0; 0; 1/:

Từ giả thiết ta có

.1 2a/C.1 2b/C.1 2c/D1:

Cho nên tổng hai số số1 2a; 2b; 2cđều khơng âm nên

xảy hai trường hợp:

(i) Tất ba số không âm Khi theo bất đẳng thức AM-GM ta có

.1 2a/ 2b/ 2c/6

.1 2a/

C.1 2b/C.1 2c/

3

;

tương đương với

1 aCbCc/C4 abCbcCca/ 8abc 27; abCbcCca/ 8abc

27 C1; abCbcCca 2abc

(119)

(ii) Có số âm Khi hiển nhiên

.1 2a/ 2b/ 2c/ < 27;

hay

abCbcCca 2abc < 27:

Đẳng thức xảy khiaDb Dc D 13:

Bài toán chứng minh

8 IMO 1995, Canada

Bài toán Choa; b; c là ba số thực dương thỏa mãnabc D1:Chứng minh rằng

a3.bCc/ C

1

b3.cCa/C

1

c3.aCb/ >

3 2: Lời giải Đặtx D a1; y D b1; z D 1c thìxyz D1và bất đẳng thức trở thành

C D x

2

yCz C y2 zCx C

z2 xCy >

3 2:

Theo bất đẳng thức Schwarz bất đẳng thức Cauchy

C > xCy Cz/

2

.yCz/C.zCx/C.x Cy/ D

xCyCz >

3p3 xyz

2 D

3 2:

Đẳng thức xảy khix Dy Dz D1, tức làaDb Dc D1: Nhận xét Ta có bất đẳng thức tổng quát sau, với điều kiện tương tự

1

ak.bCc/C

1

bk.cCa/C

1

ck.aCb/ >

3

2; k >2:

9 IMO-1996, Ấn Độ

Bài tốn Cho lục giác lồiABCDEF AB song song vớiDE; BC song song vớiEF CDsong song vớiFA:GọiRA; RC; RE là bán kính đường trịn ngoại tiếp tam giácFAB;

BCD; DEF tương ứng gọiP là chu vi lục giác Chứng minh rằng RACRC CRE >

(120)

Lời giải (John Scholes) Áp dụng định lý hàm số sin cho tam giác tương ứng, ta có 2RAD

BF

sinA; 2RC D BD

sinC; 2RE D FD

sinE:

Kéo dài cạnh BC FE kẻ đường thẳng a/ d / quaA D vng góc với

chúng Khi đóBF lớn hay đoạn vng góc quaAvà quaD:

Ta tìm độ dài đoạn cách chiếuBAvàAF lên.a/để đượcABsinBC AF sinF:Tương tự, độ dài cạnh quaD bằngCDsinC CDEsinE:Do

2BF >ABsinB CAFsinF CCDsinC CDEsinE:

Tương tự

2BD>BC sinBCCDsinDCAF sinACEF sinE;

2FD >ABsinACBC sinC CDEsinDCEF si nF:

Từ

2BF

sinA C 2BD

sinC C 2FD

sinE >AB

sin

A

sinE C

sinB sinA CBC sin B

sinC C

sinC sinE CCD sin C

sinA C

sinD

sinC

CDE

sinE

sinA C

sinD

sinE

CEF

sinE

sinC C

sinF

sinE

CFA

sinF

sinA C

sinA

sinC

Bây ta sử dụng điều kiện cạnh song song để suy raA DD; B D E; C D F:Mỗi

thừa số nhân với cạnh có dạngxC 1x có giá trị nhỏ bằng2khixD1:Do 2BF

sinA C 2BD

sinC C 2FD

sinE >2P:

(121)

10 IMO 2000, Hàn Quốc

Bài toán 10 Choa; b; c là ba số thực dương thỏa mãnabc D1:Chứng minh rằng

a 1C

b b 1C

1

c c 1C

1 a

61:

Lời giải Từ điều kiện, ta đặtxDa; y D1 zD 1b Dca;bất đẳng thức trở thành x yCz/y

z 1C

x z

z

x 1C

1 x

61;

tương đương với

z Cx y y

xCy z z

yCz x x

61; xCy z/.yCz x/.zCx y/6xyz:

Đây bất đẳng thức Schur quen thuộc Bài tốn chứng minh

11 IMO 2001, Hoa Kỳ

Bài toán 11 Chox; y; zlà ba số thực dương Chứng minh rằng x

p

x2C8yz C

y

p

y2C8zx C

z

p

z2C8xy >1:

Lời giải (Nguyễn Khắc Minh) Áp dụng bất đẳng thức Cauchy-Schwarz ta có x

p

x2C8yzC

y

p

y2C8zxC

z

p

z2C8xy >

.xCy Cz/2

xpx2C8yzCypy2C8zxCzpz2C8xy:

Lại áp dụng bất đẳng thức Cauchy-Schwarz, ta có

xpx2C8yzCypy2C8zxCzpz2C8xy 6p.xCyCz/.x3Cy3Cz3C24xyz/:

Như vậy, ta cần chứng minh

.x CyCz/3 >x3Cy3Cz3C24xyz; x2yCx2zCy2xCy2zCz2xCz2y >6xyz:

(122)

Lời giải Ta chứng minh

x

p

x2C8yz >

3 p

x4

3 p

x4Cp3

y4Cp3 z4:

Thật vậy, cách bình phương hai vế ta viết bất đẳng thức lại sau

p3

x4Cp3

y4Cp3z42 > p3 x2.x2

C8yz/:

Sử dụng bất đẳng thức AM-GM, ta có

p3

x4Cp3

y4Cp3 z42 p3 x42 D2p3 x4Cp3

y4Cp3 z4 p3

y4Cp3 z4

>4p3

x2yz2p3

y2z2

D8p3 x2yz:

hay

3 p

x4Cp3

y4Cp3 z42 >p3 x42C8p3 x2yz D p3 x2 x2

C8yz;

tương đương với

x

p

x2C8yz >

3 p

x4

3 p

x4Cp3

y4Cp3

z4:

Do

X x

p

x2C8yz >

X p3

x4

3 p

x4Cp3

y4Cp3

z4 D1:

Bài toán chứng minh

Nhận xét Từ toán xin đề xuất hai toán mở rộng sau đây:

Chox; y; zlà ba số thực dương Chứng minh với06˛61ta có x

p

x2C8yz ˛x C

y

p

y2C8zx ˛y C

z

p

z2C8xy ˛z >

3 ˛:

Cho ba số thực dươnga; b; c p; q2ZC; p>2q:Chứng minh rằng a

p p

.a2C8bc/q C

b

p p

.b2C8ca/q C

c

p p

.c2C8ab/q >.aCbCc/

p 2q p :

12 IMO 2005, Mexico

Bài toán 12 Cho ba số thực dươngx; y; zthỏa mãnxyz >1:Chứng minh rằng x5 x2

x5Cy2Cz2 C

y5 y2 y5Cz2Cx2 C

z5 z2

(123)

Lời giải (Iurie Boreico) GọiS vế trái toán, vớix; y; z > 0thì

.x3 1/x3.x2Cy2Cz2/ x5Cy2Cz2/D.x3 1/2.y2Cz2/>0:

Do

x5 x2 x5Cy2Cz2 >

x3

x.x2Cy2Cz2/ D

x2 x1 x2Cy2Cz2 >

x2 yz x2Cy2Cz2:

Cộng vế với vế bất đẳng thức với hai bất đẳng thức tương tự, ta

S > x

2Cy2Cz2 .xyCyzCzx/

x2Cy2Cz2 >0:

Đẳng thức xảy khix Dy Dz D1: Nhận xét Một số toán tổng quát

Chox,y; z là số thực dương thỏa mãnxyz > 1:Chứng minh với 06 2r 6k 5r2

ta có

xk xr

xk CyrCzr C

yk yr

yk Czr Cxr C

zk zr

zkCxrCyr >0:

Cho ba số thực dươngx; y; z thỏa mãn điều kiện xyz > ˛ > ˇC3; ˇ > 1; R:

Chứng minh rằng

x˛ xˇ

xˇCC1CyCz C

y˛ yˇ

yˇCC1CzCx C

z˛ zˇ

zˇCC1CxCy >0:

13 IMO 2008, Tây Ban Nha

Bài toán 13 Cho ba s thcx; y; z Ô1tha mónxyz D1 Chng minh rằng x2

.x 1/2 C y2 y 1/2 C

z2

.z 1/2 >1:

Chứng minh đẳng thức xảy vô số bộ.x; y; z/hu t.

Li gii Vỡx; y; z Ô xyz D nên tồn ba số thực dươnga; b; c chox D ab; y D bc; z D ca vàa; b; c ba số phân biệt Ta cần chứng minh

a2 a b/2 C

b2 b c/2 C

c2

(124)

Theo bất đẳng thức Cauchy-Schwarz

X a2 a b/2 >

Œa a c/Cb b a/Cc c b/2

.a b/2.a c/2C.b c/2.b a/2C.c a/2.c b/2: (1)

Chú ý

Œa a c/Cb b a/Cc c b/2 D.a2Cb2Cc2 ab bc ca/2;

.a b/2.a c/2C.b c/2.b a/2C.c a/2.c b/2 D a2Cb2Cc2 ab bc ca2;

nên

a2

.a b/2 C b2

.b c/2 C c2

.c a/2 >1:

Đánh giá (1) xảy đẳng thức

a a b

.a b/ a c/ D

b b c

.b c/ b a/ D

c c a

.c a/ c b/;

tương đương với

a

.a b/2.a c/ D

b

.b c/2.b a/ D

c

.c a/2.c b/ b

aC c b C

a c D3;

hay x C y C z D3:

Từ phương trình ta chọn

.x; y; z/D

n

.nC1/2; n nC1/ ;

nC1 n2

; n2 Q:

Chứng minh hoàn tất

14 IMO 2012, Argentina

Bài toán 14 Chon 1số thực dươnga2; a3; : : : ; anthỏa mãna2a3 an D1:Chứng minh

(125)

Lời giải Vớik 2N; 26k6n, áp dụng bất đẳng thức AM-GM ta có

1Cak D

1

k 1C

1

k 1C C

k

„ ƒ‚ …

k

Cak >k

k v u u u tak

1 k 1/ k 1/ k 1/ „ ƒ‚ … k ;

.1Cak/k >

kk

.k 1/k 1ak;

suy

n

Y

kD2

.1Cak/k > n

Y

kD2

"

kk k 1/k 1ak

#

Dnna2a3 anDnn:

Vậy

.1Ca2/2.1Ca3/3 1Can/n>nn: (2)

Dấu (2) xảy

a2a3 anD1; a2D1; a3 D

1

2; a4 D

3; : : : ; an D n 1;

hay

a2a3 anD1; a2a3 anD1

1

1

1 n 1;

hoặc

1

23 n 1/ D1; (vơ lý):

Do dấu đẳng thức khơng xảy Bài tốn chứng minh

Tài liệu

[1] Vũ Dương Thụy, Nguyễn Văn Nho,2003; 40năm Olympic Toán học Quốc tế, Nhà xuất

Giáo dục

[2] Trần Nam Dũng, 2017; Phương pháp giải toán qua toán Olympic, Nhà xuất

Thế giới

[3] Trần Phương, Võ Quốc Bá Cẩn, Trần Quốc Anh,2010;Vẻ đẹp bất đẳng thức kỳ

thi Olympic toán học, Nhà xuất Đại học Quốc gia Hà Nội [4] Ngô Văn Thái,2018;Sáng tạo-Làm chặt, Tạp chí Epsilon

[5] Ngơ Văn Thái,2019;Phương pháp chứng minh bất đẳng thức, Gặp gỡ Toán học-Vật lý,

(126)

[6] Phạm Kim Hùng,2006;Sáng tạo bất đẳng thức, Nhà xuất Tri thức

(127)

C

ÁC

B

ÀI

T

ỐN

C

ỰC

T

RỊ

T

RONG

K

HƠNG

G

IAN

T

OẠ

Đ

Nguyễn Tất Thu

(Trường THPT chuyên Lương Thế Vinh - Đồng Nai)

TÓM TẮT

Bài tồn cực trị nói chung hay cực trị tọa độ khơng gianOxyzthường tạo khó khăn cho học sinh Khó khăn học sinh thường gặp đứng trước tốn cực trị khơng gianOxyzlà: cách xử lí tốn đó? kiến thức cần dùng? tâm lí nữa! Bài viết nhằm giúp em học sinh tìm hướng xử lí gặp tốn cực trị khơng gianOxyz

Với tốn cực trị khơng gian Oxyz, thường xử lí theo hai hướng

sau:

Hướng 1: (Đại số) Chuyển đại lượng cần tìmmin, maxvề biểu thức đại số dùng

bất đẳng thức khảo sát hàm số để tìmmin, max

Hướng 2: (Hình học)Với hướng làm này, ta sử dụng bất đẳng thức phần để đánh giá

Với cách giải theo hướng đại số có lợi cần đến trí tưởng tượng khơng gian mà cần tính tốn nhiều hơn, nhiều thời gian dễ có sai sót

Với cách giải theo hướng Hình học địi hỏi học sinh cần có tưởng tượng khơng gian tốt thường có lời giải ngắn gọn

Dù theo cách em cần nắm kiến thức bất đẳng thức đại số, khảo sát hàm số bất đẳng thức hình học Trước hết, ơn lại kiến thức

1 Một số bất đẳng thức bản

Những kết trình bày em học cấp THCS THPT

Kết (Quan hệ góc cạnh đối diện tam giác) Trong tam giác, cạnh đối

(128)

Kết (Quan hệ đường xiên đường vng góc) Trong đường xiên đường

vng góc kẻ từ điểm nằm ngồi đường thẳng đến đường thẳng đường vng góc là đường ngắn Như hình vẽ ta ln cóAM ≥AH

A

M H

Kết (Bất đẳng thức tam giác) Với ba điểmA, B, C bất kì ta ln có bất đẳng thức AB+BC ≥AC

Tổng quát ta có bất đẳng thức đường gấp khúc: VớinđiểmA1, A2, , Anta ln có

A1A2+A2A3+· · ·+An−1An≥A1An

Kết (Bất đẳng thức trung bình cộng trung bình nhân) Với hai số khơng âm x, yta ln có

x+y

2 ≥2

xy

Đẳng thức xảy khix=y.

Kết (Bất đẳng thức tích vơ hướng hai véctơ) Với hai véctơ~a, ~bta ln có

|~a·~b| ≤ |~a| · |~b|

Đẳng thức xảy khi~a=k~b, k∈R

2 Một số toán thường gặp

Phần giới thiệu số toán thường gặp cách giải tốn

Bài toán Cho điểmAcố định điểm M di động hình(H)((H)là đường thẳng, mặt

phẳng) Tìm giá trị nhỏ củaAM.

A

(129)

Lời giải. GọiH hình chiếu vng góc củaA lên hình (H) Khi đó, tam giácAHM

vng tạiM, ta có

AM ≥AH

Đẳng thức xảy khiM ≡H

Do đóAM nhỏ khiM hình chiếu củaAlên(H)

Bài tốn Cho điểmAvà mặt cầu(S)có tâmI, bán kínhR.M là điểm di động trên(S) Tìm

giá trị nhỏ giá trị lớn củaAM.

A

M

M2

M1

I

Lời giải. XétAnằm mặt cầu(S) GọiM1,M2lần lượt giao điểm đường thẳngAI

với mặt cầu(S)(AM1 < AM2) và(α)là mặt phẳng quaM đường thẳngAI Khi đó(α)

cắt(S)theo đường trịn lớn(C) Ta cóM\1M M2 = 90◦, nênAM M\2vàAM\1M góc

tù, nên tam giácAM M1 vàAM M2ta có

AI−R=AM1 ≤AM ≤AM2 =AI+R

Tương tự vớiAnằm mặt cầu ta có

R−AI ≤AM ≤R+AI

VậyminAM =|AI −R|, maxAM =R+AI

Bài toán Cho mặt phẳng(P)và hai điểm phân biệtA, B Tìm điểmM thuộc(P)sao cho

1. M A+M Bnhỏ nhất. 2. |M A−M B|lớn nhất.

Lời giải.

1 Ta xét trường hợp sau

TH 1:NếuAvàB nằm hai phía so với(P) Khi

AM +BM ≥AB

(130)

TH 2:NếuAvàB nằm phía so với(P) GọiA0đối xứng vớiAqua(P).

Khi

AM +BM =A0M+BM ≥A0B

Đẳng thức xảy khiM giao điểm củaA0B với(P)

A

A0

M

B H

A

M

B

2 Ta xét trường hợp sau

TH 1:NếuAvàB nằm phía so với(P) Khi

|AM −BM| ≤AB

Đẳng thức xảy khiM giao điểm củaABvới(P)

TH 2:NếuAvàB nằm hai phía so với(P) GọiA0 đối xứng vớiAqua(P) Khi

đó

|AM −BM|=|A0M−BM| ≤A0B

Đẳng thức xảy khiM giao điểm củaA0B với(P).

Bài tốn Viết phương trình mặt phẳng(P)đi quaAvà cáchB một khoảng lớn nhất.

A H

B

Lời giải. GọiH hình chiếu củaB lên mặt phẳng(P), d(B,(P)) = BH ≤BA

(131)

Bài toán Cho số thực dươngα, β và ba điểmA, B, C Viết phương trình mặt phẳng

(P)đi quaCT =αd(A,(P)) +βd(B,(P))nhỏ nhất.

Lời giải.

1 XétA, Bnằm phía so với(P)

• NếuABk(P)thì

P = (α+β)d(A,(P))≤(α+β)AC

• Nếu đường thẳngAB cắt(P)tạiI Gọi Dlà điểm thỏa mãnIB~ = α

βID~ E

trung điểmBD Khi

P =αd(A,(P)) +β· IB

ID ·d(D,(P)) = 2αd(E,(P))≤2(α+β)EC

2 XétA, Bnằm hai phía so với(P) GọiI giao điểm củaABvà(P),B0 là điểm đối

xứng vớiB quaI Khi

P =αd(A,(P)) +βd(B0,(P))

Đến ta chuyển trường hợp

So sánh kết ta chọn kết lớn

Bài tốn Trong khơng gian chon điểmA1, A2, , An và điểmA Viết phương trình mặt

phẳng(P)đi quaAvà tổng khoảng cách từ điểmAi(i= 1, n) lớn nhất.

Lời giải.

• Xét nđiểm A1, A2, , An nằm phía so với(P) GọiGlà trọng tâm củan điểm

đã cho Khi n

X

i=1

d(Ai,(P)) =nd(G,(P))≤nGA

• Trongnđiểm cómđiểm nằm phía vàkđiểm nằm phía khác (m+k =n)

Khi đó, gọiG1là tâm củamđiểm,G2là trọng tâm củak điểmG3đối xứng vớiG1

quaA Khi

P =md(G3,(P)) +kd(G2,(P))

Đến ta chuyển toán

Bài toán Viết phương trình mặt phẳng(P)đi qua đường thẳngvà cáchAmột khoảng lớn

(132)

A

H

K

Lời giải. GọiH, K hình chiếu củaAlên mặt phẳng(P)và đường thẳng∆ Khi d(A,(P)) =AH ≤AK

Do đó(P)là mặt phẳng quaKvà vng góc vớiAK

Bài tốn Trong khơng gianOxyz,cho điểmA1, A2, , An.Xét véc tơ

~

w=α1M A~ 1+α2M A~ 2+· · ·+αnM A~ n

Trong đóα1, α2, , αnlà số thực cho trước thỏa mãnα1+α2+ .+αn 6= 0.Tìm điểm

M thuộc mặt phẳng(P)sao cho|w~|có độ dài nhỏ nhất.

Lời giải. GọiGlà điểm thỏa mãn

α1GA~ 1+α2GA~ 2+· · ·+αnGA~ n=~0

(điểmGhoàn toàn xác định)

Ta cóM A~ k=M G~ +GA~ k vớik = 1; 2; .;n,nên ~

w = (α1+ α2+ +αn)M G~ +α1GA~ 1+α2GA~ 2+· · ·+αnGA~ n = (α1+ α2+ +αn)M G.~

Do

|w~|=|α1+ α2+ · · ·+αn|

M G~

Vìα1 + α2+ · · ·+αn số khác khơng nên|w~|có giá trị nhỏ M G

nhỏ nhất, màM ∈(P)nên điểmM cần tìm hình chiếu củaGtrên mặt phẳng(P) Bài tốn Trong khơng gianOxyz,cho điểmA1, A2, , An.Xét biểu thức:

T =α1M A21+α2M A22+· · ·+αnM A2n

Trong đóα1, α2, , αnlà số thực cho trước Tìm điểmM thuộc mặt phẳng(P)sao cho

1. T giá trị nhỏ biếtα1+ α2+ +αn >0

(133)

Lời giải. GọiGlà điểm thỏa mãn

α1GA~ 1+α2GA~ 2+· · ·+αnGA~ n =~0

Ta cóM A~ k=M G~ +GA~ k vớik = 1; 2; .;n,nên M A2k =M G~ +GA~ k

2

=M G2+ 2M G ~~ GAk+GA2k

Do

T = (α1+ α2+ +αn)M G2+α1GA21+α2GA22+· · ·+αnGA2n

Vìα1GA21+α2GA22+· · ·+αnGA2nkhơng đổi nên

• vớiα1+ α2+ +αn >0thìT đạt giá trị nhỏ khiM Gnhỏ

• vớiα1+ α2+ +αn <0thìT đạt giá trị lớn khiM Gnhỏ

MàM ∈(P)nênM Gnhỏ điểmM hình chiếu củaGtrên mặt phẳng(P)

Bài tốn 10 Trong không gian Oxyz, cho đường thẳng d và mặt phẳng (P) cắt Viết

phương trình mặt phẳng(Q)chứadvà tạo với mặt phẳng(P)một góc nhỏ nhất.

I K H

M

P)

Q

Lời giải. GọiIlà giao điểm đường thẳngdvới mặt phẳng(P)và lấy điểmM ∈d, M 6=I

GọiH, K lầ lượt hình chiếu củaM lên(P)và giao tuyến∆của(P)và(Q)

Đặtφlà góc giữa(P)và(Q), ta cóφ=M KH\,

tanφ= HM

HK ≥

HM HI

Do (Q)là mặt phẳng qua dvà vng góc với mặt phẳng (M HI), nên(Q)đi quaM

(134)

Chú ý. Ta giải tốn phương pháp đại số sau:

• Gọi~n= (a;b;c), a2+b2+c2 >0là VTPT mặt phẳng(Q) Khi đó~n·~u d = 0,

từ ta rút đượcatheob, c(hoặcbtheoa, choặcctheoa, b)

• Gọiφlà góc giữa(P)và(Q), ta có

cosφ= |~n·~nP|

|~n| · |~nP|

=f(t),

vớit= b

c, c6= Khảo sátf(t)ta tìm đượcmaxcủaf(t)

Bài tốn 11 Trong không gianOxyz, cho hai đường thẳngd d0 chéo Viết phương

trình mặt phẳng(P)chứadvà tạo vớid0 một góc lớn nhất.

d0

d M

A

H K P)

Lời giải. Trên đường thẳngd, lấy điểmM dựng đường thẳng∆đi quaM song song vớid0.

Khi góc giữa∆và(P)chính góc giữad0và(P).

Trên đường thẳng ∆, lấy điểmA GọiH K hình chiếu Alên(P)vàd, φ

góc giữa∆và(P)

Khi đóφ=AM H\

cosφ= HM

AM ≥

KM AM

Suy ra(P)là mặt phẳng chứadvà vng góc với mặt phẳng(AM K) Do đó(P)đi quaM

nhận(~ud∧~ud0)∧~udlàm VTPT

Chú ý. Ta giải toán phương pháp đại số sau:

• Gọi~n= (a;b;c), a2+b2+c2 >0là VTPT mặt phẳng(P) Khi đó~n·~u d = 0,

từ ta rút đượcatheob, c(hoặcbtheoa, choặcctheoa, b)

• Gọiφlà góc giữa(P)vàd0, ta có

sinφ= |~n·~ud0|

|~n| · |~ud0| =f(t),

vớit= b

(135)

3 Một số ví dụ

Ví dụ (Thi thử lần 3, THPT Kim Liên - Hà Nội, 2019) Trong không gian Oxyz , cho

hai điểm M(−2;−2; 1), A(1; 2;−3)và đường thẳng d : x+ =

y−5 =

z

−1 Gọi

đường thẳng quaM, vng góc với đường thẳngd, đồng thời cách điểmAmột khoảng bé nhất.

Khoảng cách bé là

A. √29 B. C. D. √34

9

h d

∆ M A

H

Lời giải. Bài toán yêu cầu tìm khoảng cáchh nhỏ từA đến đường thẳng∆, nên ta nghĩ

đến việc so sánh khoảng cách h với AM, nhiên ta có h ≤ AM, ta khơng sử dụng

được đánh giá này!

Dựa vào điều kiện đường thẳng∆(đi quaM vng góc vớid) ta có được∆ln nằm

trên mặt phẳng(P)đi quaM vng góc vớid, mặt phẳng(P)hồn tồn xác định

h≥d(A,(P)) Từ đó, ta cóminh= d(A,(P))

Ta có(P) : 2x+ 2y−z+ = Suy raminh= d(A,(P)) = |2p·+2·2−(−3) + 9|

22+ 22+ (−1)2 =

Chọn đáp ánB.

Ví dụ (Đề tham khảo lần 3, năm 2017) Trong không gian với hệ tọa độ Oxyz, cho mặt

phẳng(P) :x−2y+ 2z−3 = 0và mặt cầu(S) :x2+y2+z2+ 2x−4y−2z+ = 0 Giả

sử M ∈ (P) N ∈(S)sao cho véctơM N~ cùng phương với véctơ~u(1; 0; 1)và khoảng cách

giữaM N lớn TínhM N.

(136)

N1

I

K

N

M H

Lời giải. Gọiφlà góc giữa~uvà~nP, ta có

cosφ = ~u·~nP

|~u| · |~nP| =

3√2 =

2

GọiHlà hình chiếu củaN lên mặt phẳng(P), đócosM N H\ = √1

2 nên

M N = N H

cosM N H\ =

2N H

Suy raM N lớn khiN H lớn Mà

maxN H =R+ d(I,(P)) = + =

Do đómaxM N = 3√2 Chọn đáp ánC.

Ví dụ (SGD Sóc Trăng 2018) Trong không gian với hệ tọa độOxyz, cho mặt phẳng(P) : x+

y+z−1 = 0và hai điểmA(1;−3; 0),B(5;−1;−2) ĐiểmM(a;b;c)nằm trên(P)|M A−

M B|lớn Giá trị tícha·b·cbằng

A. B. 12 C. 24 D. −24

Lời giải. Ta cóA, B nằm hai phía mặt phẳng(P) GọiB0 là điểm đối xứng củaB qua

(P) Theo toán (3) ta suy ra|M A−M B|lớn khiM giao điểm củaAB0 và mp(P).

Phương trình đường thẳngBB0

    

x= +t y=−1 +t z =−2 +t

GọiHlà giao điểm củaBB0 mp(P) Suy raH

14 ;−

4 3;−

7

(137)

Do H trung điểm BB0 nên B0 13

3 ;− 3;−

8

3 Ta có AB~ = 10

3 ; 3;−

8

3 , suy

phương trình đường thẳngAB0là

    

x= + 5t y=−3 + 2t z =−4t

Tọa độ điểmM(6;−1;−4), suy raa·b·c= 24 Chọn đáp ánC.

Ví dụ Cho điểmA(1; −1; 2), B(2; 0; 1)và mặt phẳng(P) : 2x−y−z+3 = 0.Gả sử M(x0;y0;z0)là điểm thuộc(P)sao choM A+M Bcó giá trị nhỏ TínhT = 3x0+y0+z0

A. T = B. T = 10 C. T = D. T = 165

Lời giải. Ký hiệu f = 2x−y−z + ta có f(A) = + 1−2 + = > 0, f(B) = 4−1−1 + = 5>0.Vì điểmA, B nằm phía so với(P)

Gọi A0 đối xứng với Aquan(P) Khi theo tốn (3) ta cóM là giao điểm củaA0B với

(P)

GọiH(x; y; z)là hình chiếu điểmAtrên mặt phẳng(P)

Ta cóAH~ (x−1; y+ 1; z−2)và

(

~

AH =t·~n(P)

H ∈(P) nên tọa độH thỏa mãn

  

x−1 =

y+

−1 =

z−2

−1 2x−y−z+ =

⇒H

−1

3; − 3;

Tọa độA0

−5 3; 3; 10

DoA~0B =

3(11; −1; −7)nênA

0B : x−2

11 =

y

−1 =

z−1

−7

Từ ta tìm tọa độ điểmM làM

−1 5; 5; 12

.Suy raT = 2.Chọn đáp ánA.

Ví dụ (Thi thử lần 1, Trường Chuyên Lương Thế Vinh, Đồng nai 2018) Trong không gian Oxzy, cho bốn điểmA(−4;−1; 3), B(−1;−2;−1), C(3; 2;−3)D(0;−3;−5) Gọi(α)

mặt phẳng qua D và tổng khoảng cách từ A, B, C đến (α) lớn nhất, đồng thời ba điểm

A, B, C nằm phía so với(α) Trong điểm sau, điểm thuộc mặt phẳng(α)

A. E1(7;−3;−4) B. E2(2; 0;−7) C. E3(−1;−1;−6) D. E4(36; 1;−1)

Lời giải. Theo kết tốn (6) ta có

(138)

Trong đóG −2

3;− 3;−

1

3 trọng tâm ba điểmA, B, C Do đó(P)là mặt phẳng qua

Dvà nhậnDG~ =

−2

3; 3;

14

làm VTPT Nên phương trình(α) :x−4y−7z−47 =

Chọn đáp ánA.

Ví dụ Trong khơng gian Oxyz, cho ba điểmA(1; 2; 3), B(−3; 4;−1) C(2; 0;−2) Gọi (P)là mặt phẳng quaC và tổng khoảng cách từAB đến(P)lớn Tính khoảng cách

htừ gốc tọa độOđến mặt phẳng(P).

A. h= √4

3 B. h=

1

3 C. h=

3 D. h= √2

3

Lời giải. Ta xét hai trường hợp

• AvàB nằm phía so với(P) Khi

d(A,(P)) + d(B,(P)) = 2d(M,(P))≤2M C = 6√3,

trong đóM(−1; 3; 1)là trung điểmAB

• AvàB nằm khác phía so với(P) Khi

d(A,(P)) + d(B,(P)) = d(A,(P)) + d(B0,(P)) = 2d(N,(P))≤2N C = 6,

trong đóB0(7;−4;−3)là điểm đối xứng vớiB quaC và N(4;−1; 0) là trung điểm của AB0.

Từ ta có (P) mặt phẳng qua C nhận véctơM C~ = (3;−3;−3)làm VTPT Suy

phương trình(P) : x−y−z−4 = 0.Do đóh= √4

3 Chọn đáp ánD.

Ví dụ (Đề TT lần 2, Ngơ Quyền, Hải Phịng 2018) Cho mặt phẳng(α) : ax+by+cz+d= 0,(a2+b2+c2 >0)đi qua hai điểmB(1; 0; 2), C(5; 2; 6)và cáchA(2; 5; 3)một khoảng lớn

nhất Khi giá trị biểu thứcT = a

b+c+d

A.

4 B.

1

6 C.

1

6 D. −2

A

H

I B

(139)

Lời giải. Theo kết toán (7), ta có (α) mặt phẳng qua B nhận véctơ AI~ làm

VTPT, vớiI hình chiếu vng góc củaAlên đường thẳngBC

Phương trình đường thẳngBC :

    

x= + 2t y =t z = + 2t

GọiIlà hình chiếu củaAtrênBCsuy raI(3; 1; 4)

Phương trình mặt phẳng(P)làx−4y+z−3 =

VậyT = a

b+c+d =−

1

6 Chọn đáp ánC.

Ví dụ (GHK2, THPT Nghèn - Hà Tĩnh, 2019) Trong không gian tọa độOxyz, cho điểm A(1; 1; 2), B(0;−1;−3) Xét điểm thay đổi mặt phẳng(Oxz), giá trị nhỏ của

P =OM~ + 2M A~ + 3M B~ bằng

A. B.

2 C.

1

2 D.

1

Lời giải. GọiI điểm thỏa mãn ~

OI+ 2IA~ + 3IB~ =~0

Ta cóI

1 2;−

1 4;−

5

Khi theo tốn (8), ta có

OM~ + 2M A~ + 3M B~ = 4M I

Do đóP đạt giá trị nhỏ khiM hình chiếu vng góc củaI lên(Oxz)

Theo đóM

1 2; 0;−

5

Khi đóminP = 4·d (I; (Oxz)) = 4IM = 4·

−14

= Chọn đáp ánA.

Ví dụ (Đề thi thử THPTQG, 2018, SGD Phú Thọ) Trong không gian Oxyz, cho mặt

phẳng (P) : 3x−3y+ 2z−15 = 0và ba điểm A(1; 2; 0), B(1;−1; 3), C(1;−1;−1) Điểm

M(x0;y0;z0)thuộc(P)sao cho2M A2−M B2+M C2nhỏ Giá trị2x0+ 3y0+z0 bằng

A. 11 B. 15 C. D. 10

Lời giải. GọiG(a;b;c)là điểm thỏa mãn2GA~ −GB~ +GC~ =~0 Ta cóG(1; 2;−2)

Khi theo tốn (9), ta cóM hình chiếu củaGlên(P)

Phương trình đường thẳng quaGvà vng góc với(P)là x−1 =

y−2

−3 =

(140)

Xét hệ

  

x−1 =

y−2

−3 =

z+ 2 3x−3y+ 2z−15 =

⇔     

x+y= 2x−3z = 3x−3y+ 2z = 15

⇔     

x=

y=−1

z =

VậyM(4;−1; 0)và giá trị biểu thức cần tìm bằng2·4 + 3·(−1) + =

Chọn đáp ánC.

Ví dụ 10 (Đề tập huấn tỉnh Lai Châu,2019) Trong không gian với hệ trục tọa độOxyz, cho

đường thẳng ∆ : x−1 =

y−1 =

z

2 và mặt phẳng (α) : x−2y+ 2z −5 = 0 Gọi (P)

mặt phẳng chứavà tạo với mặt phẳng(α)một góc nhỏ Phương trình mặt phẳng(P)

dạngax+by+cz+d = 0(với a, b, c, d∈ Z a, b, c, d ∈ [−5; 5]) Khi tíchabcd bằng

bao nhiêu?

A. 120 B. 60 C. −60 D. −120

Lời giải. Theo kết tốn (10) thì(P)là mặt phẳng quaM(1; 1; 0)và nhận

~n= (~nα∧~u∆)∧~u∆= (−8; 20;−16)

làm VTPT Suy phương trình(P) : 2x−5y+ 4z+ =

Từ đó, ta cóa= 2,b=−5,c= 4,d= 3nênabcd=−120 Chọn đáp ánD.

Ví dụ 11 Chod: x−1 =

y+ 2 =

z

−1 d

0 : x+

2 =

y−1

−1 =

z

2.Gọi(P)là mặt phẳng

chứa đường thẳng (P)và góc mặt phẳng (P)và đường thẳng d0 lớn Tọa độ giao

điểm của(P)và trụcOy

A. (0; 3; 0) B. (0; 9; 0) C. (0;−9; 0) D. (0;−3; 0)

Lời giải. Theo kết tốn (11), ta có(P)là mặt phẳng qua M(1;−2; 0) nhận

vec tơ

~n= (~ud∧~u0d)∧~ud = (14;−2; 10)

làm VTPT Suy phương trình(P) : 7x−y+ 5z−9 =

Từ ta tìm giao điểm của(P)vàd0 là(0;−9; 0) Chọn đáp ánC.

Ví dụ 12 (KSCL L4, Yên Lạc - Vĩnh Phúc, 2019) Trong không gian tọa độOxyz, cho mặt cầu

(S) : (x−3)2+ (y−2)2+z2 = 4và hai điểmA(−1; 2; 0),B(2; 5; 0) GọiK(a;b;c)là điểm

thuộc(S)sao choKA+ 2KBnhỏ Giá trịa−b+cbằng

(141)

Lời giải. Mặt cầu(S)có tâmI(3; 2; 0), bán kínhR=

Vì cần đánh giá tổng KA + 2KB, nên ta tìm cách dựng điểm M cho KA = 2KM ⇔ KA

KM = 2khiK thay đổi trên(S)

Ta thấy IK = R = IA = 4, nên IA

IK = = KA

KM Điều gợi ý ta xét hai tam giác IAK vàIKM đồng dạng với Do doạnAI ta lấyM choIM = Khi hai

tam giácIAKvàIKM có gócI chung IA

IK = = IK

IM, nên hai tam giác đồng dạng với

nhau

Ta tìm đượcM(2; 2; 0) Khi

KA+ 2KB = 2(KM +KB)≥2M B

Hơn nữa, dễ thấyB nằm mặt cầu(S)và M nằm mặt cầu(S), nên ta có dấu

xảy khiKlà giao điểm đoạn thẳngM Bvới mặt cầu(S)

Phương trình củaM B :

    

x=

y= + 3t z =

, suy raK(2; + 3t; 0)

K ∈(S)⇒1 + (9(1 +t)2 = 4 ⇔t=−1± √1

3 ⇒K(2; 2−

3; 0)vàK(2; +√3; 0)

DoKnằm giữaB, M nênK(2; +√3; 0)⇒a−b+c=−√3 Chọn đáp ánB.

Ví dụ 13 (THPT QUỐC GIA 2018 - 101) Trong không gianOxyz, cho mặt cầu(S)có tâm

I(−2; 1; 2) và qua điểmA(1;−2;−1) Xét điểmB, C, Dthuộc(S)sao cho AB, AC, ADđôi vng góc với Thể tích khối tứ diệnABCDcó giá trị lớn bằng

A. 72 B. 216 C. 108 D. 36

D P

A

B E

I

M N

C a

b c

Lời giải. ĐặtAB =a,AC =b,AD=cthìABCDlà tứ diện vuông đỉnhA, nội tiếp mặt cầu

(S)

(142)

XétV =VABCD =

6abc⇔V =

36a

2b2c2 Mà

a2+b2+c2 >3√3 a2b2c2 ⇔

a2+b2+c2

3

>a2b2c2

4R2

3

>36·V2 ⇔V 6R3·

3 27

VớiR =IA = 3√3 VậyVmax= 36 Chọn đáp ánD.

Ví dụ 14 (TT, THPT Nghèn, Hà Tĩnh, lần 2, 2018) Trong không gian Oxyz, cho mặt cầu

(S)có phương trình x2 +y2 +z2 = 8và điểmM

2 ;

2 ;

!

Đường thẳngd thay đổi đi

qua điểmM, cắt mặt cầu(S)tại hai điểm phân biệtA,B Tính diện tích lớn nhấtSmaxcủa tam

giácOAB.

A. Smax= B. Smax=

7 C. Smax=

7 D. Smax =

2

Lời giải. (S)có tâmO(0; 0; 0)và có bán kínhR= 2√2

Gọitlà khoảng cách từ tâmO đến đường thẳngd(t≤OM = 1) Diện tích tam giácOAB S =

2t·AB =t

R2−t2 =t√8−t2 = √1

7

7t·√8−t2 ≤ 7t

2+ 8−t2 2√7 = 6t

2 + 8 2√7 ≤

14 2√7 =

7

Dấu xảy khidvuông góc vớiOM Chọn đáp ánC.

Ví dụ 15 (Thi thử, Chuyên Phan Bội Châu - Nghệ An, 2019-L1) Trong không gianOxyz, cho

hai điểmB(2;−1;−3)C(−6;−1; 3) Trong tam giácABC thỏa mãn đường trung

tuyến kẻ từB Cvng góc với nhau, điểmA(a;b; 0),(b >0)sao cho gócAlớn nhất, giá trị

của a+b

cosA bằng

A. 10 B. −20 C. 15 D. −5

Lời giải. GọiGlà trọng tâm tam giácABC Ta có

GB ⊥GC ⇔GB2+GC2 =BC2 ⇔AB2 +AC2 = 5BC2

Khi

cosA= AB

2+AC2−BC2 2AB·AC =

4BC2 2AB·AC ≥

4BC2

AB2 +AC2 = 4BC2 5BC2 =

4

Do gócAlớn khicosA=

5 ⇔AB =AC =

(143)

Ta có hệ phương trình

(

(a−2)2+ (b+ 1)2+ = (a+ 6)2+ (b+ 1)2+ 9 (a−2)2+ (b+ 1)2+ = 250 ⇔

(

a=−2

b= 14 (vìb >0)

Vậy a+b

cosA = 15 Chọn đáp ánC.

Ví dụ 16 (Đề thức THPTQG 2019, Mã đề 101) Trong khơng gian Oxyz, cho điểm A(0; 4;−3) Xét đường thẳng d thay đổi, song song với trụcOz và cách trụcOz một khoảng

bằng3 Khi khoảng cách từAđếndnhỏ nhất,dđi qua điểm đây?

A. P(−3; 0;−3) B. M(0;−3;−5) C. N(0; 3;−5) D. Q(0; 5;−3)

y

z x

O

−3

3 d

A

Lời giải. Ta có đường thẳng d thay đổi, song song với trục Oz cách trục Oz khoảng

bằng3nêndnằm mặt trụ trịn xoay có trục làOzvà bán kính bằng3

Ta có

d(A;d)≥ |d(A;Oz)−d(d;Oz)|=

và dấu xảy khid, trụcOzvà điểmAđồng phẳng

Do đód(A;d)đạt nhỏ bằng1khi đường thẳngdnằm mặt phẳng(Oyz)và cáchOz

một khoảng là3nên có phương trình làd:

    

x=

y=

z =t

Trong bốn điểm M(0;−3;−5), N(0; 3;−5), P(−3; 0;−3), Q(0; 5;−3) đường thẳng d

qua điểmN(0; 3;−5) Chọn đáp ánC.

Ví dụ 17 (Thi thử, Sở GD ĐT Lạng Sơn, 2019) Trong không gian với hệ trục tọa độOxyz

cho điểm M(−2;−2; 1), A(1; 2;−3) và đường thẳng d: x+ =

y−5 =

z

−1 Trong các

vectơ −→u cho đây, đâu vectơ phương đường thẳngđi quaM vng góc

(144)

A. −→u(1; 0; 2) B. −→u(2; 1; 6) C. →−u(−1; 0; 2) D. −→u(2; 2;−1)

Lời giải. Gọi (P) mặt phẳng qua M vng góc vớid Ta có (P) qua (M), nhận

véc-tơ phương củadlà(2; 2;−1)làm véc-tơ pháp tuyến nên(P)có phương trình: 2(x+ 2) + 2(y+ 2)−z(−1) = 0⇔2x+ 2y−z+ =

GọiH,Klần lượt hình chiếu vng góc củaAlên mặt phẳng(P)và đường thẳngd Ta H(−3;−2;−1) Ta cód(A,(∆)) =AK ≤AH Xảy dấu bằng⇔K ≡H Vậy đường thẳng

∆là đường thẳngAHcó véc-tơ phương−−→AH = (1; 0; 2) Chọn đáp ánA.

Ví dụ 18 (Đề tập huấn số 2, Sở GD ĐT Quảng Ninh, 2019) Trong không gian với hệ tọa

độ Oxyz, cho bốn điểm A(3; 0; 0), B(0; 2; 0), C(0; 0; 6) D(1; 1; 1) Gọilà đường thẳng

đi qua Dvà thỏa mãn tổng khoảng cách từ điểmA, B,C đếnlà lớn nhất, hỏiđi qua

điểm điểm đây?

A. M(−1;−2; 1) B. M(5; 7; 3) C. M(3; 4; 3) D. M(7; 13; 5)

Lời giải. Phương trình mặt phẳng(ABC)là x 3+

y

2+

z

6 = 1hay(ABC) : 2x+ 3y+z−6 =

Dễ thấyD ∈(ABC)

GọiH,K,Ilần lượt hình chiếu vng góc củaA,B,C trên∆ Do∆là đường thẳng qua

DnênAH ≤AD,BK ≤BD,CI ≤CD Khi

AH+BK+CI ≤AD+BD+CD

Vậy để tổng khoảng cách từ điểm A,B,C đến∆lớn thì∆là đường thẳng quaDvà

vng góc với mặt phẳng(ABC)

Vậy phương trình đường thẳng∆là

    

x= + 2t y= + 3t z = +t

(t∈R)Ta thấyM(5; 7; 3)∈∆

Chọn đáp ánB.

Ví dụ 19 (GHK2, Nguyễn Đình Chiểu-Tiền Giang, lần 1, 2019) Trong không gianOxyz, cho

các điểm A(−2; 1; 2), B(2; 1;−2) C(1; 1; 1) Gọi dlà đường thẳng qua C sao cho tổng

khoảng cách từABđếndlớn Giao điểm củadvới mặt phẳng(P) : 2x+y+z = 0

tọa độ là A. 1;−1

10;

B. (1; 3; 1) C. (1;−3; 1) D. 1; 10;

(145)

A B C

H

K

Lời giải. Ta cóAC = √10,BC = √10 Gọi∆là đường thẳng quaC, gọiH,K theo

thứ tự hình chiếu củaAvàB lên∆ Ta có (AH+BK)2 ≤ AH2+BK2

2 =

20−(CH2+CK2)

2 ,

do đóAH+BK lớn khiCH2+CK2 nhỏ MàCH2+CK2 ≥0nênCH2+CK2

nhỏ khiHvàKtrùng vớiC, đó∆vng góc với mặt phẳng(ABC) Vậy đường thẳng

dcần tìm đường thẳng qua Cvà vng góc với mặt phẳng(ABC) Ta có−→AB = (4; 0;−4),

−→

AC = (3; 0;−1) Suy rah−→AB,−→ACi = (0;−8; 0)là véc-tơ phương củad

Phương trình củadlà 

   

x=

y = + 8t z =

Suy giao điểm củadvà mặt phẳng(P)là điểmM(1;−3; 1) Chọn đáp ánC.

Qua ví dụ trên, hy vọng em có số kỹ cách tiếp cận gặp toán cực trị không gian tọa độOxyz

4 Bài tập

Bài 1(TT, Lê Xoay, Vĩnh Phúc, 2018, L3). Trong không gian với hệ tọa độOxyz, cho hai điểm A(−2; 2;−2), B(3;−3; 3) ĐiểmM không gian thỏa mãn M A

M B =

2

3 Khi độ dàiOM

lớn

A. 6√3 B. 12√3 C. 5√23 D. 5√3

Bài 2(Đề TT lần 1, Chuyên Nguyễn Thị Minh Khai, Sóc Trăng 2018). Trong không gianOxyz,

cho điểmA(1; 2;−1)và mặt phẳng(P) : x+y+ 2z −13 = Xét mặt cầu (S)có tâm

I(a;b;c)đi qua điểmA, tiếp xúc với mặt phẳng(P) Tính giá trị biểu thứcT =a2+2b2+3c2

(146)

A. T = 35 B. T = 20 C. T = 25 D. T = 30

Bài 3 (Thi thử L6, Đại Học Ngoại Thương Hà Nội, 2018). Trong không gian với hệ tọa độ

Oxyz, cho ba điểmA(a; 0; 0), B(0;b; 0), C(0; 0;c)vớia, b, c số thực dương thay đổi

sao choa2+b2+c2 = 3 Tính khoảng cách lớn từOđến mặt phẳng(ABC).

A.

3 B. C.

1

3 D.

Bài 4(HK2 (2017-2018), Sở Giáo Dục Lâm Đồng). Trong không gianOxyz, cho hai mặt cầu

(S1), (S2)có phương trình là(x−2)2+ (y−1)2+ (z−1)2 = 16và(x−2)2+ (y− 1)2+ (z−5)2 = Gọi(P)là mặt phẳng thay đổi tiếp xúc với hai mặt cầu(S1), (S2) Tính

khoảng cách lớn từ gốc tọa độO đến mặt phẳng(P)

A.

15 B. √15 C. 9+√215 D. 8√3+2 √5

Bài 5(Đề tập huấn số 2, Sở GD ĐT Quảng Ninh, 2019). Trong không gian với hệ tọa độ

Oxyz, gọidlà đường thẳng qua điểmA(1;−1; 2), song song với(P) : 2x−y−z+ = 0,

đồng thời tạo với đường thẳng∆ : x+ 1 =

y−1

−2 =

z

2 góc lớn Phương trình đường

thẳngdlà

A. x−1

1 =

y+

−5 =

z−2

B. x−1

4 =

y+

−5 =

z+

C. x−1

4 =

y+ =

z−2

D. x−1

1 =

y+

−5 =

z−2

−7

Bài 6(TT L3,Minh Châu,Hưng Yên,1718). Trong không gianOxyz, cho hai điểmA(1; 2;−1),

B(0; 4; 0)và mặt phẳng(P) : 2x−y−2z+ 2018 = Gọi(Q)là mặt phẳng qua hai điểm

A,B vàαlà góc nhỏ hai mặt phẳng(P)và(Q) Giá trị củacosαlà

A. cosα=

6 B. cosα=

3 C. cosα =

9 D. cosα=

3 Bài 7 (KSCL, Sở GD ĐT - Thanh Hóa, 2018). Trong khơng gian với hệ tọa độOxyz, cho

các mặt cầu (S1), (S2), (S3) có bán kính r = có tâm điểmA(0; 3;−1),

B(−2; 1;−1), C(4;−1;−1) Gọi(S)là mặt cầu tiếp xúc với ba mặt cầu Mặt cầu(S)

có bán kính nhỏ

A. R = 2√2 B. R =√10−1 C. R=√10 D. R= 2√2−1 Bài 8 (KSCL, Sở GD ĐT - Thanh Hóa, 2018). Trong khơng gian với hệ tọa độOxyz, cho

bốn điểmA(7; 2; 3), B(1; 4; 3), C(1; 2; 6), D(1; 2; 3)và điểmM tùy ý Tính độ dài đoạnOM

khi biểu thứcP =M A+M B+M C +√3M D đạt giá trị nhỏ

A. OM =√26 B. R =√10−1 C. OM = 5√417 D. 3√421

Bài 9(Đề thi thử - Trường THPT chuyên Lương Thế Vinh - Đồng Nai - Lần - 2018). Trong không gianOxyz,cho hai điểmA(1; 0; 1),B(0; 1;−1) Hai điểmD, Ethay đổi đoạn OA, OBsao cho đường thẳngDEchia tam giácOAB thành hai phần có diện tích

(147)

A. I 42; 42; B. I 32; 32; C. I 14; 4;

D. I 13; 13; Bài 10(Đề thi thử trường THPT Sơn Tây - Hà Nội, 2018). Cho hai mặt cầu(S1) : (x−3)2 + (y−2)2 + (z−2)2 = 4và (S

2) : (x−1)2 +y2 + (z−1)2 = Gọidlà đường thẳng đồng

thời tiếp xúc với hai mặt cầu trên, cắt đoạn thẳng nối tâm hai mặt cầu cách gốc tọa độ khoảng lớn Nếu~u = (a; 1;b)là véc-tơ phương củadthì tổngS = 2a+ 3b

bao nhiêu?

A. S = B. S = C. S = D. S=

Bài 11 (Thi thử L4, THPT chuyên ĐHSP Hà Nội, 2018). Cho mặt cầu (S) có phương trình

x2+y2+z2−2x+ 4y+ 2z+ = 0 Đường thẳngdđi quaOvà cắt mặt cầu hai điểm phân

biệtA, B Giá trị lớn củaOA+OB

A. 3√6 B. 2√3 C. 2√6 D. √6

Bài 12(Đề khảo sát chất lượng ,THPT Hàm Rồng, Thanh Hóa 2018). Trong khơng gian với hệ tọa độ Oxyz, choA(m; 0; 0), B(0; 2m + 1; 0), C(0; 0; 2m + 5)khác O D điểm nằm

khác phía với O so với mặt phẳng (ABC) cho tứ diện ABCD có cặp cạnh đối diện

bằng Tìm khoảng cách ngắn từOđến tâmI mặt cầu ngoại tiếp tứ diệnABCD

A. √11 B. √10 C. √6 D. √10

2

Bài 13(Thi thử L1, THPT Hậu Lộc 2, Thanh Hố, 2019). Trong khơng gian với toạ độOxyz,

cho hai điểmA(0; 0;−2)và B(3; 4; 1) Gọi(P)là mặt phẳng chứa đường tròn giao tuyến

hai mặt cầu(S1) : (x−1)2+ (y−1)2+ (z+ 3)2 = 25và(S2) : x2+y2+z2−2x−2y−14 =

M, N hai điểm thuộc(P)sao choM N = Giá trị nhỏ củaAM +BN

A. √34−1 B. C. √34 D.

Bài 14(Thi Thử L1, Trường THPT Phụ Dực- Thái Bình, 2019 ). Trong không gianOxyz, cho

mặt cầu(S) :x2 + (y−3)2+ (z−6)2 = 45vàM(1; 4; 5) Ba đường thẳng thay đổid

1,d2,d3

nhưng đơi vng góc tạiOcắt mặt cầu điểm thức hai làA,B, C Tính khoảng

cách lớn từM đến mặt phẳng(ABC)là

A. B. √5 C. D. √6

Bài 15(Thi Thử L4, Chuyên Phan Bội Châu, Nghệ An, 2018). Trong không gian tọa độOxyz

choA(1; 3; 10),B(4; 6; 5)vàM điểm thay đổi mặt phẳng(Oxy)sao choM A, M Bcùng

tạo với mặt phẳng(Oxy)các góc Tìm giá trị nhỏ củaAM

A. 6√3 B. 10 C. √10 D. 8√2

Bài 16(Tập huấn, Sở GD ĐT lần 1, 2019). Trong không gianOxyz, cho hai điểmA(3;−2; 3),

B(1; 0; 5)và đường thẳngd: x−1 =

y−2

−2 =

z−3

2 Tìm tọa độ điểmM đường thẳngd

(148)

A. M(1; 2; 3) B. M(2; 0; 5) C. M(3;−2; 7) D. M(3; 0; 4) Bài 17(Đề GHK2, Hàm Rồng, Thanh Hóa, năm 2019). Trong khơng gian với hệ tọa độOxyz,

cho3 điểmA(1; 0; 1), B(3;−2; 0), C(1; 2;−2) Gọi (P) mặt phẳng qua Asao cho tổng

khoảng cách từB vàCđến mặt phẳng(P)lớn nhất, biết rằng(P)khơng cắt đoạnBC Khi

pháp tuyến mặt phẳng(P)là

A. ~n= (2;−2;−1)

B. ~n= (1; 0; 2)

C. ~n= (−1; 2;−1)

D. ~n= (1; 0;−2)

Bài 18 (Thi thử L1, Đức Thọ, Hà Tĩnh 2018). Trong không gian với hệ tọa độ Oxyz, cho

tứ diện ABCD có cặp cạnh đối diện D khác phía với O so với (ABC);

đồng thời A, B, C giao điểm trục tọa độOx, Oy, Oz với mặt phẳng(P) :

x

m +

y m+ +

z

m−5 = 1, m /∈ {0;−2;−5} Tính khoảng cách ngắn từ tâmI mặt

cầu ngoại tiếp tứ diệnABCDđếnO

A. √30 B. √13

2 C.

26 D. √26

Bài 19(Hàm Rồng - Thanh Hóa,lần - 2019). Trong khơng gian với hệ trụcOxyz, cho điểm A(1; 4; 3)và mặt phẳng(P) : 2y−z = Biết điểmB thuộc(P), điểmCthuộc(Oxy)sao cho

chu vi tam giácABCnhỏ Hỏi giá trị nhỏ

A. 4√5 B. 6√5 C. 2√5 D. √5

Bài 20 (Thi thử, Chuyên Lê Quý Đôn - Điện Biên, 2019). Trong không gian Oxyz, cho hai

điểm A(1; 2;−1), B(3; 0; 3) Biết mặt phẳng (P) qua điểm A cách B khoảng lớn

nhất Phương trình mặt phẳng(P)là

A. x−2y+ 2z+ =

B. x−y+ 2z+ =

C. 2x−2y+ 4z+ =

D. 2x−y+ 2z =

Bài 21(Thi thử L1, Chuyên Ngoại Ngữ, Hà Nội, 2018). Trong không gian với hệ tọa độOxyz,

cho mặt cầu(S) : (x−1)2+ (y−2)2+ (z−2)2 = 9hai hai điểmM(4;−4; 2),N(6; 0; 6) Gọi

E điểm thuộc mặt cầu(S)sao cho EM +EN đạt giá trị lớn Viết phương trình tiếp

diện mặt cầu(S)tạiE

A. x−2y+ 2z+ =

B. 2x+y−2z−9 =

C. 2x+ 2y+z+ =

D. 2x−2y+z+ =

Bài 22(KSCL (2017-2018) lần 4,Thanh Miện 2,Hải Dương). Trong không gian với hệ trục tọa độOxyz, cho điểmM(0;−1; 2)vàN(−1; 1; 3) Gọi(P)là mặt phẳng quaM, N tạo với

(149)

A.

3 B.

7

11 C.

3 D. 3

Bài 23(Đề thi thử THPTQG sở Bình Phước - lần - 2018). Trong không gian với hệ trục tọa độOxyz, cho hai mặt cầu(S1) :x2 +y2+z2 = 1, (S2) : x2+ (y−4)2 +z2 = 4và điểm

A(4; 0; 0),B

1 4; 0;

,C(1; 4; 0),D(4; 4; 0) GọiMlà điểm thay đổi trên(S1),N điểm thay

đổi trên(S2) Giá trị nhỏ biểu thứcQ=M A+ 2N D+ 4M N + 6BC

A. 2√265 B. 5√265

2 C.

265 D. 7√265

Bài 24. Trong không gianOxyz, cho ba điểmA(1; 1; 1),B(2; 1; 0), C(2; 0; 2) Gọi(α)là mặt

phẳng qua hai điểmB, C cáchAmột khoảng cách lớn Véc-tơ sau véc-tơ

pháp tuyến của(α)?

A. −→n(1; 0;−1)

B. −→n(5; 2;−1)

C. −→n(5;−2;−1)

D. −→n(5; 1;−2)

Bài 25 (Đề KSCL học kỳ Toán 12 năm học 2017 – 2018 sở GD ĐT Nam Định). Trong không gian với hệ tọa độOxyz, cho điểmA(1; 2;−3)và mặt phẳng(P) : 2x+ 2y−z+ =

Đường thẳng quaAvà vng góc với mặt phẳng(Q) : 3x+ 4y−4z+ = 0cắt mặt phẳng (P)tạiB ĐiểmM nằm mặt phẳng(P)sao choM ln nhìn đoạn thẳngAB

góc vng độ dàiM B lớn Tính độ dàiM B

A. M B =√5 B. M B = √25 C. M B = √241 D. M B =√41 Bài 26(Đề KSCL trường THPT chuyên Hùng Vương, Phú Thọ, năm 2018, lần 4). Trong không gian với hệ toạ độOxyz, cho mặt phẳng(P) :x−2y+z−1 = 0và điểmA(0;−2; 3),B(2; 0; 1)

ĐiểmM(a;b;c)thuộc(P)sao choM A+M B nhỏ Giá trị củaa2+b2+c2 bằng

A. 41

4 B.

9

4 C.

7

4 D.

Bài 27 (Đề KSCL Toán 12 THPT năm học 2017 – 2018 sở GD ĐT Thanh Hóa). Trong khơng gian với hệ trục tọa độ Oxyz, cho điểm A(2;−1;−2) đường thẳng (d) có phương

trình x−1

1 =

y−1

−1 =

z−1

1 Gọi (P) mặt phẳng qua điểm A, song song với đường

thẳng(d)và khoảng cách từ đường thẳng(d)tới mặt phẳng(P)là lớn Khi đó, mặt phẳng (P)vng góc với mặt phẳng sau đây?

A. x−y−z−6 =

B. x+ 3y+ 2z+ 10 =

C. x−2y−3z−1 =

D. 3x+z+ =

Bài 28 (2-GHK2-96-ThithuTHTT-Lan7). Trong không gian với hệ tọa độ Oxyz cho điểm A(3;−1; 0)và đường thẳngd: x−2

−1 =

y+ =

z−1

1 Mặt phẳng(α)chứadsao cho khoảng

(150)

A. x+y−z =

B. x+y−z−2 =

C. x+y−z+ =

D. −x+ 2y+z+ =

Bài 29(Đề thức THPTQG 2019, Mã đề 110). Trong không gianOxyz, cho điểmA(0; 4;−3)

Xét đường thẳngdthay đổi, song song với trụcOzvà cáchOz khoảng bằng3 Khi khoảng

cách từAđếndlớn nhất,dđi qua điểm đây?

A. P(−3; 0;−3) B. Q(0; 11;−3) C. N(0; 3;−5) D. M(0;−3;−5) Bài 30(Đề thức THPTQG 2019, Mã đề 103). Trong không gianOxyz, cho điểmA(0; 3;−2)

Xét đường thẳng d thay đổi, song song với trụcOz cách trục Oz khoảng Khi

khoảng cách từAđếndnhỏ nhất,dđi qua điểm đây?

(151)

M

Ở RỘNG BẤT ĐẲNG THỨC HÌNH HỌC

F

INSLER

-H

ADWIGER

Trần Quang Hùng, Hà Nội

T

ÓM TẮT

Trong [1] giới thiệu bất đẳng thức hình học Finsler-Hadwiger Bài viết coi tiếp nối [1] nhằm giới thiệu thêm vài mở rộng liên quan tới bất đẳng thức hình học quan trọng

1 Mở đầu

Bất đẳng thức hình học Finsler-Hadwiger phát biểu sau, xem [4]

Định lý 1(Finsler-Hadwiger, 1937). Cho tam giácABC có độ dài cạnhBC =a,CA=b, AB =cvà số diện tích làS Khi ta có bất đẳng thức hình học sau

a2+b2+c2 ≥4√3S+ (b−c)2+ (c−a)2+ (a−b)2

Theo [4], bất đẳng thức hình học Finsler-Hadwiger coi mở rộng trực tiếp cho bất đẳng thức hình học Weitzenbăock

nh lý 2(Weitzenbăock). Cho tam giỏcABC cú dài cạnhBC = a,CA = b,AB =c

và số diện tích làS Khi ta có bất đẳng thức hình học sau a2+b2+c2 ≥4√3S

Điều hiển nhiên đại lượng

Q= (b−c)2+ (c−a)2+ (a−b)2 ≥0

(152)

được biết tới hai ông xuất báo định lý Finsler-Hadwiger, định lý túy hình học đề cập đến hình vng xuất phát từ hai hình vng khác có chung đỉnh Trong hình học phẳng, việc đánh giá tương quan cạnh tam giác diện tích quan trọng Trong việc đánh giá diện tích với bình phương cách cạnh coi đánh giá kinh điển đại lượng thứ nguyên 2, đặc trưng mặt phẳng Vì vy, hai bt ng thc hỡnh hc ca Weitzenbăock v Finsler-Hadwiger dấu mốc quan trọng bất đẳng thc hỡnh hc Trong ú, bt ng thc ca Weitzenbăock đánh giá sơ khai bất đẳng thức Finsler-Hadwiger tổng quát Tổng quát nói "mạnh" có đại lượngQ

Theo dịng lịch sử phát triển bất đẳng thức hình học Finsler Hadwiger, năm bất đẳng thức cơng bố, sau có nhiều phát triển mở rộng liên quan đời Vào tháng năm 2008, tác giả viết có đưa mở rộng cho bất đẳng thức này, toán xuất năm 2012, dạng đề tốn tạp chí Tốn học tuổi trẻ, xem [2]

Định lý 3 (Mở rộng bất đẳng thức Finsler Hadwiger dùng trọng tâm). Cho tam giácABC nội

tiếp đường trịn(O, R), trọng tâmG, diện tíchS Khi

a2+b2 +c2 ≥

4√3 + OG

R2

S+ (a−b)2 + (b−c)2+ (c−a)2

Sau nghiên cứu toán này, tác giả Nguyễn Văn Quý đưa mở rộng khác tương tự sau

Định lý 4 (Mở rộng bất đẳng thức Finsler Hadwiger dùng tâm đường tròn nội tiếp). Cho tam giácABCnội tiếp đường trịn(O, R), ngoại tiếp đường trịn(I, r), diện tíchS Khi

a2+b2+c2 ≥

4√3 + OI

R2

S+ (a−b)2+ (b−c)2 + (c−a)2

Sau đây, để sâu chứng minh hai mở rộng này, đưa số cơng thức hình học thơng dụng đại lượng tam giác

Định lý 5. Cho tam giácABCcó độ dài cạnhBC =a,CA=b,AB =c Diện tíchS, bán

kính đường trịn ngoại tiếpR, bán kính đường trịn nội tiếpr, nửa chu vip Khi

1) ab+bc+ca=p2+r2 + 4Rr.

2) a2+b2+c2 = 2(p2−r2 −4Rr).

3) Q= (b−c)2+ (c−a)2+ (a−b)2 =p2−3r2−12Rr.

4) a2+b2+c2−Q= 2(ab+bc+ca)−(a2+b2+c2) = 4r(4R+r).

5) a2+b2+c2−Q

S =

2(ab+bc+ca)−(a2+b2+c2)

4S = tan A

2 + tan B

2 + tan C

2

(153)

2 Các chứng minh

Trong mục chúng tơi trình bày chứng minh chi tiết cho Định lý Định lý Để đưa lời giải cho Định lý 3, xin giới thiệu chứng minh hai bổ đề

Bổ đề 1. Cho tam giácABC, ta có bất đẳng thức lượng giác sau

1

4(sinA+ sinB + sinC) + tan

A

2 + tan

B

2 + tan

C

2 ≥ 3√3

8 +

3

Chứng minh. Ta xét hàmf(x) = 14sinx+ tanx2, ∀x∈(0, π), ta có

f00(x) = −1

4sinx+ 2tan

x

2(1 + tan x

2)

Đặtt= tanx2 >0, ta suy

f00(x) = −2t 4(1 +t2) +

t

2(1 +t

2) = t[(1 +t2)2−1]

2(1 +t2) ≥0, ∀t >0

Do đóf(x)lồi trên(0, π), sử dụng bất đẳng thức Jensen áp dụng vào tam giácABCta có

f(A) +f(B) +f(C)≥3f

A+B+C

3

hay

1

4(sinA+ sinB + sinC) + tan

A

2 + tan

B

2 + tan

C

2 ≥ 3√3

8 +

3

Đó điều phải chứng minh

Bổ đề 2. Cho tam giácABC nội tiếp đường trịn(O, R), có trọng tâmG Thì

3√3

2 −(sinA+ sinB+ sinC)≥

OG2

R2

Chứng minh. Áp dụng bất đẳng thức Cauchy-Schwarz ta dễ thấy

27

4 −(sinA+ sinB+ sinC)

2 ≥ 27

4 −3(sin

2A+ sin2B+ sin2C) = 3(9 4−

a2+b2+c2

4R2 ) (1)

Ta có hệ thức bảnOG2 =R2−a2+b2+c2

9 (2)

Mặt khác dosinA+ sinB + sinC ≤ 3√3 nên

3√3[3

3

2 −(sinA+sinB+sinC)]≥[ 3√3

2 +(sinA+sinB+sinC)][ 3√3

(154)

= 27

4 −(sinA+ sinB+ sinC)

2. (3)

Từ(1),(2),(3), ta suy

3√3[3

3

2 −(sinA+ sinB+ sinC)]≥

27OG2 4R2

Vậy

3√3

2 −(sinA+ sinB+ sinC)≥

3√3OG2 4R2 ≥

OG2

R2

Đó điều phải chứng minh

Hai bổ đề sử dụng hai bất đẳng đại số bất đẳng thức Cauchy-Schwarz bất đẳng thức Jensen Mặt khác, đạo hàm khái niệm quan trọng toán học đại, việc ứng dụng đạo hàm tốn học sơ cấp thông qua bất đẳng thức Jensen điều thú vị

Chứng minh Định lý 3. Theo Định lý 5, mục 5), ta có

2(ab+bc+ca)−(a2+b2+c2)

4S = tan A

2 + tan

B

2 + tan

C

2

Vậy áp dụng bổ đề ta suy

2(ab+bc+ca)−(a2+b2+c2) 4S = tan

A 2+tan B 2+tan C ≥ √ 3+1 4[

3√3

2 −(sinA+sinB+sinC)]

Áp dụng tiếp tục bổ đề suy

2(ab+bc+ca)−(a2+b2+c2)

4S ≥

3 + OG

4R2 ,

hay dễ thấy bất đẳng thức tương đương

a2+b2 +c2 ≥

4√3 + OG

R2

S+ (a−b)2 + (b−c)2+ (c−a)2

Đó điều phải chứng minh

Tiếp tục với Định lý Lời giải tác giả báo kết hợp ý tưởng tác giả toán phần đầu sử dụng kết bất đẳng thức hình học [3]

Chứng minh thứ cho Định lý 4. Để ý rằngOI < R, sử dụng cơng thức Euler, ta có

3 + OI

4R2

2

= +

3OI2 2R2 +

OI4

16R4 ≤3 +

OI2

R2 =

4R−2r

(155)

Vậy ta cần chứng minh

2ab+ 2bc+ 2ca−a2−b2−c2 4S

2

≥ 4R−2r

R (∗)

Sử dụng Định lý mục 4), bất đẳng thức tương đương với

4r(4R+r) 4S

2

≥4−2r

R

hay

p2 ≤ R(4R+r)

2

4R−2r

Bất đẳng thức bất đẳng thức điểm Mittenpunk Bài toán [3] Vậy ta kết thúc chứng minh

Lời giải thứ hai Nguyễn Văn Quý, tác giả toán

Chứng minh thứ hai cho Định lý 4. Ta chứng minh bất đẳng thức (∗) biến

đổi đại số Đặt a = m(n+p), b = n(p+m), c = p(m +n), với m, n, p > 0, ta có p =

mn+np+pm, S =pp(p−a)(p−b)(p−c) =mnp√mn+np+pm.Do

2ab+ 2bc+ 2ca−a2−b2−c2

4S =

4mnp(m+n+p) 4mnp√mn+np+pm =

m+n+p

mn+np+pm,

2r

R =

8S2

abcp =

8(p−a)(p−b)(p−c)

abc =

8mnp

(m+n)(n+p)(p+m)

Bất đẳng thức cần chứng minh trở thành

(m+n+p)2

mn+np+pm ≥4−

8mnp

(m+n)(n+p)(p+m),

hay

m2+n2 +p2

mn+np+pm +

8mnp

(m+n)(n+p)(p+m) ≥2

Khơng tổng qt, giả sửm≥n ≥p,ta có m2+n2+p2

mn+np+pm ≥

m2+n2+p2+p2

mn+np+pm+p2 =

m2+n2+ 2p2 (m+p)(n+p)

Ta cần chứng minh

m2+n2+ 2p2 (m+p)(n+p)+

8mnp

(m+n)(n+p)(p+m) ≥2,

(m+n)(m2 +n2+ 2p2) + 8mnp≥2(m+n)(n+p)(p+m),

(m+n−2p)(m−n)2 ≥0.

(156)

Lời kết. Tác giả viết vui sau thời gian dài lại có dịp viết bất đẳng thức hình học giới thiệu báo Epsilon Tốn học nói chung hình học nói riêng có nhiều vẻ đẹp, đại số hình học sơ cấp có nét đẹp đặc trưng khỏc bit Cỏc bt ng thc hỡnh hc ca Weitzenbăock Finsler-Hadwiger thực kết hay có ý nghĩa Chắc chắn bất đẳng thức hình học có ý nghĩa nhiều so với toán khác tự gắn mác "thuần túy hình học" lại bóp méo hình học Nhân dịp này, tác giả cảm ơn học trò Nguyễn Văn Quý trường THPT chuyên KHTN K43 Hiện nay, Quý đồng nghiệp tác giả đồng hành với bất đẳng thức hình học từ ngày cịn học làm thầy Những người học trị u tốn thủy chung với tốn Q động lực lớn cho nghiệp dạy

Tài liệu

[1] Trần Quang Hùng,Một số bất đẳng thức diện tích tam giác, tạp chí Epsilon số 16 [2] Trần Quang Hùng,Bài tốn T12/417, mục đề kỳ này, Tạp chí Toán học tuổi trẻ, tháng

3 năm 2012, số 417

[3] Trần Quang Hùng, Applying R, r, p method in some hard problems, Tạp chí tốn học MathVn, số năm 2009

[4] Wikipedia, Hadwiger–Finsler inequality, https://en.wikipedia.org/wiki/ Hadwiger-Finsler_inequality

[5] Wikipedia, Roland Weitzenbăock, https://en.wikipedia.org/wiki/Roland_ Weitzenbock

(157)

M

ỘT MỞ RỘNG CỦA ĐƯỜNG THẲNG

S

TEINER

Nguyễn Ngọc Giang (ĐH Ngân hàng TPHCM)

Lê Viết Ân (PTNK TPHCM)

TÓM TẮT

Trong số trước tạp chí Epsilon, có mở rộng cho đường thẳng Simson Trong số này, xin giới thiệu tiếp mở rộng cho đường thẳng Steiner Hơn nữa, bên cạnh chúng tơi cịn giới thiệu thêm áp dụng mở rộng

1 Giới thiệu

Định lí đường thẳng Steiner định lí tiếng, nhắc đến [1], [2] [3] Trong [5], định lí phát biểu sau

Định lí 1.1.NếuP là điểm nằm đường trịn ngoại tiếp tam giácABC, ảnh đối

xứng củaP theo thứ tự qua trục đối xứngBC, CAABnằm đường thẳng qua

trực tâm tam giácABC.Đường thẳng gọi làđường thẳng Steiner củaP

với tam giácABC

Và [4] [5], có kết liên quan sau

Định lí 1.2.(N.S Collings) Nếu đường thẳng L đi qua trực tâm tam giácABC, thì

các ảnh đối xứng củaLtheo thứ tự qua trục đối xứngBC, CAABlà đồng quy một

điểm nằm đường tròn ngoại tiếp tam giácABC. Điểm gọi làcực SteinercủaL

đối với tam giácABC Tất nhiên,Llà đường thẳng Steiner củaP tam giácABCkhi

chỉ khiP cực Steiner củaLđối với tam giácABC

Định lí 1.1 mở rộng sau

Định lí 1.3.Cho tam giácABCcó đường tròn ngoại tiếp(O)và trực tâmH Một đường thẳng`

đi quaH Cho điểmP nằm trên(O)và điểmQnằm trên`(Qcó thể điểm vô cực) Các đường

thẳngAQ, BQ, CQtheo thứ tự cắt lại(O)tạiA0, B0, C0 Các đường thẳngP A0, P B0, P C0theo

thứ tự căt `tại AP, BP, CP GọiA0, B0, C0 theo thứ tự đối xứng của AP quaBC, BP qua

CA,CP quaAB Khi bốn điểmA0, B0, C0 Hnằm đường thẳng.

Rõ ràng khiQthuộcP H hoặcQthuộc(O)thì nhận định lí 1.1

(158)

2 Chứng minh định lý 1.3

Chúng ta cần có bổ đề sau

Bổ đề 2.1.Cho tam giácABC nội tiếp đường tròn (O) Một đường thẳng` Cho điểmP nằm

trên (O) và điểm Q nằm trên ` (Q có thể điểm vơ cực) Các đường thẳng AQ, BQ, CQ

theo thứ tự cắt lại (O) tại A0, B0, C0 Các đường thẳng P A0, P B0, P C0 theo thứ tự cắt ` tại AP, BP, CP Khi đường trịn(BCAP),(CABP) (ABCP) có điểm chung nằm

trên`.

Chứng minh.(xem hình 1) GọiA1 :=BCP ∩CBP;Rlà giao điểm thứ hai của(ABCP)và`

Áp dụng định lí Pascal đảo cho sáu điểm

B P C

C0 A

1 B0

với ý ba điểmCP =BA1∩

P C0,Q=BB0∩CC0,BP =P B0∩CA1cùng thuộc đường thẳng`và năm điểmB, P, C, C0, B0

cùng thuộc đường tròn(O) Suy raA1 thuộc(O)

(159)

Sử dụng góc định hướng( mod π), góc hai đường thẳng, ta có

(RA, RBP)≡(RA, RCP) (vìR, BP, CP thẳng hàng)

≡(BA, BCP) (vìB ∈(ARCP))

≡(BA, BA1) (vìB, CP, A1 thẳng hàng)

≡(CA, CA1) (vìC ∈(ABA1))

≡(CA, CBP) (vìC, BP, A1 thẳng hàng)

Điều chứng tỏRthuộc đường tròn(CABP)

Chứng minh tương tự, ta cóRthuộc đường trịn(BCAP)

Bổ đề 2.1 chứng minh

Chứng minh định lí 1.3. (xem hình 2) Theo bổ đề 2.1, có điểm R := ` ∩ (BCAP) ∩ (CABP)∩(ABCP)

VìH thuộc`nên theo định lí 1.2,`có điểm cực SteinerSứng với tam giácABC

Đường thẳngAH cắt(O)tạiAvàA2 Dễ dàng thấy rằngA2 đối xứng vớiHquaBC Do

SA2đối xứng với`quaBC (1)

Suy ba đường thẳngBC, `vàSA2 đồng quy đôi song song

Nếu BC, ` SA2 đôi song song ý bốn điểm (B, C, R, AP) (B, C, S, A2)cũng thuộc đường tròn nênR, S theo thứ tự đối xứng vớiAP, A2 qua trung

trực củaBC Suy raR, S, AP, A2cùng thuộc đường trịn Ngược lại, nếuI :=BC∩`∩SA2

thì theo tính chất phương tích, ta có IR.IAP = IB.IC = IS.IA2 Suy bốn điểm

R, S, AP, A2 thuộc đường tròn

Vậy trường hợp, ta ln có

R, S, AP, A2cùng thuộc đường tròn (2)

Mặt khác, dễ thấy

A2AP đối xứng vớiHA0 quaBC (3)

Ta có,

(RS, `)≡(RS, RAP) (vìR, AP ∈`)

≡(A2S, A2AP) (vì (2))

≡(HA0, `) (vì (1) (3))

Suy raRS song song trùng vớiHA0

Chứng minh tương tự ta cóRSsong song trùng vớiHB0 vàHC0

Do bốn điểmA0, B0, C0vàH thuộc đường thẳng song song trùng vớiRS

(160)

Hình

3 Một áp dụng định lý 1.3

Định lí 3.1.Cho tam giácABCcó đường tròn ngoại tiếp(O)và trực tâmH Một đường thẳng `đi qua H Cho điểmP nằm trên(O)và hai điểmQDcùng nằm trên` Các đường thẳng AQ, BQ, CQtheo thứ tự cắt lại(O)tạiA0, B0, C0 Các đường tròn(A0DP),(B0DP),(C0DP)

theo thứ tự căt `tạiAP, BP, CP GọiA0, B0, C0 theo thứ tự đối xứng củaAP quaBC, BP

quaCA,CP quaAB Khi bốn điểmA0, B0, C0 Hnằm đường thẳng.

•KhiP nằm trên`thì đường thẳngA0, B0, C0, H0 đường thẳng Steiner củaP ứng

với tam giácABC

Chứng minh.(xem hình 3) Đường thẳngDP cắt(O)tạiP vàE; dựng dây cungEF của(O)

song song trùng với`

Sử dụng góc định hướng, ta có

(A0AP, EF)≡(A0AP, `) (vìEF phương với`)

≡(APA0, APD) (vìAP, D ∈`)

≡(P A0, P D) (vìP ∈(A0DAP))

≡(P A0, P E) (vìE ∈DP)

(161)

Hình

Suy haiA0AP vàA0F trùng Do đóF thuộcA0AP

Chứng minh tương tự ta cóF thuộcB0B

P vàC0CP

Áp dụng định lí 1.3 với ý rằngAP =F A0 ∩`,BP =F B0 ∩`và CP =F C0∩`thì ta có

bốn điểmA0, B0, C0 vàHcùng nằm đường thẳng

Định lí 3.1 chứng minh

Tài liệu

[1] Steiner line, available at

http://users.math.uoc.gr/~pamfilos/eGallery/problems/ SteinerLine.html

[2] Steiner line, available at

http://www.xtec.cat/~qcastell/ttw/ttweng/llistes/l_Steiner_ r.html

[3] Droite de Steiner,

https://fr.wikipedia.org/wiki/Droite_de_Steiner

(162)

[5] D Grinberg, Anti-Steiner points with respect to a triangle, preprint 2003, available at

http://www.cip.ifi.lmu.de/~grinberg/geometry2.html

[6] Jean-Pierre Ehrmann, Steiner’s Theorems on the Complete Quadrilateral, Forum Geomet-ricorum, Volume (2004) 35–52

[7] C Pohoata, On the Euler Reflection Point, Forum Geometricorum, Volume 10 (2010) 157–163

[8] P Yiu, Introduction to the Geometry of the Triangle, 2001, new version of 2013,math fau.edu/Yiu/YIUIntroductionToTriangleGeometry130411.pdf

(163)

B

ÀI

T

OÁN

H

AY

L

ỜI

G

IẢI

Đ

ẸP

Ban Biên Tập

GIỚI THIỆU

Chuyên mục lấy cảm hứng từ viết thầy Nguyễn Duy Liên số báo trước toán số6trong kỳ thi IMO2001với 5cách giải khác Mục để dành

viết toán hay, lời giải đẹp câu chuyện thú vị xung quan tốn lời giải

Tên chuyên mục mượn từ tên nhóm người yêu toán Facebook anh Nguyễn Văn Lợi sáng lập “Bài toán hay – Lời giải đẹp – Đam mê toán học” Chuyên mục ghi nhận đề cử bạn đọc chọn đăng kỳ 1, toán Số gặp lại thầy Nguyễn Duy Liên, GV trường THPT chuyên Vĩnh Phúc với câu chuyện toán bất đẳng thức đề thi chọn đội tuyển Iran năm2016

và thầy Trần Nam Dũng với câu chuyện toán đề thi Olympic toán sinh viên quốc tế năm2019:

1 Câu chuyện thầy Nguyễn Duy Liên

Giải toán Đại số hay khó, ta cảm thích thú Nhưng tốn đại số hay khó mà giải nhiều cách mà từ ta giải được, hay tạo số toán lớp tốn niềm vui cịn nhân lên nhiều lần Bài viết này, xin giới thiệu với bạn2cách giải cho toán số1về bất đẳng thức hay khó kỳ thi chọn đội

tuyển dự thi Olympic Iran năm2016:Chúng ta bắt đầu với tốn Bài tốn Gọia; b; c; d là số thực dương cho

1 aC1C

1 bC1 C

1 cC1 C

1

d C1 D2:

Chứng minh rằng

r

a2C1

2 C

r

b2C1

2 C

r

c2C1

2 C

r

d2C1

2 >3

p

(164)

Lời giải Viết bất đẳng thức lại sau

X ra2C1

2

p

a

!

>2Xpa 4:

Theo bất đẳng thức Cauchy-Schwarz ta có

X ra2C1

2

p

a

!

DX

a2C1

2 a

q

a2C1

2 C

p

a

>

2

X a 1/2

r

2a22C1Ca

D 12X.a 1/

2

aC1 D

X

a 3C

aC1

D 12Xa

D2 X a D2 X

.aC1/CX a aC1

D2Xpa 4:

Dấu đẳng thức xẩy khiaDb Dc Dd D1:

Lời giải Ta cần chứng minh

r

x2C1

2

p

x >

xC1 4; 8x > 0: (1)

Nếux D1thì bất đẳng thc (1) ỳng

Nux Ô1;ta bin i (1) nh sau

r

x2C1

2

p

x >2px 4x xC1;

tương đương với

.x 1/2

q

x2C1

2 C

p

x

>

px px 12

xC1 ;

.xC1/ pxC12>4px

r

x2C1

2 C

p

x

!

;

.xC1/2 4x >2px

r

x2C1

2 xC1/

!

;

.x 1/2 >

p

x.x 1/2

q

x2C1

2 C.x C1/

;

xC1C2

r

x2C1

2 >2

p

(165)

Bất đẳng thức cuối theo bất đẳng thức AM-GM Áp dụng (1) ta có

X ra2C1

2

p

a

!

>X 2pa 4a aC1

;

hay

X ra2C1

2 >3

X

a

Bài toán chứng minh

Nhận xét Qua hai lời giải cho ta vẻ đẹp lời giải Lời giải1là kết hợp nhuần

nhuyễn bất đẳng thức cổ điển Lời giải2về giải tích thơng qua phương pháp tiếp tuyến

đánh giá hàm thức biến với hàm phân thức bậc liên quan đến giả thiết sau dùng biến đổi tương đương giải tốn Ngồi cách giải bạn tơi tiếp tục tìm lời giải mới, cho hay khó vốn bạn Cứ thiết nghĩ đời ta đạt lời giải thật ngắn, đẹp đắt Nên ta trân trọng lời giải cá nhân có dài dịng chút đỉnh với phương châm “Cách giải dài với này nhưng ngắn với khác

2 Câu chuyện thầy Trần Nam Dũng

Bài toán nằm đề Olympic sinh viên quốc tế năm2019:Để giải nó, cần đến

số kiến thức cao cấp hàm sinh, phương trình vi phân Tuy nhiên, học sinh phổ thơng có quen biết chút với hàm sinh tích phân hồn tồn hiểu tốt Bài tốn cho thấy bên cạnh ý tưởng chói sáng, kỹ thuật điêu luyện cần thiết để giải tốn thành cơng

Bài toán (IMC 2019) Cho dãy sốa0; a1; : : :được xác định bởi

a0D1; a1 D2.nC3/anC2 D.6nC9/anC1 nan; 8n>0: Chứng minh tất số hạng dãy số nguyên.

Để chứng minh tất số hạng dãy số số nguyên, cách tiếp cận tìm hệ thức truy hồi mà theo đóanC1tính theo số hạng trước với biểu thức dạng

đa thức có hệ số nguyên Sự xuất biểu thức chứantrong hệ số gợi ý cho

(166)

Ta có ý tưởng dùng hàm sinh, từ hệ thức truy hồi tìm phương trình hàm hàm sinh Giải phương trình hàm để tìm hàm sinh Nếu thuận lợi khai triển hàm sinh để tìm cơng thức tổng qt choan(trong trường hợp bước chứng minh biểu thức tường minh

này nguyên) Một hướng khác từ hàm sinh lại tìm cơng thức truy hồi khác phần phân tích

Lời giải Xét hàm sinh dãy.an/W

f x/D

X

nD0

anxn:

Ta có

f x/D1C2xC

X

nD2

anxnD1C2xC

1

X

nD0

anC2xnC2

D1C2xC

X

nD0

6nC9 nC3 anC1x

nC2

1

X

nD0

n nC3anx

nC2

:

Đặt

f1.x/D

1

X

nD0

6nC9 nC3 anC1x

nC2

; f2.x/D

1

X

nD0

n nC3anx

nC2

:

Khi

.xf1.x//0D

1

X

nD0

.6nC9/xnC2 D6x2

X

nD0

.nC1/anC1xnC3x

1

X

nD0

anC1xnC1

D6x2f0.x/C3x.f x/ 1/;

.xf2.x//0 D

1

X

nD0

nanxnC2 Dx3

1

X

nD0

nanxn 1Dx3f0.x/:

Từ hệ thức này, ta đến phương trình vi phân sau củaf xf x//0D1C4xC.xf1.x//

0

.xf2.x//

0

D1C4xC6x2f0.x/C3x.f x/ 1/ x3f0.x/;

hay

.x3 6x2Cx/f0.x/C.1 3x/f x/ x D0:

Viết dạng tắc sau

f0.x/C 3x

x3 6x2Cxf x/D

1Cx x3 6x2Cx:

Phương trình thuộc dạng vi phân tuyến tính bậc

(167)

Phương trình giải phương pháp thừa số tích phân sau

Ta nhân hai vế phương trình vớic.x/là hàm số ta chọn thích hợp, để trở thành c.x/f0.x/Cc.x/a.x/f x/Dc.x/b.x/:

Ta muốn vế trái có dạng.c.x/f x//0:Muốn ta phải cóc0.x/ D c.x/a.x/hay ln.c.x///0 D a.x/:Như vậyc.x/ D exp R a.x/dx;chính thừa số tích phân cần tìm Từ

đây ta tiếp tục tìm

f x/D

R

c.x/b.x/dx

c.x/ :

Như vậy, để tìm thừa số tích phân, ta cần tính tích phân hàm hữu tỷ 3x

x3 6x2Cx:Để tính tích phân

này, ta tìm khai triển biểu thức dạng tổng phân số đơn giản, cụ thể

1 3x x3 6x2Cx D

A x C

B x x1 C

C x x2

:

ở đâyx1; x2là hai nghiệm phương trìnhx2 6xC1D0:

Đồng hệ số, ta hệ điều kiện sau

8 ˆ ˆ < ˆ ˆ :

AD1

ACBCC D0

–6A–B–2C D–3

Hay làAD1; B DC D–12:Từ ta tính

Z 1 3x

x3 6x2Cx D

Z 1

x

1 2.x x1/

1 2.x x2/

dx Dlnp x

x2 6xC1;

c.x/D p x

x2 6xC1:

Tiếp tích phânc.x/b.x/;trong tình tích phân

Z 1Cx

.x2 6xC1/32

dx:

Để tính tích phân dạng này, ta dự đốn có dạng

g.x/D axCb x2 6xC1/12

:

Lấy đạo hàm củag.x/;ta

g0.x/D

a.x2 6xC1/

1

2

2x

.x2 6xC1/12.axCb/

x2 6xC1 D

a.x2 6xC1/ .x 3/.axCb/

.x2 6xC1/32

(168)

Để kết

1Cx x2 6xC1/32

;

ta cần có 8

< :

aC3bD1

6aC3a bD1

Từ đâyaD 12; bD 12:Vì

Z 1Cx

.x2 6xC1/32

dxD

1 2xC

1

p

x2 6xC1 CC:

Và từ

f x/D

1 2xC

1 2CC

p

x2 6xC1

x :

Choxdần về0với ýf 0/D1;ta tìm đượcC D 12:Từ f x/D xC1

p

x2 6xC1

2x :

Tìm khai triển Taylor hàmpx2 6xC1là khơng khả thi, hướng tiếp cận tìm cơng

thức tường minh cho an loại bỏ Ta thấyf x/có dạng nghiệm phương trình bậc

hai Bằng cách tính ngược từ công thức nghiệm

x1;2D

b˙pb2 4ac

2a ;

ta dễ dàng tìm phương trình

xf2.x/C.x 1/f x/C1D0:

Bây thay công thức hàm sinh vào, ta

x

X

nD0

anxn

!2

C.x 1/

1

X

nD0

anxn

!

C1D0:

Tính hệ số củaxnC1ở vế trái, ta

n

X

iD0

aian i Can anC1 D0:

Từ ta có

anC1 DanC n

X

iD0

aian i:

(169)

V

K

T

HI

T

HÁCH

T

HỨC

T

OÁN

H

ỌC

T

RỰC

T

UYẾN

BTC kỳ thi Thách Thức Toán Học

1 Ý tưởng hình thành kỳ thi

Dưới ảnh hưởng dịch COVID-19, thời gian hoạt động giáo dục trực tuyến đẩy mạnh Tuy nhiên, dạy học trực tuyến cách đơn thời gian dài, hứng thú tinh thần học học sinh nhiều bị chùng xuống, động lực dạy học thầy cô học sinh phần bị ảnh hưởng Điều dẫn đến cần thiết có sân chơi cho bạn học sinh, giúp tạo thêm hứng khởi, giữ vững khuyến khích tinh thần học tập cho bạn thời gian nghỉ dài Ý tưởng cho kỳ thi online Việt cho bạn học sinh từ lớp đến lớp 12 thành viên thuộc dự án BM2E Bring Math to Everyone hình thành từ

Hình 1: Logo Thách thức Toán học trực tuyến

2 Cách thức đăng kí

(170)

Hình 2: BM2E (Bring Math to Everyone): Đơn vị tổ chức kỳ thi

Trong vịng 48 sau đăng kí, thí sinh giáo viên đăng kí nhận email phản hồi từ ban tổ chức, kèm thông tin chuyển khoản lệ phí thi (100,000 VNĐ) Sau nhận email phản hồi thơng tin chuyển khoản, thí sinh đăng kí cá nhân tiến hành chuyển khoản theo cú pháp sau đây:

[Họ tên thí sinh][Lớp][Ngày sinh][LephithiOMC2020]

Đối với giáo viên đăng kí tập thể, sau nhận email phản hồi thông tin chuyển khoản, quý thầy cô tiến hành chuyển khoản theo cú pháp đây:

[Họ tên giáo viên đăng kí][Đơn vị][Số lượng thí sinh đăng kí][LephithiOMC2020]

3 Hình thức thi

3.1 Vịng 1: Tournament

Thí sinh sau hồn tất thủ tục đăng kí (điền form, chuyển khoản) nhận tài khoản (tên đăng nhập + mật khẩu) hướng dẫn truy cập vào hệ thống thi online muộn 24 trước ngày thi

Vào lúc19 W 30ngày 26=04=2020, thí sinh đăng nhập vào hệ thống theo hướng dẫn

(171)

3.2 Vòng 2: Super Cup

Sau cơng bố kết quả, 8% thí sinh cao điểm khối lớp tiếp tục thi vòng Super Cup Cách thức thi vịng thơng báo sau tới thí sinh đủ điều kiện

4 Tinh thần kỳ thi

Với cách thức tổ chức trực tuyến, thành công kỳ thi phụ thuộc nhiều vào tinh thần tự giác thí sinh Yêu cầu quan trọng BTC thí sinh làm độc lập, trung thực, không nhận trợ giúp từ người khác Chúng ta tham gia với tinh thần thoải mái, mục đích học hỏi, tạo thêm động lực giữ vững tinh thần thời gian nghỉ dài, để thi đấu tranh đua với

Tuy nhiên, Ban tổ chức cần có biện pháp riêng để phát trường hợp khơng trung thực thời gian thi Nếu Ban tổ chức phát thi nhóm thi phát có dấu hiệu gian lận

5 Cơ cấu giải thưởng

1 Tất thí sinh đăng kí tham dự nhận tài khoản VIP trang TaiLieu.vn

https://tailieu.vn/(trị giá 120,000đ/tài khoản) Cơ cấu giải thưởng cho vòng Tournament:

Giải (8% thí sinh nhóm lớp):Giấy chứng nhận, quyền thi tiếp vòng Super Cup, học bổng số môn học thuộc nhóm CC1 Đại học Trực tuyến FUNiX lớp Scratch Đại học Trực tuyến FUNiX mentor Trương Đắc Tài đồng tài trợ Chi tiết xem trang sau

Giải nhì (12% thí sinh nhóm lớp):Giấy chứng nhận, quà tặng từ Sputnik Edu-cation

Giải ba (20% thí sinh nhóm lớp):Giấy chứng nhận

(172)

6 Đề thi

Đề thi Thách thức Toán học trực tuyến gồm ba phần:

1 Phần A gồm câu hỏi trắc nghiệm với phương ánA; B; C; D; E Thí sinh chọn

5 phương án để trả lời Trả lời điểm, trả lời sai không trả lời không bị trừ điểm

2 Phần B gồm câu hỏi điền đáp số Thí sinh tính tốn điền đáp số để trả lời Lưu ý điền đáp số, thí sinh điền số, dấu chấm, dấu phẩy, kí hiệu tốn học, khơng điền kí tự chữ

3 Phần C gồm tốn có nhiều câu hỏi điền đáp số nhỏ Thí sinh làm theo yêu cầu cụ thể đề thi nhóm thi, nhóm thi thí sinh u cầu điền đáp số theo cách khác

Dưới Ban tổ chức xin đăng số câu hỏi mẫu để bạn học sinh nhóm thi tham khảo

6.1 Nhóm lớp 3-4

Câu 1. Tìm chữ số tận tích sau:

123 1415:

A B C D E Đáp số khác

Câu 2. Có số có chữ số chia hết cho lập từ chữ số0; 1; 2; 3; 4; 5? (số

chia hết cho5là số có tận là0hoặc5)

A B 10 C 11 D 12 E Đáp số khác

Câu 3. Tìm số cịn thiếu dãy số sau:

(173)

A 24và30 B 22và28 C 21và27 D 23và29 E Đáp số khác

Câu 4. Số lớn có5chữ số mà tổng chữ số là27là:

A 98765 B 98730 C 99900 D 99999 E Đáp số khác

Câu 5. Cả lớp 4A, 4B, 4C trồng được120 Lớp 4B trồng nhiều lớp 4A

nhưng lại lớp 4C Hỏi lớp 4C trồng cây?

Câu 6. Chu vi hình tơ màu đỏ bao nhiêu?

10cm

10cm

Câu 7. Biết hình thang hình sau giống nhau, tính diện tích hình chữ nhật lớn

(1)

(2)

(3) (4)

4cm

A B

C D

P Q

M

N

Câu 8. Có30câu hỏi thi tốn Mỗi câu trả lời cộng thêm5điểm

mỗi câu trả lời sai bị trừ đi2điểm Nếu Colin được115điểm bạn trả lời sai

câu?

Câu 9. Sudoku trò chơi câu đố xếp chữ số dựa logic theo tổ hợp Mục tiêu trò chơi điền chữ số vào lưới99sao cho cột, hàng, phần số

(174)

cả chữ số từ tới Câu đố hoàn thành phần, người chơi phải giải tiếp việc điền số Mỗi câu đố thiết lập tốt có cách làm

Sudoku có nhiều biến thể kích thước lưới vng, cách điền số, Biến thể sau biển thể kích thước, lưới99được thay lưới66được chia thành lưới 32

a b

4 c d e f

g h i

5 j k

l m n o

p q r s t

Hãy điền số thích hợp vào chứa chữ lưới66trên cho hàng, cột

mỗi hình chữ nhật32đều có đủ số từ1đến6

6.2 Nhóm lớp 5-6

Câu 1. Hỏi tổng chữ số tận tích sau bao nhiêu?

1234&1415

A B C 10 D 12 E Đáp số khác

Câu 2. Các chữ số1; 2; 3; 4; 5; 6mỗi chữ số dùng lần để tạo hai số Hỏi tổng

của hai số nhỏ bao nhiêu?

A 379 B 381 C 579 D 480 E Đáp số khác

Câu 3. Có số có chữ số khác mà có tích chữ số là18? Ví dụ36; 129; :::

A B 16 C 18 D 20 E Đáp số khác

Câu 4. Cho hình bên Ta cần đổi chỗ hai số tổng số hàng, cột hai đường chéo nhau; hỏi tổng hai số bao nhiêu?

13 16

8 10

12 11

(175)

A 10 B 12 C 16 D 20 E Đáp số khác

Câu 5. Hôm thầy hái tổng cộng240kg trái bao gồm bưởi, cam chanh Biết

tổng khối lượng cam chanh nặng gấp đôi bưởi Tổng khối lượng cam nặng chanh40kg

Hỏi thầy hái kg cam?

Câu 6. Ta có hai bìa hình chữ nhật có chiều rộng có chiều dài là8cm 10cm Người ta tạo hình chữ nhật cách chồng hai bìa lên nhau, biết

phần hình chồng lên (phần tơ đậm) hình vng (như hình vẽ bên dưới) Tính chu vị hình chữ nhật

8cm 10cm

Câu 7. Nhân dịp khai trương có chương trình giảm giá, mẹ mua xe đạp cho Nhi Mẹ thấy xe đạp cần mua giảm 20% nên số tiền mẹ mang theo dư 260; 000 VNĐ

Nhưng khơng giảm (mua với giá bình thường) số tiền mẹ thiếu40; 000VNĐ Hỏi giá

bình thường (chưa giảm) xe đạp bao nhiêu?

Câu 8. Hai xe A B ngược chiều từ đoạn đường Xe A chạy với vận tốc50km/giờ, xe B chạy với vận tốc45km/giờ Hai xe gặp vị trí cách điểm

một đoạn10km Tính chiều dài đoạn đường? (đơn vị km)

Câu 9. Ta điền vào điểm có tọa độ nguyên mặt phẳng tọa độ số nguyên dương

1; 2; 3; 4; :::theo hình xoắn trôn ốc sau: số1điền vào điểm.0; 0/, số2điền vào điểm.0; 1/,

số3điền vào điểm.1; 1/, số 4điền vào điểm.1; 0/, số5điền vào điểm.1; 1/, số6điền vào

(176)

6 1

6 1

10 111213 92314 81415 7651617

1 Cho biết điểm.3; 3/được điền số mấy?

2 Cho biết số30được điền vào điểm có tọa độ nào? Viết hai tọa độ phân biệt dấu

phẩy, hoành độ trước, tung độ sau Điểm.5; 5/được điền số mấy?

4 Điểm.10; 5/được điền số mấy?

5 Số2020được điền vào điểm có tọa độ nào?

6.3 Nhóm lớp 7-8

Câu 1. Có số khối vng đỏ số khối vuông xanh, khối vuông có khả chuyển màu (từ đỏ sang xanh từ xanh sang đỏ) Nếu có

7 số khối vng đỏ chuyển sang

màu xanh có

(177)

vng đỏ hốn đổi cho Hỏi tỉ lệ số khối vuông xanh tổng số khối vuông ban đầu bao nhiêu?

A

12 B

1

2 C

5

12 D

2

3 E Đáp số khác Câu 2. Cho tam giácABC vuông cân tạiAcóAB D4p3;trung tuyếnBE:Lấy điểmF

cạnhBC choFE ?BE:Tính diện tích tam giácCEF:

B

A E C

F

A B p3 C D 2p3 E Đáp số khác

Câu 3. Cho số thựcx; y thay đổi thỏa mãn4x2Cy2 D4x 2yC7:Tìm giá trị lớn

của5xC6y:

A 16 B 17 C 18 D 19 E Đáp số khác

Câu 4. Trong hình vẽ bên dưới, số tự nhiên 1; 2; : : : ; điền vào hình lục giác

đều cho tổng tất số lục giác cạnh tam giác lớn bằng19:

4

2

9

Bây giờ, đổi chỗ số cho tổng tất số lục giác cạnh tam giác lớn Gọi M tổng chung lớn thu m tổng chung nhỏ thu

được (tổng chung tổng tất số lục giác cạnh tam giác lớn) Tính

M Cm:

A 40 B 41 C 42 D 43 E Đáp số khác

(178)

Câu 6. Có1082người tham gia bầu cử để chọn ra14đại biểu Hỏi đại biểu cần bao

nhiêu phiếu bầu để chắn trúng cử, số đại biểu lớn hơn14;mỗt người bầu cho

đại biểu phiếu bầu hợp lệ?

Câu 7. Cho số nguyênakhác0:Biết rằngP2C695chia hết choavới số nguyên tốP lớn

hơn3:Tìm giá trị lớn củaa:

Câu 8. Có trung đội xếp thành hàng dài50m tiến phía trước với tốc độ khơng

đổi phía sau, người huy chạy lên gửi thơng tin đến vị trí đầu hàng, sau quay trở lại vị trí cuối hàng Biết vận tốc người huy không đổi trình di chuyển tính từ lúc người huy bắt đầu chạy đến lúc người huy quay cuối hàng đội được100m:Hỏi người huy quãng đường bao nhiêu?

Câu 9. Ta điền vào điểm có tọa độ nguyên mặt phẳng tọa độ số nguyên dương

1; 2; 3; 4; :::theo hình xoắn trơn ốc sau: số1điền vào điểm.0; 0/, số2điền vào điểm.0; 1/,

số3điền vào điểm.1; 1/, số 4điền vào điểm.1; 0/, số5điền vào điểm.1; 1/, số6điền vào

điểm.0; 1/; :::

6 1

6 1

(179)

1 Cho biết điểm.3; 3/được điền số mấy?

2 Cho biết số30được điền vào điểm có tọa độ nào? Viết hai tọa độ phân biệt dấu

phẩy, hoành độ trước, tung độ sau Điểm.5; 5/được điền số mấy?

4 Điểm.10; 5/được điền số mấy?

5 Số2020được điền vào điểm có tọa độ nào?

6.4 Nhóm lớp 9-10

Câu 1. Với số nguyên dươngn, ta ký hiệunŠ tích số nguyên dương từ 1đến n Ví dụ 4Š D 1:2:3:4 D24 Ta quy ước0Š D1 Vớin >2, biểu thức n 2/Š.nC1/Š

.nŠ/2 rút

gọn lại thành:

A

n B

1

n C

n

n.nC1/ D

nC1 n 1/n

E Đáp án khác

Câu 2. Trong hình vẽ đây, chu vi tam giácPQRbằng bao nhiêu?

5

13

37

R

Q P

(180)

A 55 B 74 C 80 D 84 E Đáp án khác

Câu 3. Có viên xúc xắc đặc biệt với mặt ghi số2; 2; 3; 3; 5và8 Người ta tung

xúc xắc hai lần cộng kết hai lần tung lại Hỏi ta thu tổng khác nhau?

A B C D E Đáp án khác

Câu 4. Có số nguyên dương nhỏ hay 100và có 12 ước số nguyên

dương?

A B C D E Đáp án khác

Câu 5. Ta định nghĩa phép toán hai số thựcx; ynhư sau xy DaxCbyCc;

trong đóa; bvàc số Nếu35D16và47D30, thì11bằng bao nhiêu?

Câu 6. Tổng số số nguyên dương phân biệt bằng30:Hỏi tích chúng lớn

bao nhiêu?

Câu 7. Trong hộp có8viên bi màu vàng,7viên bi màu đỏ và5viên bi màu xanh Hoàng bốc

raN viên bi mà khơng nhìn Hồng muốn cho dù hộp cịn lại 4viên bi màu nhất3viên bi màu khác HỏiN lớn

bao nhiêu?

Câu 8. Trong hình chữ nhật ABCD ta có AB D 8; BC D H điểm trênBC cho BH D 6; E điểm trênAD choDE D Các đường thẳngEC vàAH cắt tạiG,

(181)

4

6

A B C

D E

H G

F

Câu 9. Ta điền vào điểm có tọa độ nguyên mặt phẳng tọa độ số nguyên dương

1; 2; 3; 4; :::theo hình xoắn trơn ốc sau: số1điền vào điểm.0; 0/, số2điền vào điểm.0; 1/,

số3điền vào điểm.1; 1/, số 4điền vào điểm.1; 0/, số5điền vào điểm.1; 1/, số6điền vào

(182)

6 1

6 1

10 111213 92314 81415 7651617

1 Cho biết điểm.3; 3/được điền số mấy?

2 Cho biết số30được điền vào điểm có tọa độ nào? Viết hai tọa độ phân biệt dấu

phẩy, hoành độ trước, tung độ sau Điểm.10; 10/được điền số mấy?

4 Số2020được điền vào điểm có tọa độ nào?

Ngày đăng: 08/02/2021, 07:40

TỪ KHÓA LIÊN QUAN

w